高考英语试题分类汇编阅读理解

申明敬告: 本站不保证该用户上传的文档完整性,不预览、不比对内容而直接下载产生的反悔问题本站不予受理。

文档介绍

高考英语试题分类汇编阅读理解

2017 年高考英语试题分类汇编——阅读理解 (2016 湖南) A L1PITOR ABOUT LIPITOR Lipitor is a prescription medicine.Along with diet and exercise,it lowers “bad,’ cholesterol(胆固醇)in your blood.It can also raise “good'’ cholesterol. [ Lipitor can lower the risk of heart attack in patients with several common risk factors, including family history of early heart disease,high blood pressure,age and smoking. [ WHO IS LIPITOR FOR? Who can take LIPITOR: .People who cannot lower their cholesterol enough with diet and exercise · Adults and children over l0 Who should NOT take LIPITOR: .Women who are pregnant,may be pregnant,or may become pregnant. Lipitor may harm your unborn baby. [ .women who are breast-feeding.Lipitor can pass into your breast milk and may harm your baby. [ · People with liver(肝脏)problems POSSIBLE SIDE EFFECTS OF LIPITOR Serious side effects in a small number of people: .Muscle(肌肉)problems that can lead to kidney(肾脏)problems,including kidney failure .Liver problems.Your doctor may do blood tests to check your liver before you start Lipitor and while you are taking it. Call your doctor right away if you have: .Unexplained muscle pain or weakness,especially if you have a fever or feel very fired .Swelling of the face,lips,tongue,and/or throat that may cause difficulty in breathing or swallowing · Stomach pain Some common side effects of LIPITOR are: · Muscle pain · Upset stomach · Changes in some blood tests HOW TO TAKE LIPITOR DO: .Take Lipitor as prescribed by your doctor. .Try to eat heart-healthy foods while you take Lipitor. [ .Take Lipitor at any time of day, with or without food. [ .If you miss a dose(一剂),take it as soon as you remember. [But if it has been more than 12 hours since your missed dose,wait.Take the next dose at your regular time. [ Don’t: .Do not change or stop your dose before talking to your doctor. [ .Do not start new medicines before talking to your doctor. [ 56. What is a major function of Lipitor? A. To help quit smoking. B. To control blood pressure. C. To improve unhealthy diet. D. To lower "bad" cholesterol. 57. Taking Lipitor is helpful for . A. breast-feeding women B. women who are pregnant C. adults having heart disease D. teenagers with liver problems. 58. If it has been over 12 hours since you missed a dose, you should A. change the amount of your next dose B. eat more when taking your next dose C. have a dose as soon as you remember D. take the next dose at your regular time 59. Which of the following is a common side effect of taking Lipitor? A. Face swelling. B. Upset stomach. C. Kidney failure. D. Muscle weakness. 60. What is the main purpose of the passage? A. To teach patients ways for quick recovery. B. To present a report on a scientific research. C. To show the importance of a good lifestyle. D. To give information about a kind of medicine. 本文为药品说明,主要介绍了药品 Lipitor 的作用、适用人群、副作用及服药的相关注意事项。 56. 答案:D 考点::细节理解 解析:根据 “Along with diet and exercise,it lowers “bad,’cholesterol(胆固醇)in your blood.It can also raise “good'’ cholesterol. [”可以推断出答案。 57. 答案:C 考点::细节理解 解析:根据 “Lipitor can lower the risk of heart attack in patients with several common risk factors, including family history of early heart disease,high blood pressure,age and smoking. [”和 “Adults and children over l0”可以判断选 C 58. 答案:D 考点::细节理解 解析:根据 “If you miss a dose(一剂),take it as soon as you remember. [But if it has been more than 12 hours since your missed dose,wait.Take the next dose at your regular time. [”可以得出答 案 59. 答案:B 考点::细节理解 解析:根据 “Some common side effects of LIPITOR are:Muscle pain, Upset stomach, Changes in some blood tests” 可以得出答案 60. 答案:D 考点::推理判断题 解析:文章先总体介绍药品 Lipitor 的作用,接着介绍药品的适用人群及不适用人群,然后介 绍药品的副作用,最后介绍服药的注意事项,由此可以判断选 D 项。 B When Mary Moore began her high school in 1951, her mother told her, "Be sure and take a typing course so when this show business thing doesn't work out, you'll have something to rely on. " Mary responded in typical teenage fashion. From that moment on, "the very last thing I ever thought about doing was taking a typing course," she recalls. The show business thing worked out, of course. In her career, Mary won many awards. Only recently, when she began to write Growing Up Again, did she regret ignoring her morn," I don't know how to use a computer," she admits. Unlike her 1995 autobiography, After All, her second book is less about life as an award-winning actress and more about living with diabetes (糖尿病). All the money from the book is intended for the Juvenile Diabetes Research Foundation (JDRF), an organization she serves as international chairman. "I felt there was a need for a book like this," she says. " I didn't want to lecture, but I wanted other diabetics to know that things get better when we're self-controlled and do our part in managing the disease. " But she hasn't always practiced what she teaches. In her book, she describes that awful day, almost 40 years ago, when she received two pieces of life-changing news. First, she had lost the baby she was carrying, and second, tests showed that she had diabetes. In a childlike act, she left the hospital and treated herself to a box of doughnuts (甜甜圈). Years would pass before she realized she had to grow u p ---again---and take control of her diabetes, not let it control her. Only then did she kick her three-pack-a-day cigarette habit, overcome her addiction to alcohol, and begin to follow a balanced diet. Although her disease has affected her eyesight and forced her to the sidelines of the dance floor, she refuses to fall into self-pity. "Everybody on earth can ask, 'why me?' about something or other," she insists. "It doesn't do any good. No one is immune (免疫的) to heartache, pain, and disappointments. Sometimes we can make things better by helping others. I've come to realize the importance of that as I've grown up this second time. I want to speak out and be as helpful as I can be. " 61. Why did Mary feel regretful? A. She didn't achieve her ambition. B. She didn't take care of her mother. C. She didn't complete her high school. D. She didn't follow her mother's advice. 62, We can know that before 1995 Mary A. had two books published B. received many career awards C. knew how to use a computer D. supported the JDRF by writing 63. Mary's second book Growing Up Again is mainly about her . A. living with diabetes B. successful show business C. service for an organization D. remembrance of her mother 64. When Mary received the life-changing news, she . A. lost control of herself B. began a balanced diet C. Med to get a treatment D. behaved in an adult way 65. What can we know from the last paragraph? A. Mary feels pity for herself. B. Mary has recovered from her disease. C. Mary wants to help others as much as possible. D. Mary determines to go back to the dance floor. 【语篇解读】本文为人物介绍说明文。主要介绍了玛丽·摩尔的职业生涯及抗病之争。 61. 答案 D 考点::细节理解题。 解析:根据第二段“Only recently,when she began to write Growing Up Again,did she regret ignoring her mom,‘I don t know how to use a computer,’she admits. ”可判断选 D 项。 62. 答案 B 考点::细节理解题。 解析:根据第二段“The show business thing worked out,of course. In her career,Mary won many awards. ”可判断选 B 项。 63. 答案 A 考点::细节理解题。 解析:根据第三段“her second book is less about life as an award winning actress and more about living with diabetes (糖尿病). ”可判断选 A 项。 64. 答案 A 考点::细节理解题。 解析:根据第四段“In a childlike act,she left the hospital and treated herself to a box of doughnuts (甜甜圈). Years would pass before she realized she had to grow up again—and take control of her diabetes,not let it control her. ”可判断选 A 项。 65. 答案 C 考点::推理判断题。 解析:根据最后一段中的 Mary Moore 所说的话“I ve come to realize the importance of that as I ve grown up this second time. I want to speak out and be as helpful as I can be. ”可推断她想尽 可能地去帮助他人,故选 C 项。 C People from East Asia tend to have more difficulty than those from Europe in distinguishing facial expressions — and a new report published online in Current Biology explains why. Rachael Jack, University of Glasgow researcher, said that rather than scanning evenly(均匀 的) across a face as Westerners do, Easterners fix their attention on the eyes. "We show that Easterners and Westerners look at different face features to read facial expressions," Jack said. "Westerners look at the eyes and the mouth in equal measure, whereas Easterners favor the eyes and neglect (忽略) the mouth. " According to Jack and her colleagues, the discovery shows that human communication of emotion is more complex than previously believed. As a result, facial expressions that had been considered universally recognizable cannot be used to reliably convey emotion in cross-cultural situations. The researchers studied cultural differences in the recognition of facial expressions by recording the eye movements of 13 Western Caucasian and 13 East Asian people while they observed pictures of. expressive faces and put them into categories: happy, sad, surprised, fearful, disgusted, angry, or neutral. They compared how accurately participants read those facial expressions using their particular eye movement strategies. It turned out that Easterners focused much greater attention on the eyes and made significantly more errors than did Westerners. "The cultural difference in eye movements that they show is probably a reflection of cultural difference in facial expressions," Jack said. "Our data suggest that whereas Westerners use the whole face to convey emotion, Easterners use the eyes more and mouth less. " In short, the data show that facial expressions are not universal signals of human emotion. From here on, examining how cultural factors have diversified these basic social skills will help our understanding of human emotion. Otherwise, when it comes to communicating emotions across cultures, Easterners and Westerners will find themselves lost in translation. 66. The discovery shows that Westerners . A. pay equal attention to the eyes and the mouth B. consider facial expressions universally reliable C. observe the eyes and the mouth in different ways D. have more difficulty in recognizing facial expressions 67. What were the people asked to do in the study? A. To make a face at each other. B. To get their faces impressive. C. To classify some face pictures. D. To observe the researchers' faces. 68. What does the underlined word "they" in Paragraph 6 refer to? A. The participants in the study. B. The researchers of the study. C. The errors made during the study. D. The data collected from the study. 69. In comparison with Westerners, Easterners are likely to . A. do translation more successfully B. study the mouth more frequently C. examine the eyes more attentively D. read facial expressions more correctly 70. What can be the best title for the passage? A. The Eye as the Window to the Soul B. Cultural Differences in Reading Emotions C. Effective Methods to Develop Social Skills D. How to Increase Cross-cultural Understanding 【语篇解读】本文为科普说明文。主要介绍了最新的科学研究发现:东方人比西方人更难于 读懂他人的面部表情。 66. 答案:A 考点::细节理解题 解 析 : 根 据 第 三 段 “Westerners look at the eyes and the mouth in equal measure, whereas Easterners favor the eyes and neglect (忽略) the mouth. ” 可以得出答案 67. 答案:C 考点::细节理解题 解析:根据第五段 “…by recording the eye movements of 13 Western Caucasian and 13 East Asian people while they observed pictures of. expressive faces and put them into categories: happy, sad, surprised, fearful, disgusted, angry, or neutral. ”可以判断选 C 68. 答案:A 考点::词义猜测 解析:根据定语从句 “that they show”所修饰的 eye movements 在研究中为 the participants 所 作(从 the eye movements of 13 Western Caucasian and 13 East Asian people”可知)可判断选 A 69. 答案:C 考点::细节理解题 解析:根据第六段 “It turned out that Easterners focused much greater attention on the eyes and made significantly more errors than did Westerners. ”可判断选 C 70. 答案:B 考点::主旨大意题 解析:文章首先指出科学研究的最新发现:东方人比西方人更难于读懂人的面部表情。之后 更具体地介绍研究的结构及其研究过程,最后得出结论:文化差异丰富了理解情感的基本社 会技巧,即:不同的文化背景使人理解他人情感的方式也不尽相同。由此判断最佳标题应为 B Part 1V Writing (45 marks) Section A (2016 marks) Directions: Read the following passage. Fill in the numbered blanks by using the information from the passage. Write NO MORE THAN THREE WORDS for each answer. An apprenticeship is a form of on-the-job training that combines workplace experience and classroom learning. It can last anywhere from one to six years, but four years is typical for most. An apprentice spends the majority of the time in a workplace environment learning the practical skills of a career from a journeyman-- someone who has done the job for many years. The rest of the apprentice's time is spent in a classroom environment learning the theoretical skills the career requires. Being an apprentice is a full-time undertaking. One of the advantages of apprenticeship is that it does not cost apprentices anything. The companies that hire them pay for school. What's more, it offers apprentices an "earn while you learn" opportunity. They usually start out at half the pay of a journeyman, and the pay increases gradually as they move further along in the job and studies. Near the end of the apprenticeship, their wages are usually 90 percent of what a journeyman would receive. Apprenticeship also pays off for employers. It can offer employers a pool of well-trained workers to draw from. Despite the advantages, apprentices are usually required to work during the day and attend classes at night, which leaves little time for anything else. Sometimes, they might be laid off(下岗) if business for the employers is slow. Once they have completed the apprenticeship and become journeymen, they receive a nationally recognized and portable certification and their pay also increases again. Some journeymen continue employment with the companies they apprenticed with; others go onto different companies or become self-employed contractors. 71. Apprenticeship 72. classroom learning 73. the majority 74. theoretical 75. at the beginning 76. a jouneyman’s wages 77. many well-trained workers. 78. being laid off 79. Results 80. another pay increase 71. 答案 Apprenticeship 解析:全文是围绕“Apprenticeship”而展开描写的,故填 Apprenticeship。 72. 答案 classroom learning[ 解析:根据第一段第一句“An apprenticeship is a form of on the job training that combines workplace experience and classroom learning. ”可判断。 73. 答案 the majority 解析:根据第一段“An apprentice spends the majority of the time in a workplace environment learning the practical skills of a career from a journeyman”可判断。 74. 答案 theoretical 解析:根据第一段“The rest of the apprentice s time is spent in a classroom environment learning the theoretical skills the career requires. ”可判断。 75. 答案 at the beginning 解析:根据第二段“They usually start out at half the pay of a journeyman,and the pay increases gradually as they move further along in the job and studies. ”可判断。 76. 答案 a journeyman’s wages 解析:根据第二段“Near the end of the apprenticeship,their wages are usually 90 percent of what a journeyman would receive. ” 可判断作答 77. 答案:many well trained workers 解析:根据第二段 “It can offer employers a pool of well-trained workers to draw from. ”可判断 78. 答案: being laid off 解析: 根据第三段 “Sometimes, they might be laid off(下岗) if business for the employers is slow. ” 可判断 79. 答案: Results 解析:综合右栏内容可判断 80. 答案:another pay increase 解析:根据最后一段 “Once they have completed the apprenticeship and become journeymen, they receive a nationally recognized and portable certification and their pay also increases again. ”可判断 Section B (2016 marks) Directions: Read the following passage, Answer the questions according to the information given in the passage. Secret Santas On Christmas morning, Linda wakes up, and tries to imagine the wide-eyed surprise of children in another household as they unwrap the presents she carefully chose for them. Linda has never met the children, but that’s all part of the joy of giving as secret Santas, she says. "It's an amazing feeling to buy gifts on an anonymous (匿名的) basis," says Linda. "It brings a whole new meaning to the holidays. " Linda and Tony are an American couple living in Toronto, Canada, and Linda did charitable work as a member of the American Women's Club of Toronto. As the name suggests, members are U. S. citizens living in Toronto, who join together for fellowship and community service. To find her "adopted" family, Linda goes to the local schools and requests a wish list for a family that's struggling to survive. Last year she helped a single mother with three children. The mother works as a cleaning lady in a nursing home. "The list is always heartbreaking. They have an opportunity to ask for anything and do just the opposite, asking for basic clothes or simple toys," she says. "We always buy the kids a new winter coat, hats, and gloves. " She also buys gifts for the parents. Last year Linda asked the mother for a second wish list--one that didn't include the basics. "Every child should have a Christmas that sticks with them for a lifetime. " She purchased iPods for the two older children and a video game system for the youngest. "I have learned a very valuable lesson in all of this," says Linda. "Pay attention to what's going on in your own backyard--no matter where you live. " The joy of giving as secret Santas is much sweeter when the gift is anonymous. 81. What reaction does Linda imagine the children will have? (No more than 5 words) (2 marks) They will feel greatly surprised. 82. Why did Linda join the American Women's Club of Toronto? (No more than 2016 words) (2 marks) She joined it for fellowship and community service. 83. Why did Linda ask for a second wish list? (No more than 15 words) (3 marks) She wanted to give the children some other gifts rather than the basics. 84. What kind of people does "secret Santas" in the passage refer to? (No more than 12 words) (3 marks) It refers to people who give away anonymous gifts on Christmas. 81. 答案:They will feel greatly surprised 解析:根据第一段 “Linda wakes up, and tries to imagine the wide-eyed surprise of children in another household as they unwrap the presents she carefully chose for them. ”可判断作答 82. 答案:She joined it for fellowship and community service 解析:根据第三段 “Linda did charitable work as a member of the American Women's Club of Toronto. As the name suggests, members are U. S. citizens living in Toronto, who join together for fellowship and community service. ”可判断作答 83. 答案:She wanted to give the children some other gifts rather than the basics 解析:根据最后一段 “Last year Linda asked the mother for a second wish list--one that didn't include the basics. "和 “ She purchased iPods for the two older children and a video game system for the youngest. " 可判断作答 84. 答案:it refers to people who give away anonymous gifts on Christmas 解析:综合第一段和最后一段可推断作答。 (2016 上海) (A) The elephant was lying heavily on its side, fast asleep. A few dogs started barking at it. The elephant woke up in a terrible anger: it chased the dogs into the village where they ran for safety. That didn't stop the elephant. It destroyed a dozen houses and injured several people. The villagers were scared and angry. Then someone suggested calling Parbati, the elephant princess. Parbati Barua's father was a hunter of tigers and an elephant tamer. He taught Parbati to ride an elephant before she could even walk. He also taught her the dangerous art of the elephant round-up -- how to catch wild elephants. Parbati hasn't always lived in the jungle. After a happy childhood hunting with her father, she was sent to boarding school in the city. But Parbati never got used to being there and many years later she went back to her old fife. "Life in the city is too dull. Catching elephants is an adventure and the excitement lasts for days after the chase," she says. But Parbati doesn't catch elephants just for fun. "My work," she says, "is to rescue man from the elephants, and to keep the elephants safe from man. " And this is exactly what Parbati has been doing for many years. Increasingly, the Indian elephant is angry: for many years, illegal hunters have attacked it and its home in the jungle has been reduced to small pieces of land. It is now fighting back. Whenever wild elephants enter a tea garden or a village, Parbati is called to guide the animals back to the jungle before they can kill. The work of an elephant tamer also involves love and devotion. A good elephant tamer will spend hours a day singing love songs to a newly captured elephant. "Eventually they grow to love their tamers and never forget them. They are also more loyal than humans," she said, as she climbed up one of her elephants and sat on the giant, happy animal. An elephant princess indeed! 65. For Parbati, catching elephants is mainly to . A. get long lasting excitement B. keep both man and elephants safe C. send them back to the jungle D. make the angry elephants tame 66. Before Parbati studied in a boarding school, . A. she spent her time hunting with her father B. she learned how to sing love songs C. she had already been called an elephant princess D. she was taught how to hunt tigers 67. Indian elephants are getting increasingly angry and they revenge because __________. A. they are caught and sent for heavy work B. illegal hunters capture them and kill them C. they are attacked and their land gets limited D. dogs often bark at them and chase them 68. The passage starts with an elephant story in order to explain that in India _________. A. people easily fall victim to elephants' attacks B. the man-elephant relationship is getting worse C. elephant tamers are in short supply D. dogs are as powerful as elephants 65. B. 本题为归纳概括题。通读全文可知 Parbati 主要是为了保障大象与人之间的安全而驯象 的。 66. A. 通读文章后可知她从小与父亲一起在丛林中度过了她的童年,之后才去寄宿学校的。 67. C. 可从文章第四段直接得出答案。 68. B. 逻辑推理题。 (B) The following card includes a brief summary and a short assessment of a research paper. It can provide a guide for further reading on the topic. 69. The research paper published is primarily concerned with A. the way of preventing employee turnover B. methods of improving employee performance C. factors affecting employee turnover and performance D. pay structures based on employee performance 70. As is mentioned in the card, the limitation of the research paper mainly lies in that . A. the data analysis is hardly reliable B. the research sample is not wide enough C. the findings are of no practical value D. the research method is out-of-date 71. Who might be most interested in this piece of information? A. Job hunters. Trevor, C. O., Lansford, B. and Black, J. W., 2004, "Employee turnover ( 人 事 变 更 ) and job performance: monitoring the influences of salary growth and promotion", Journal of Armchair Psychology, vol. 113, no.1, pp. 56-64. In this article Trevor et al. review the influences of pay and job opportunities in respect of job performance, turnover rates and employees' job attitude. The authors use data gained through organizational surveys of blue-chip companies in Vancouver, Canada to try to identify the main cause of employee turnover and whether it is linked to salary growth. Their research focuses on assessing a range of pay structures such as pay for performance and organizational reward plans. The article is useful as Trevor et al. suggest that there are numerous reasons for employee turnover and a variety of differences in employees' job attitude and performance. The main limitation of the article is that the survey sample was restricted to mid-level management, thus the authors indicate that further, more extensive research needs to be undertaken to develop a more in-depth understanding of employee turnover and job performance. As this article was published in a professional journal, the findings can be considered reliable. It will be useful additional information for the research on pay structures. B. Employees in blue-chip companies. C. Mid-level managers. D. Researchers on employee turnover. 69. C. 逻辑推理题。通读全文可知这篇文章介绍的是影响人事变更和工作表现的一些因素。 70. B. 从文中“the survey sample was restricted to mid-level management (此次调查只局限于中层 管理人员)”可知答案为 B。 71. D. (C) The 2012 London Olympics had enough problems to worry about. But one more has just been added - a communications blackout caused by solar storms. After a period of calm within the Sun, scientists have detected the signs of a flesh cycle of sunspots that could peak in 2012, just in time for the arrival of the Olympic torch in London. Now scientists believe that this peak could result in vast solar explosions that could throw billions of tons of charged matter towards the Earth, causing strong solar storms that could jam the telecommunications satellites and interact links sending five Olympic broadcast from London. "The Sun's activity has a strong influence on the Earth. The Olympics could be in the middle of the next solar maximum which could affect the functions of communications satellites," said Professor Richard Harrison, head of space physics at the Rutherford Appleton Laboratory in Oxfordshire. At the peak of the cycle, violent outbursts called coronal mass ejections (日冕物质抛射) occur in the Sun's atmosphere, throwing out great quantities of electrically-charged matter. " A coronal mass ejection can carry a billion tons of solar material into space at over a million kilometres per hour. Such events can expose astronauts to a deadly amount, can disable satellites, cause power failures on Earth and disturb communications," Professor Harrison added. The risk is greatest during a solar maximum when there is the greatest number of sunspots. Next week in America, NASA is scheduled to launch a satellite for monitoring solar activity called the Solar Dynamics Observatory (SDO), which will take images of the Sun that are 2016 times clearer than the most advanced televisions available. The Rutherford Appleton Laboratory helped to make the high-tech cameras that will capture images of the solar flares (太阳耀斑) and explosions as they occur. Professor Richard Hold away, the lab's director, said that the SDO should be able to provide early warning of a solar flare or explosion big enough to affect satellite communications on Earth "If we have advance warning, we'll be able to reduce the damage. What you don't want is things switching off for a week with no idea of what's caused the problem," he said. 72. The phrase "communications blackout" in paragraph 1 most probably refers to during the 2012 Olympics. A. the extinguishing of the Olympic torch B. the collapse of broadcasting systems C. the transportation breakdown in London D. the destruction of weather satellites 73. What can be inferred about the solar activity described in the passage? A. The most fatal matter from the corona falls onto Earth. B. The solar storm peak occurs in the middle of each cycle. C. It takes several seconds for the charged matter to reach Earth. D. The number of sunspots declines after coronal mass ejections. 74. According to the passage, NASA will launch a satellite to _________. A. take images of the solar system B. provide early warning of thunderstorms C. keep track of solar activities D. improve the communications on Earth 75. Which of the following might be the best title of the passage? A. Solar Storms: An Invisible Killer B. Solar Storms: Earth Environment in Danger C. Solar Storms: Threatening the Human Race D. Solar Storms: Human Activities to Be Troubled 72. B. 此题线索为第二三段的内容。 73. D. 74. C. 可从文章第六段直接得出答案。 75. D. 通读全文可知整篇文章讨论的是太阳运动所可能会带来的困扰。 Section C Directions: Read the following text and choose the most suitable heading from A-F for each paragraph. There is one extra heading which you do not need. A. Drug overuse and its consequence B. The problem of drug overuse in America C. Benefits of medicine and its wise use D Female drug overuse with reference to that of males E Misuse of medicine among the young generation F. Improper use of medicine among senior citizens 76 . Nowadays. millions of people misuse and even overuse pain medications and other drugs. Research by the American National Institute on Drug Abuse (NIDA, 1999) shows that around 2% of the population over age 12 were using drugs non-medically. 77. NIDA views medications as a powerful force for good in the contemporary world. They reduce and remove pain for millions of people suffering from illness and disease. They make it possible for doctors to perform complicated surgery to save lives. Many people afflicted by serious medical conditions are able to control their symptoms and become active, contributing citizens. NIDA points out that most individuals who take these drags use them in a responsible. 78. Nevertheless, overuse of drugs such as opioids, central nervous system (CNS) depressants and stimulants does lead to harmful reliance in some people and is therefore becoming a serious public health concern. Although this abuse affects many people worldwide, particular trends of concern to the medical profession in the US appear among older adults, teenagers arid women. 79. Though it may be a surprise to many, the misuse of medications may be the most common form of drug abuse among the elderly. Dr Kenneth Schrader of Duke University, North Carolina states that although the elderly represent about 13% of the US population, those aged 65 and over account for the consumption of one third of all drugs. People in this age group use medications roughly three times more than the general population and have poorer compliance with instruction for use. In another study of elderly patients admitted to treatment programs, 70% were women who had overused medicines. 80. Unfortunately, this trend among women does not only affect those aged over In general, among women and men who are using either an anti-anxiety drug or a sedative, women are twice as likely to become addicted. In addition, statistics compiled for 12-17 year olds show that teenage girls are more likely than teenage boys to begin overusing psychotherapeutic medication such as painkillers, tranquillizers, stimulants and sedatives. 76. E 77. C. 78. A 79. F 80. D Section D Directions: Read the passage carefully. Then answer the questions or complete the statements in the fewest possible words. Phys ed (physical education) is making a comeback as a part of the school core curriculum(核 心课程),but with a difference. While group sports are still part of the curriculum, the new way is to teach skills that are useful beyond gym class. Instead of learning how to climb a rope, children are taught to lift weights, balance their diets and build physical endurance. In this way,kids are given the tools and skills and experiences so they can lead a physically active life the rest of their life. Considering that 15 percent of American children 6 to 18 are overweight, supporters say more money and thought must be put into phys ed curriculum. In many cases, that may mean not just replacing the old gym-class model with fitness programs but also starting up phys ed programs because school boards often "put P. E. on the chopping block, cutting it entirely or decreasing its teachers or the days it is offered," says Alicia Moag-Stahlberg, the executive director of Action for Health Kids. The difference in phys ed programs is partly due to the lack of a national standard. "Physical education needs to be part of the core curriculum," she added. The wisdom of the new approach has some scientific support. Researchers at the University of Wisconsin have demonstrated how effective the fit-for-life model of gym class can be. They observed how 50 overweight children lost more weight when they cycled and skied cross-country than when they played sports. The researchers also found that teaching sports like football resulted in less overall movement, partly because reluctant students were able to sit on the bench. Another problem with simply teaching group sports in gym class is that only a tiny percentage of students continue playing them after graduating from high school. The new method teaches sells that translate to adulthood. (Note: Answer the questions or complete the statements in NO MORE THAN TEN WORDS. ) 81. In the new P. E. program, children learn to lift weights, balance their diets and build physical endurance rather than ___________________. 82. As for P. E. , some school boards either ____________. 83. What are the two problems with simply teaching group sports? 84. What is the long-term benefit of the new P. E program? 81. learning how to climb a rope. 82. cutting it entirely or decreasing its teachers or time. 83. less overall movement and students' quiting playing after graduation. 84. It can give children a physically active life. (2016 福建) A F . Scott Fitzgerald, born on September 24,1896, an American novelist, w as once a student of St. Paul Academy, the Newman School and attended Princeton. University for a short while. In 1917 he joined the army and was posted in Alabama, where he met his future wife Zelda Sayre. Then he had to make some money to impress her. His life with her was full of great happiness, as he wrote in his diary : “ My own happiness in the past often approached such joy that I could share it even with the person dearest to me but had to walk it away in quiet streets and take down parts of it in my diary. ” This side of paradise, his first novel, was published in 1920. encouraged by its success, Fitzgerald began to devote more time to his writing. Then he continued with the novel the Beautiful and Damned (1922), a collection of short stories Thales of the Jazz Age (1922), and a play The Vegetable (1923). But his greatest success was The Great Gatsby, published in 1925,which quick brought him praise from the literary world. Yet it failed to give him the needed financial security. Then, in 1926, he published another collection lf short stories All the Sad Young Men. However, Fitzgerald’s problems with his wife www. ks5u. com Zelda affected his writing. During the 1920s he tried to reorder his life, but failed. By 1930, his wife had her first breakdown and went to a Swiss clinic. During this period he completed novels Tender Is the Night in 1934 and The love of the last Tycoon in 1940. while his wife was in hospital in the United States, he got totally addicted to alcohol. Sheila Graham, his dear friend, helped him fight his alcoholism. 56. How many novels written by Fitzgerald are mentioned in the passage ? A. 5 B. 6 C. 7 D. 8 57. Which of the following is the correct order to describe Fitzgerald’s life according to the passage? 高考 a. He became addicted to drinking. b. He studied at St. Paul Academy. c. He published his first novel This Side of Paradise. d. The Great Gatsby won high praise. e. He failed to reorder his life. f. He joined the army and met Zelda. A. f-c-e-a-b-d B. b-e-a-f-c-d C. f-d-e-c-b-a D. b-f-c-d-e-a 58. We can infer from the passage that Fitzgerald . A. had made some money when he met Zelda in Alabama. B. was well educated and well off before he served in the army C. would have completed more works if his wife hadn’t broken down D. helped his friend get rid of drinking while his wife was in hospital 59. The passage is probably followed by a concluding paragraph about . A. Zelda’s personal life B. Zelda’s illness and treatment C. Fitzgerald’s friendship with Graham D. Fitzgerald’s contributions to the literary world B Forget Twitter and Facebook, Google and the Kindle. Television is still the most influential medium around. Indeed ,for many of the poorest regions(地区)of the world, it remains the next big thing——finally becomes globally available. And that is a good thing, because the TV revolution is changing lives for the better. Across the developing world, around 45% of families had a TV in 1995; by 2005 the number had climbed above 60%. That is some way behind the U. S. ,where are more TVs than people, and where people now easily get access to the Internet. Five million more families in sub-Saharan Africa will get a TV over the next five years. In 2005 , after the fall of the Taliban(塔利班),which had outlawed TV, I in 5 Afghans had one. The global total is another 150 million by 2013——pushing the numbers to well beyond two thirds of families. Television’s most powerful effect will be on the lives of women. In India, researchers Robert Jensen and Emily Oster found that when TVs reached villages, women were more likely to go to the market without their husbands approval and less likely to want a boy rather than a girl. They were more likely to make decisions over child health care. TV is also a powerful medium for adult education. In the Indian state of Gujarat, Chitrageet is a popular show that plays Bollywood songs with words in Gujarati on the screen. Within six months, viewers had made a small but significant (有意义的)improvement in their reading skills. Too much TV has been associated with violence, overweight and loneliness. However, TV is having a positive influence on the lives of billions worldwide. 60. The underlined word “outlawed” in paragraph 2 probably means “ ”. A. allowed B. banned C. offered D. refused 61. Which of the following is TRUE according to the passage? A. Americans used to get access to the Internet easily. B. The world’s TV sets will total 150million by 2013. C. 45% of families in the developing countries had a TV in 2005. D. Over two thirds of families in the world will have a TV by 2013. 62. The author intends to . A. stress the advantages of TV to people’s lives B. persuade women to become more independent C. encourage people to improve their reading skills D. introduce the readers some websites such as Google 63. What would be the best title for the passage? A. TV Will Rule the World B. TV Will Disturb the World C. TV Will Better the World D. TV Will Remain in World C Businesses are witnessing a difficult time, which has in turn produced influence on consumers’ desire to go green . However, shoppers are still laying stress on environmental concerns. Two thirds of customers say that environmental considerations inform their purchases to the same degree as they did a year ago, while more than a quarter say that they are now even better aware of the environmental effect on what they buy. This may help to influence how shops store goods on their shelves. And the companies should still make efforts to become more environmentally friendly. Two out of three people think it is important to buy from environmentally responsible companies, with about one in seven saying that they had even decided to take their custom elsewhere if they felt a company’s environmental reputation was not good enough. Harry Morrison, chief executive(主管)of the Carbon Trust, sympathizes:“I understand this situation where survival is very important now. But from environmental considerations, the clock is ticking—we don’t have much time. In addition, cutting carbon has an immediate effect as costs drop and a medium-term benefit for the brand. ” Larger companies have an extra motivation to look at reducing their carbon footprint, as new rules next year will require businesses to buy carbon allowances to make up for their emissions(排 放). Those that have taken early action will have a head start. More than two thirds of consumers are not clear about which companies are environmentally responsible. This suggests that firms that are able to relay clearly their message to the public will be in a pole position to attract shoppers. The Carbon Trust believes that it can help by informing customers about the good work companies are doing. “When companies are granted(授予)the standard, they can use a logo(标 识)in all their marketing which makes it clear that they are working towards cutting emissions,”Mr. Morrison said. 64. What’s the main idea of the passage? A. Businesses are finding ways to send their message to the shoppers. B. Companies will soon get information about cutting carbon emissions. C. Firms are making efforts to encourage customers to keep goods at home. D. Firms are urged to cut carbon emissions by shoppers’ environmental awareness. 65. The underlined word“inform”in Paragraph 2 probably means“ ”. A. affect B. change C. disturb D. reject 66. According to Harry Morrison, businesses . A. will benefit from cutting carbon emissions B. should buy carbon allowances for shoppers C. are required to make up for their carbon emissions D. have encouraged shoppers to take their custom elsewhere 67. We can learn from the passage that businesses will . A. have a strong desire to reduce costs B. use the same logo in their marketing C. gain advantages by taking early action D. attract more shoppers by storing goods D Columbus College, 241 Queen Elizabeth Drive, Kowloon City Memo To: All Stall From: Jalie Mok, Secretary; Sports Development Committee Date: May 20, 202016 A week ago, “Sports for Life” programme was sent to the parents, requiring them to select a sport they wanted their child to play. Since then, our staff have received lots of calls from parents asking for more information about it. Here is a memo(备忘录)for your reference when you answer the phones. Sports 1;Basketball We expect that this will be the most popular of the four sports. Therefore, students should be advised to sign up as soon as possible. Students will take a private bus to and from Kwun Tong Sports Park. To cover the cost of hiring a bus, each student will have to pay $2016 each time. There will be four basketball courts available for our use with one teacher watching over each game. Sports 2: Gym We will be using St. Peter’s Memorial Park. There are two reasons for choosing this park. First, it is not very busy and crowded before 6:00pm. Second, it has lots of trees with plenty of shade. Students must bring along two bottles of water to prevent thirst. Three activities, shipping ,jogging , outdoor aerobics(有氧运动),all of which are free of chare, will be arranged. And there will be a teacher on duty for each of the activities. Sprots3: Hiking Hiking(远足)will take place at Kowloon Peak. The activity will start at 2:30 pm and finish 90 minutes later. Three teachers will accompany the students, and a hiking instructor will accompany each group of 15 hikers. Each instructor will cost $75/hr. Sports 4;Swimming The Kowloon City Aquatic Centre is a 2016-minute walk from our school. Four teachers will go to the pool and conduct the going-on from the poolside. We will only be able to reserve the pool for one hour (i. e. 2:45pm to 3:45 pm). Only students skilful at swimming can take up this activity. The pool will have two life guards present. Girls must wear a swimming suit. The cost is $2016 per visit. 68. Which of the following is TRUE according to the passage? A. Students selecting basketball had better register early. B. Students participating in gym should arrive at 6:00 pm. C. Hiking usually begins at 2:30 pm and lasts 2 hours. D. Students having swimming suits can take up swimming. 69. It can be inferred from the passage that A. students taking part in basket ball will walk to the courts. B. every student can gain admission to one of the four sports C. hiking students can have a regular rest in the shade of trees D. students taking up the four sports should bring bottles of water 70. It can be concluded that A. each sport will take only on hour B. students will be charged for the four sports C. all the sports will take place at Kowloon Peak D. teachers will accompany students on each sport 71. The purpose of the memo is to A . attract students’ interest in the programme B. require the parents to select a sport for their child C. help the staff explain the programme to the parents D. remind teachers and lifeguards to be present on time E In the United States, there were some well-constructed houses for native Indians, ranging from the simple brush shelter to the five-storied pueblo. In the eastern United States, one of the existing types was that commonly know under the Algonkian name of wigwam in which the Iroquois Indians lived. The wigwams were of wagon-top shape with straight sides and ends, made by bending young trees to form the round shape. Over this shape pieces of tree bark were laid to protect the Indians from bad weather. Over the bark dried grass was added. A small hole allowed smoke to escape from the top. Doorways at each en served also as windows, The Iroquois Indians built trunk walls all around their villages. The wall had only one opening, They could quickly close this opening if their enemies came near. Interestingly, the Choctaw Indians in Mississippi also lived in a wigwam of a most primitive(原 始的) construction, but different from those of the Iroquois Indians. The Choctaw Indians’ wigwams, made from mud, cane and straw, were in the form of a bee-hive. The covering was made of a long, tough grass. A post in the centre supported the roof. A hole in the top admitted the light, and allowed the smoke to pass out. The tipi tent-housing of the upper lake and plains area was put up with poles set lightly in the ground, tied together near the top, and covered with bark and grass in the lake country. It was easily portable, and two women could set it up or take it down within an hour. The Pawnee, Mandan and other Indian tribes (部落) along the Missouri built solid ring-shaped structures of trunk, covered with earth and dried grass, housing a dozen families. The Wichita and other tribes of the Texas border built large ring-shaped houses covered with dried grass. Apart from the regular housing, almost every tribe had some style of housing. 72. Which of the following pictures shows the house for the Iroquois Indians? 73. According to the passage, the Pawnee Indians built their houses _____. A. with openings in the trunk walls B. large enough for several families C. in a ring shape with bark and mud D. by bending young trees to form the shape 74. All the native Indian houses described in the passage were_____. A. of the same shape B. covered with grass C. built with a post in the centre D. built with doorways at each end 75. The passage suggests that ________ A. all the native Indians built trunk walls all around their houses B. all the native Indian houses were built with poles tied together C. the Iroquois Indians took safety into account while building their wigwams D. the Choctaw Indians in Mississippi built their wigwams with straight sides and ends 答案:56. A 57. D 58. C 59. D 60. B 61. D 62. A 63. C 64. D 65. A 66. A 67. C 68. A 69. B 70. D 71. C 72. A 73. B 74. B 75. C (2016 江西) A Andy rode slowly on his way to school, day-dreaming about the fishing trip that his father had promised him. He was so busy dreaming about all the fish he would catch that he was unaware of everything else around him. He rode along until a strange sound drew him to the present. He came to a stop and looked curiously up to the heavens. What he saw shocked and terrified him. A huge swarm of bees filled the sky like a black cloud and the buzzing mass seemed to be heading angrily towards him. With no time to waste, Andy sped off in the opposite direction, riding furiously—but without knowing how to escape the swarm. With a rapidly beating heart and his legs pumping furiously, he sped down the rough road. As the bees came closer, his panic increased. Andy knew that he was sensitive to bee stings(蜇). The last sting had landed him in hospital—and that was only one bee sting! He had been forced to stay in bed for two whole days. Suddenly, his father’s words came to him. “When you are in a tight situation, don’t panic. Use your brain and think your way out of it. ” On a nearby hill, he could see smoke waving slowly skywards from the chimney of the Nelson family home. “Bees don’t like smoke,” he thought. “They couldn’t get into the house. ” Andy raced towards the Nelson house, but the bees were gaining ground. Andy knew he could not reach the house in time. He estimated that the bees would catch up with him soon. Suddenly, out of the corner of his eyes, he spotted a small dam used by Mr. Nelson to irrigate his vegetable garden. Off his bike and into the cool water he lived, disappearing below the surface and away from the savage insects. After holding his breath for as long as he could, Andy came up for air and noticed the bees had gone. Dragging himself out of the dam, he struggled up the hilly slope and rang the doorbell. Mrs. Nelson took him inside and rang his mother. “You’ll really need that fishing break to help you recover,” laughed his mother with relief. “Thank goodness you didn’t panic!” But Andy did not hear her. He was dreaming once again of the fish he would catch tomorrow. 本文讲述了主人公 Andy 在上学途中遭遇蜂群攻击, 急中生智, 躲入水中避险成功的故事。 56. Why did Andy fail to notice the swarm of bees earlier? A. He was riding to school. B. He was listening to a strange sound. C. He was going fishing with his father. D. He was lost in the thought of the fishing trip. 答案:D 考点::细节理解 解析: 题目: 为什么 Andy 未能早些发现蜂群? A. 他正在骑车去学校 B. 他正在听一个奇怪的声音 C. 他将要和他的父亲去钓鱼 D. 他的思绪沉浸于钓鱼之旅。 从第一段最后一句: He was so busy dreaming about all the fish he would catch that he was unaware of everything else around him. 可见, Andy 正忙于做抓鱼的白日梦, 没有意识到他周围的事情。 与 D 选项, 他的思绪沉浸于钓鱼之旅相符。 57. Which of the following is NOT mentioned about the swarm of bees in the passage? A. They crowded like a black cloud. B. They shocked and terrified Andy. C. They tried to attack Andy in a mass. D. They made Andy stay in hospital for two days. 答案:D 考点::细节理解 题目: 关于蜂群哪一项文中并没有提到? A. 他们拥挤的像一片乌云 B. 他们吓到了 Andy C. 他们想成群攻击 Andy D. 他们让 Andy 在医院住了两天 解析: A. C. 三项的出处均为原文第二段的最后一句: A huge swarm of bees filled the sky like a black cloud and the buzzing mass seemed to be heading angrily towards him. 意为, 一大群蜜蜂 像乌云一样布满天空而嗡嗡作响的蜂群似乎正怒气冲冲的向他飞来。可见, A. 蜂群像乌云一样, C. 蜜蜂想成群攻击 Andy 均有提到;B 选项的出处为第二段倒数第二句:What he saw shocked and terrified him, Andy 看到的景象吓坏了他, 故 B 选项也有提到。D 选项出处为第三段第五句 话和第六句话, 但是文中所述是 Andy 上次被蜜蜂蛰过一次后进了医院住了两天, 选项偷换了 概念, 故 D 选项文章并未提到, 选择 D。 58. How did Andy avoid the bees in the end? A. He asked Mr. Nelson for help. B. He did himself under the water. C. He rushed into the Nelson house. D. He rode off in the opposite direction. 答案:B 考点::细节理解 题目: 最终 Andy 是如何躲开蜂群的? A. 他向 Mr. Nelson 求助 B. 他把自己藏在了水下 C. 他冲进了 Nelson 的房子 D. 他骑向了相反的方向 解析: 出处为第五段第二三句话, Off his bike and into the cool water he dived, disappearing below the surface and away from the savage insects. After holding his breath for as long as he could, Andy came up for air and noticed the bees had gone . Andy 下了车潜入了水中, 躲开了蜂群, 在屏 息了尽可能长的时间后 Andy 出来发现蜜蜂已经飞走了。故选择 B 选项。 59. Which of the following can best describe Andy’s escape from the bees? A. No pains, no gains. B. Once bitten, twice shy. C. Where there is a will, there is a way. D. In time of danger, one’s mind works fast. 答案:D 考点::主旨大意 题目: 以下哪个能最好的描述 Andy 的蜂群脱险? A. 不劳无获 B. 一朝被蛇咬, 三年怕井绳 C. 有志者. 事竟成 D. 急中生智 解析: 出处, 文章最后一段的第二句, Andy 的妈妈说道: Thank goodness you didn't panic, 谢天 谢地你没有慌神, 可知情急之下 Andy 仍然成功想到办法脱险, 再综合 A. B. C. D 四个选项的 意思, 可以判断出 D 选项。 B CWU The communication union Head of Research Salary:£55. 271 We are looking for a Head of Research to manage the CWU Research Department and Information Centre. You would be required to exercise control of all research work of the department and manage a team of three researchers and four support staff. The person appointed would be expected to carry out research work of a strategic nature across the range of businesses in which the CWU has or seeks membership and to contribute to the strategic thinking and direction of the union as a whole. You will need: proven line management skills, especially in managing and motivating a team; good research skills, holding a good degree in a related subject or other similar experience; a high level of mathematical and calculating skills; the ability to produce high quality work under pressure; a commitment to and knowledge of the trade union movement and social democratic politics; and knowledge and/ or experience of the postal and/ or telecommunications industry. To apply, please request an application pack by emailing hr@cwu. org or by telephoning HR( Human Resources ) on 020 8971 7482. When applying please state your source. Closing Date for Applications: 4th August 202016 Anticipated interview date: 17th August 202016 No agencies please 60. In which column of a newspaper could we find this advertisement? A. Arts. B. Sales. C. Jobs. D. News. 答案:C 考点::推断题。 题目: 在报纸的哪一个专栏我们能发现这个广告? A 艺术 B 销售 C 求职 D 新闻。 解析: 全文讲的是这项研究工作, 并且提及薪水、工作要求等, Head of Research 也是一个职 位, 因此选 C, 求职专栏。 61. One of the duties of the person to be appointed is . A. taking charge of research work. B. seeking membership for the trade union. C. managing a team of three or four members. D. running a telecommunications company. 答案:A 考点::细节理解 题目: 被委任的职责之一是? A 负责研究工作 B 为贸易联盟寻找会员 C 管理有 3 到 4 个成员的团队 D 经营一个电子通讯公司 解析:原文第一段最后一句话说明了所要求的工作,练习控制好所有研究工作,因此选 A。B 错在原文第二段是说 CWU 寻找成员不是贸易联盟。C 错在第一段最后一句话是说管理有三个 研究人员和四个工作人员的团队,不是或者。D 错在第三段最后一句话是说有电子通讯行业的 知识或经验。 62. If you want to apply for this position, you can do all EXCEPT . A. ask an agency for an application form. B. dial 020 8971 7482 for more information. C. email hr@cwu. org for an application pack. D. send in your application before 4th August 202016 答案:A 考点::细节理解 题目: 如果你想申请这个职位,你要做除了以下哪个选项 A 向代理部门要一张申请表 B 拨打 020 8971 7482 这个电话询问更多信息 C 给 hr@cwu. org 发电子邮件要一个申请文件包 D 在 202016 年 8 月 4 日之前发来申请表 解析:原文最后一段提到了 B、C 两个选项,文章最后也提及了截止时间,只有 A 没有提到。 63. Which of the following applicants is most likely to be employed? A. A school teacher with a master’s degree. B. A university graduate majoring in computer science. C. A director from a research centre with a master’s degree. D. A clerk from a telecommunications company. 答案:C 考点::推断题。 题目: 以下哪一个申请人最有可能被录用? A 一个有研究生学历的在校教师 B 一个主修计算机科学的大学毕业生 C 一个来自某研究中心的主管并有研究生学历 D 一个电子通讯公司的职员 解析: 通过文中所说的工作职责——掌管研究工作、管理团队,职位要求——有很好的研究 技术、在相关领域有不错的学历或工作经验,因此选 C。 C Kong Zi , also called Confucius (551-479 B. C) , and Socrates (469-399 B. C) lived only a hundred years apart , and during their lifetimes there was no contact between China and Greece, but it is interesting to look at how the world that each of these great philosophers came from shaped their ideas , and how these ideas in turn ,shaped their societies. Neither philosopher lived in times of peace, though there were more wars un Greece than in China. The Chinese states were very large and feudal, while the Greek city-states were small and urban. The urban environment in which Socrates lived allowed him to be more radical than Confucius. Unlike Confucius, Socrates was not asked by rules how to govern effectively. Thus, Socrates was able to be more idealistic, focusing on issues like freedom, and knowledge for its own sake. Confucius, on the other hand, advised those in government service, and many of his students went out to government service. Confucius suggested the Golden Rule as a principle for the conduct of life:” Do not do to others what you would not want others to do to you. ” He assumed that all men were equal at birth, though some bad more potential than others, and that it was knowledge that set men apart. Socrates focused on the individual, and thought that the greatest purpose of man was to seek wisdom. He believed that the superior class should rule the inferior(下层的)classes. For Socrates, the family was of no importance, and the community of little concern. For Confucius, however, the family was the centre of the society, with family relations considered much more important than political relations. Both men are respected much more today than they were in their lifetimes. 64. Which of the following is TURE according to the first paragraph? A. Socrates and Confucius had much in common, B. Confucius had much influence on Socrates’ ideas. C. The societies ware influenced by the philosophers’ ideas. D. There were cultural exchanges between China and Greece. 答案:C 考点::细节理解。 题目: 从第一段中得出以下哪一个是正确的? A: 孔子和苏格拉底有很多共同之处 B: 孔子对于苏格拉底的观念起了很大影响 C: 社会被哲学家的观念所影响 D:在中国和希腊之间有文化交流 解析: 从文中第一段最后一句的后半句得出答案。文中说到两位哲学家孔子和苏格拉底,世 界影响了他们哲学观念的塑造,同样他们的观念也影响了他们的社会。C 选项最符合文意, ABC 选项原文均未提及。 65. Socrates shared with Confucius the idea that . A. all men were equal when they were born B. the lower classed should be ruled by the upper class C. the purpose of man was to seek freedom and wisdom D. people should not ask others to do what they did not want to 答案:B 考点::细节理解 题目: 苏格拉斯与孔子一致的观念是? A: 众生平等 B:下层阶级应该被上层阶级管理 C: 人的目标就是追求自由和智慧 D: 己所不欲,勿施于人 解析: 第三段最后一句得出答案。ABC 选项均是在陈述孔子的观点,只有 B 选项是在说苏格 拉底的哲学理念。像孔夫子一样,苏格拉底相信上层阶级应该统治下层阶级。 66. What made some people different from others according to Confucius? A. Family. B. Potential. C. Knowledge. D. Community. 答案:C 考点::细节理解 题目: 根据孔子所述,什么使众生彼此不相同? A:家庭 B: 潜能 C: 知识 D: 社团 解析:从文中第三段第二句得出答案。孔子认为人出生时都是平等的,虽然有些人有过人的 潜能,是知识使人与人之间不相同。 67. This passage is organized in the pattern of , A. time and events B. comparison and contrast C. cause and effect D. definition and classification 答案:B 考点::篇章结构理解题 题目: 文章是以什么结构组织展开? A: 时间和事件 B: 比较和对照 C: 起因和结果 D: 定义和分类 解析:本篇文章通篇采用对比的手法,将孔子和苏格拉底在哲学观念上的相同以及不同之处 进行比较论述。故选择 B 选项。 D Modern inventions have speeded up people’s loves amazingly. Motor-cars cover a hundred miles in little more than an hour, aircraft cross the world inside a day, while computers operate at lightning speed. Indeed, this love of speed seems never-ending. Every year motor-cars are produced which go even faster and each new computer boats (吹嘘) of saving precious seconds in handling tasks. All this saves time, but at a price. When we lose or gain half a day in speeding across the world in an airplane, our bodies tell us so. We get the uncomfortable feeling known as jet-lag; our bodies feel that they have been left behind on another time zone. Again, spending too long at computers results in painful wrists and fingers. Mobile phones also have their dangers, according to some scientist; too much use may transmit harmful radiation into our brains, a consequence we do not like to think about. However, what do we do with the time we have saved? Certainly not relax, or so it seems. We are so accustomed constant activity that we find it difficult to sit and do nothing or even just one thing at a time. Perhaps the days are long gone when we might listen quietly to a story on the radio, letting imagination take us into another world. There was a time when some people’s lives were devoted simply to the cultivation of the land or the care of cattle. No multi-tasking there; their lives went on at a much gentler pace, and in a familiar pattern. There is much that we might envy about a way of life like this. Yet before we do so, we must think of the hard tasks our ancestor faced: they farmed with bare hands, often lived close to hunger, and had to fashion tools from wood and stone. Modern machinery has freed people from that primitive existence. 68. The new products become more and more time-saving because . A. our love of speed seems never-ending B. time is limited. C. the prices are increasingly high. D. the manufactures boast a lot. 答案:A 题目: 新发明变得越来越省时, 是因为? A. 我们对速度的热爱从未停止。 可回原文定位 never-ending(因为有连词符号)。原文第一段 说到 Indeed, this love of speed seems never-ending. 事实上, 我们对速度的热爱从未停止。A 选 项是对原文的同意改写。 B. 时间有限。原文未提及。 C. 价格日渐攀升。原文未提及价格因素。 D. 生产商大肆吹嘘。回原文定位 Boast(因为有中文注释). 第一段最后一句说 Every year motor-cars are produced which go even faster and each new computer boats ( 吹 嘘 ) of saving precious seconds in handling tasks. 解析:每年都有更高速的新车出产, 而且新电脑也吹嘘能节省宝贵的每一秒钟。可见, 题目和 选项矛盾。越来越省时是事实, 不是因为吹嘘而变得省时了。 69. What does “the days” in Paragraph 3 refer to ? A. Imaginary life B. Simple life in the past. C. Times of inventions D. Time for constant activity. 答案:B 题目: 第三段的“the days”指的是什么? A: 想象的生活 B: 过去的简单生活 C: 发明的时间 D: 连续活动的时间 解析: 回原文定位第三段该句 Perhaps the days are long gone when we might listen quietly to a story on the radio, letting imagination take us into another world. 也许, 我们静静聆听广播里的故 事节目, 任想象插上翅膀肆意翱翔的日子, 已经一去不返了。根据前后一句可得知, 文章重点 是说的高速生活和简单生活的对比。故:B 选项是对原文的正确解读。而 A 选项的理解有偏差, 重点转移了。 70. What is the author’s attitude towards the modern technology? A. Critical B. Objective. C. Optimistic. D. Negative. 答案:B 题目: 作者对现代科技的态度是? A: 批判的 B: 客观的 C: 乐观的 D: 消极的 解析: 根据文章结构, 第一段陈述事实, 说现代高科技省时省力。第二段说高速生活的弊端。 第三段说对简单生活的向往, 但最后第四段却说高科技对现代生活的积极影响。所以, 作者的 观点是不偏不倚的。 71. What does the passage mainly discuss? A. The present and past times. B. Machinery and human beings. C. Imaginations and inventions. D. Modern technology and its influence. 答案:D 题目: 这篇文章主要说的是什么? A: 过去与现在 B: 机器和人类 C: 想象力和创造力 D: 现代科技和它的影响 解析: 根据文章首尾段的大意可知, 文章重在分析高科技的利与弊。 E Every day we experience one of the wonders of the world around us without even realizing it. It is not the amazing complexity of television, nor the impressive technology of transport. The universal wonder we share and experience is our ability to make noise without mouths, and so transmit ideas and thoughts to each other’s minds. This ability comes so naturally that we tend to forget what a miracle (奇迹) it is. Obviously, the ability to talk is something that marks humans off from animal. Of course, some animals have powers just as amazing, Birds can fly thousands miles by observing positions of the stars in the sky in relation to the time of day and year. In Nature’s talent show, humans are a species of animal that have developed their own special act. If we reduce it to basic terms, it’s an ability for communicating information to others, by varying sounds we make as we breathe out. Not that we don’t have other powers of communication. Our facial expressions convey our emotions, such as anger, or joy, or disappointment. The way we hold our heads can indicate to others whether we are happy or sad. This is so-called “body language”. Bristling (直立的) fur is an unmistakable warning of attack among many animals. Similarly, the bowed head or drooping tail shows a readiness to take second place in any animal gathering. Such a means of communication is a basic mechanism that animals, including human beings, instinctively acquire and display. Is the ability to speak just another sort of instinct? If so, how did human beings acquire this amazing skills ? Biologist can readily indicate that particular area of our brain where speech mechanisms function, but this doesn’t tell us how that part of our bodies originated in our biological history. 本文是介绍关于人类使用语言的能力的说明文, 主要阐述人区分动物的特殊能力及其相似性。 72. According to the passage, the wonder we take for granted is . A. our ability to use language B. the miracle of technology C. the amazing power of nature D. our ability to make noises with mouth 答案:A 考点::细节理解题。 题目: 根据文章, 我们认为理所当然的奇迹是: A: 我们使用语言的能力 B: 技术的奇迹 C: 自然的惊人力量 D: 我们用嘴制造噪音的能力。 解析:根据第一段倒数第二句话 The universal wonder we share and experience is our ability to make noise without mouths, and so transmit ideas and thoughts to each other’s minds. 可知: 我们 把使用语言的能力这个奇迹当做理所当然。B, C 选项与文章不符, 在第一段第二句话中可得 知。D 选项易误导学生, 在文章中虽然有原话, 但一句话没有说完, 关键是通过嘴巴发出的声 音去给其他人传递信息。故 D 选项也是错误的。 73. What feature of “body language” mentioned in the passage is common to both human an animals? A. Lifting heads when sad. B. Keeping long faces when angry. C. Bristling hair when ready to attack. D. Bowing heads when willing to obey. 答案:D 考点::细节理解题。 题目: 文章中提到的“肢体语言”的什么特征对于人类和动物都是共同的: A: 伤心时抬头 B: 愤怒时拉长脸 C: 准备攻击时直立毛皮 D: 乐意服从时低头 解析: 根据第三段最后一句话“Similarly, the bowed head or drooping tail shows …. . ”可知答案。 A 和 B 选项描绘的是人的肢体语言特征。C 选项描绘的是动物的肢体语言。 74. What can be inferred from Paragraph 3? A. Body language is unique to humans. B. Animals express emotions just as humans do. C. Humans have other powers of communication. D. Humans are no different from animals to some degree. 答案:D 考点::推理判断题。 题目: 从第三段可以推段出: A: 肢体语言是人类独一无二的 B: 动物和人类一样表达情感 C: 人类有其他交流的能力 D: 人类在某种程度上和动物是相同的 解析: 根据第三段后三句话可知: 人类在某种程度上和动物没有不同。从段落中可得知 A, B, C 均与文章不符。 75. This passage is mainly about . A. the development of body language. B. the special role humans play in nature C. the power to convey information to others D. the difference between humans and animals in language use 答案:C 考点::主旨大意题。 题目: 这篇文章主要讲述什么? A: 肢体语言的发展 B: 人类在自然中扮演的特殊角色 C: 给他人传递信息的力量 D: 人类和动物在语言使用中的差异 解析: 整篇文章谈论的都是给他人传递信息的力量, 使用语言的能力。A, B 选项文章未提及. D 选项较片面, 文章第三段中提到了相似点和差异二种。 第二卷(非选择题 满分 35 分) 第四部分 写作(共两节,满分 35 分) 第一节 对话填空(共 2016 小题;每小题 1 分,满分 2016 分) 请认真阅读下面对话,并根据各题所给首字母的提示,在答题卡上标有题号的横线上, 写出一个英语单词的完整、正确形式、使对话通顺。 M=Mum S=Sara P=Peter (When Sara came home from work, she found her mother in an anxious state) S: What’s wrong, Mum? You seem upset. M: It’s your (76) y sister Jill. She should have been here an hour ago. S: Don’t (77) w , Mum! It’s not all that late and she has probably not (78) r how late it is. I think she’ll be here soon. (Just then Sara’s brother Peter came into the kitchen. ) P: What’s up? Mum’s on the phone and she seems as though she’s going to cry. S: It’s Jill. She should have (79) r home by now and Mum’s getting anxious. P: You needn’t feel so anxious Mum! You know what teenage girls are like. I’m (80) s she’s all right. M: I can’t help but he concerned. I’ve just rung Lucy and she said Jill left her outside the cinema ages ago to get the bus home. P: Well, the bus service isn’t very frequent in the evenings. Perhaps she (81) m one and is waiting for another. Have you tried (82)p her on her mobile? M: Yes, but there’s no reply. That made me even more worried. I think (83)s must have happened to Jill. Do you think we should call the (84)P ? S: Calm down, Mum! You’ll make yourself ill. Perhaps she switched her mobile (85)o in the cinema and forgot to switch it back on again. Or it may need to be charged. 答案:76. younger/youngest 77. worry 78. realized 79. returned/reached 80. sure 81. missed 82. phoning 83. something 84. police/policeman 85. off (2016 山东) A Christopher Thomas, 27, was a writer by night and a teacher by day when he noticed he was always tired and was losing weight fast. Diagnosed with diabetes(糖尿病), Thomas would need to inject himself with insulin(胰岛素) three times a day for the rest of his life or risk nerve damage, blindness, and even death. And if that weren't bad enough, he had no health insurance. After a month of feeling upset, Thomas decided he'd better find a way to fight back. He left Canton, Michigan for New York, got a job waiting tables, nicknamed himself the Diabetic Rockstar, and created diabeticrockstar. com, a free online community for diabetics and their loved ones—a place where over 1,20160 people share personal stories, information, and resources. Jason Swencki’s son, Kody, was diagnosed with type diabetes at six. Father and son visit the online children's forums(论坛) together most evenings. "Kody gets so excited, writing to kids from all over," says Swencki, one of the site's volunteers. "They know what he's going through, so he doesn't feel alone. " Kody is anything but alone: Diabetes is now the seventh leading cause of death in the United States, with 24 million diagnosed cases. And more people are being diagnosed at younger ages. These days, Thomas's main focus is his charity(慈善机构), Fight It, which provides medicines and supplies to people—225 to date—who can't afford a diabetic's huge expenses. Fight-it. org has raised about $23,000—in products and in cash. In May, Thomas will hold the first annual Diabetic Rockstar Festival in the Caribbean. Even with a staff of 22 volunteers, Thomas often devotes up to 50 hours a week to his cause, while still doing his full-time job waiting tables. "Of the diabetes charities out there, most are putting money into finding a cure," says Bentley Gubar, one of Rockstar's original members. "But Christopher is the only person I know saying people need help now. " 56. Which of the following is true of Christopher Thomas? A. He needs to go to the doctor every day. B. He studies the leading cause of diabetes C. He has a positive attitude to this disease. D. He encourages diabetics by writing articles. 答案:C 考点::细节理解题。 解析:通读全文可知,Christopher Thomas 对于自己的疾病采取了积极的态度,所以 C 项正确。 A 项与 Thomas would need to inject himself with insulin(胰岛素) three times a day for the rest of his life 不符;B 项与文中的 Thomas 开办网站,与病人及其家属分享有关的信息和资源不符; D 项提到的写作是 Thomas 的专职工作,而他的志愿工作才是运作 Fight It 这个慈善机构。 57. Diabeitcrockstar. com was created for _________. A. diabetics to communicate B. volunteers to find jobs C. children to amuse themselves D. rock stars to share resources. 答案:A 考点::细节理解题。 解析:根据 a free online community for diabetics and their loved ones—a place where over 1,20160 people share personal stories, information, and resources 可知 A 项正确。 58. According to the text, Kody ______. A. feel lonely because of his illness B. benefits from diabeticrockstar. com C. helps create the online kid’s forums D. writes children’s stories online 考点::细节理解题。 答案:B 解析:根据 Father and son visit the online children's forums(论坛) together most evenings. "Kody gets so excited, writing to kids from all over," says Swencki, one of the site's volunteers. "They know what he's going through, so he doesn't feel alone. " 可知 B 项正确。 59. What can we learn about Fight It? A. It helps the diabetics in financial difficulties. B. It organizes parties for volunteer once a year. C. It offers less expensive medicine to diabetics. D. It owns a well-known medical website. 考点:: 解析:细节理解题。根据 Fight It, which provides medicines and supplies to people—225 to date—who can't afford a diabetic's huge expenses. Fight-it. org has raised about $23,000—in products and in cash. 可知 A 项正确。 答案:A 60. The last paragraph suggests that Thomas ______. A. works full-time in a diabetes charity B. employs 22 people for his website C. helps diabetics in his own way D. ties to find a cure for diabetes 考点:: 解析:推理判断题。根据But Christopher is the only person I know saying people need help now. 可 知 Thomas 在用自己的方式帮助糖尿病人,所以 C 项正确。A 项与 Thomas often devotes up to 50 hours a week to his cause, while still doing his full-time job waiting tables 不符;B 项与 Even with a staff of 22 volunteers ;D 项与"Of the diabetes charities out there, most are putting money into finding a cure," says Bentley Gubar, one of Rockstar's original members. "But Christopher is the only person I know saying people need help now. "不符。 答案:C B Ask someone what they have done to help the environment recently and they will almost certainly mention recycling. Recycling in the home is very important of course. However, being forced to recycle often means we already have more material than we need. We are dealing with the results of that over-consumption in the greenest way possible, but it would be far better if we did not need to bring so much material home in the first place. The total amount of packaging increased by 12% between 1999 and 2005. It now makes up a third of a typical household’s waste in the UK. In many supermarkets nowadays food items are packaged twice with plastic and cardboard. Too much packaging is doing serious damage to the environment. The UK, for example, is running out of it for carrying this unnecessary waste. If such packaging is burnt, it gives off greenhouse gases which go on to cause the greenhouse effect. Recycling helps, but the process itself uses energy. The solution is not to produce such items in the first place. Food waste is a serious problem, too. Too many supermarkets encourage customers to buy more than they need. However, a few of them are coming round to the idea that this cannot continue , encouraging customers to reuse their plastic bags, for example. But this is not just about supermarkets. It is about all of us. We have learned to associate packaging with quality. We have learned to think that something unpackaged is of poor quality. This is especially true of food. But is also applies to a wide range of consumer products, which often have far more packaging than necessary. There are signs of hope. As more of us recycle, we are beginning to realise just how much unnecessary material are collecting. We need to face the wastefulness of our consumer culture, but we have a mountain to climb. 61. What does the underlined phrase “over-consumption” refer to? A. Using too much packaging. B. Recycling too many wastes. C. Making more products than necessary. D. Having more material than is needed. 考点:: 解析:词义猜测题。根据文中的 We are dealing with the results of that over-consumption in the greenest way possible, but it would be far better if we did not need to bring so much material home in the first place. 可知划线部分意义应为“过度消费”,所以 D 项意义正确。 答案:D 62. The author uses figures in Paragraph 2 to show _______. A. the tendency of cutting household waste B. the increase of packaging recycling C. the rapid growth of super markets D. the fact of packaging overuse 考点:: 解析:作者用意猜测题。根据作者在文中使用的数字可知,作者使用数字的目的是为了让读 者清楚地理解过度包装的现实问题,所以 D 项正确。 答案:D 63. According to the text, recycling ______. A. helps control the greenhouse effect B. means burning packaging for energy C. is the solution to gas shortage D. leads to a waste of land 考点:: 解析:细节理解题。根据 If such packaging is burnt, it gives off greenhouse gases which go on to cause the greenhouse effect. Recycling helps, but the process itself uses energy. 可知 A 项正确。 答案:A 64. What can be inferred from Paragraph 4? A. Unpackaged products are of bad quality. B. Supermarkets care more about packaging. C. It is improper to judge quality by packaging. D. Other products are better packaged than food. 考点:: 解析:推理判断题。第四段讲述了人们对于包装不好的物品的固有认识,即:人们普遍认为 包装不好的物品质量肯定差,所以 C 项应为正确答案。 答案:C 65. What can we learn from the last paragraph? A. Fighting wastefulness is difficult. B. Needless material is mostly recycled. C. People like collecting recyclable waste. D. The author is proud of their consumer culture. 考点:: 解析:推理判断题。文章讲述了人们对于过度包装的危害的认识及人们应有的积极态度,所 以最后一段给我们带来了希望,但是最后一句 we have a mountain to climb. 提示我们事情不会 一蹴而就,所以 A 项正确。 答案:A C Rae Armantrout, who has been a poetry professor at the University of California San Diego(UCSD) for two decades, has won the 202016 Pulitzer Prize in the poetry category for her most recent book, “Versed”. “I’m delighted and amazed at how much media recognition that the Pulitzer brings, as compared to even the National Book Critics Award, which I was also surprised and delighted to win,” said Armantrout. “For a long time, my writing has been just below the media radar, and to have this kind of attention, suddenly, with my 2016th book, is really surprising. ” Armantrout, a native Californian, received her bachelor’s degree at UC Berkeley, where she studied with noted poet Denise Levertov, and her master’s in creative writing from San Francisco State University. She is a founding member of Language Poets, a group in American poetry that analyzes the way language is used and raises questions to make the reader think. In March, she won the National Book Critics Circle Award for “Versed. ” “This book has gotten more attention,” Armantrout said, “but I don’t feel as if it’s better. ” The first half of “Versed” focuses on the dark forces taking hold of the United States as it fought the war against Iraq. The second half looks at the dark forces casting a shadow over her own life after Armantrout was diagnosed with cancer in 2006. Armantrout was shocked to learn she had won the Pulitzer but many of her colleagues were not. “Rae Armantrout is a unique voice in American poetry,” said Seth Lerer, head of Arts and Humanities at UCSD. “Versed”, published by the Wesleyan University Press, did appear in a larger printing than her earlier works, which is about 2,700 copies. The new edition is scheduled to appear in May. 66. According to Rae Armantrout, ____________. A. her 2016th book is much better B. her winning the Pulitzer is unexpected C. the media is surprised at her works D. she likes being recognized by her readers 考点:: 解析:细节理解题。根据文章第二段的叙述可知,Armantrout 对于自己获得普利策奖是很惊讶 的,所以 B 项正确。 答案:B 67. Which of the following is true of Rae Armantrout? A. She published a poetry textbook. B. She used to teach Denise Levertov. C. She started a poets’ group with others. D. She taught creative writing at UC Berkeley. 考点:: 解析:细节理解题。根据文章第三段中的 She is a founding member of Language Poets, a group in American poetry 可知 C 项正确。 答案:C 68. What can we learn about “Versed”? A. It consists of three parts. B. It is mainly about the American army. C. It is a book published two decades ago. D. It partly concerns the poet’s own life. 考点:: 解析:细节理解题。根据文章中的 The second half looks at the dark forces casting a shadow over her own life after Armantrout was diagnosed with cancer in 2006. 可知 D 项正确。 答案:D 69. Rae Armantrout’s colleagues think that she ____________. A. should write more B. has a sweet voice C. deserves the prize D. is a strange professor 考点:: 解析:细节理解题。根据文章中的倒数第二段可知 C 项正确。 答案:C 70. What can we learn from the text? A. About 2,700 copies of “Versed” will be printed. B. Cancer made Armantrout stop writing. C. Armantrout got her degrees at UCSD. D. “Versed” has been awarded twice. 考点:: 解析:归纳判断题。根据文章中的 won the 202016 Pulitzer Prize in the poetry category for her most recent book, “Versed”. 以 及 In March, she won the National Book Critics Circle Award for “Versed. ”可知 D 项正确。 答案:D D Batteries can power anything from small sensors to large systems. While scientists are finding ways to make them smaller but even more powerful, problems can arise when these batteries are much larger and heavier than the devices themselves. University of Missouri(MU) researchers are developing a nuclear energy source that is smaller, lighter and more efficient. “To provide enough power, we need certain methods with high energy density(密度)”,said Jae Kwon, assistant professor of electrical and computer engineering at MU. “The radioisotope(放射性 同位素) battery can provide power density that is much higher than chemical batteries. ” Kwon and his research team have been working on building a small nuclear battery, presently the size and thickness of a penny, intended to power various micro / nanoelectromechanreal systems (M/NEMS). Although nuclear batteries can cause concerns, Kwon said they are safe. “People hear the word ‘nuclear’ and think of something very dangerous,” he said, “However, nuclear power sources have already been safely powering a variety of devices, such as pace-makers, space satellites and underwater systems. ” His new idea is not only in the battery’s size, but also in its semiconductor(半导体). Kwon’s battery uses a liquid semiconductor rather than a solid semiconductor. “The key part of using a radioactive battery is that when you harvest the energy, part of the radiation energy can damage the lattice structure(晶体结构) of the solid semiconductor,” Kwon said, “By using a liquid semiconductor, we believe we can minimize that problem. ” Together with J. David Robertson, chemistry professor and associate director of the MU Research Reactor, Kwon is working to build and test the battery. In the future, they hope to increase the battery’s power, shrink its size and try with various other materials. Kwon said that battery could be thinner than the thickness of human hair. 71. Which of the following is true of Jae Kwon? A. He teaches chemistry at MU. B. He developed a chemical battery. C. He is working on a nuclear energy source. D. He made a breakthrough in computer engineering. 考点:: 解析:细节理解题。根据第三段的叙述可知 Kwon 在研究一种原子能电池,所以 C 项正确。 答案:C 72. Jae Kwon gave examples in Paragraph 4_________. A. to show chemical batteries are widely applied. B. to introduce nuclear batteries can be safely used. C. to describe a nuclear-powered system. D. to introduce various energy sources. 考点:: 解析:细节理解题。根据文章第四段可知 Kwon 举例的目的是为了说明原子能电池是安全的, 所以 B 项正确。 答案:B 73. Liquid semiconductor is used to _________. A. get rid of the radioactive waste B. test the power of nuclear batteries. C. decrease the size of nuclear batteries D. reduce the damage to lattice structure. 考点:: 解析:细节理解题。根据 By using a liquid semiconductor, we believe we can minimize that problem 可知 D 项正确。 答案:D 74. According to Jae Kwon, his nuclear battery _______. A. uses a solid semiconductor B. will soon replace the present ones. C. could be extremely thin D. has passed the final test. 考点:: 解析:细节理解题。根据 Kwon said that battery could be thinner than the thickness of human hair 可知 C 项正确。 答案:C 75. The text is most probably a ________. A. science news report B. book review C. newspaper ad D. science fiction story 考点:: 解析:推理判断题。本文讲述了 Kwon 的原子能电池的有关情况,文章体裁应属于科普文章, 所以 A 项正确。 答案:A 阅读下面短文并回答问题,然后将答案写到答题卡相应的位置上(请注意问题后的字数 要求)。 [1]One out of many important things for the whole family to do is to have dinner together. Researchers began reporting the benefits of family dinner about a decade ago, focusing mainly on how it affects children. Studies show that those families who eat dinner at least 3 or 4 times a week together, benefit in many ways. [2]Eating together helps families achieve open communication and build stronger and healthier relationships. Children have better grades at school and are better adjusted as teens and adults, and the entire family enjoys healthier nutrition. For many families, eating dinner together proves to be good and effective way to reduce the risk of youth rates of addiction, and helps to raise healthier children. It is also a great time to share the events of the day, discuss news and ideas and just be together and enjoy each other's company. [3]During dinner time parents have better opportunity to show that they are the prior to their children. Sitting at the same table and sharing meals is where and when parents can find out more about their children's school performance, daily activities and attitudes toward life. When parents have all this information they can better direct their kids toward positive things in life. They also have better chance to reduce the possibility that children will get involved with alcohol, tobacco, and illegal drugs. [4]However, family researchers say that the benefits of family dinner by no means can be considered as automatic. Parents can sit down to dinner with their kids every day and achieve nothing. ____________________ if there is too much arguing going on, if there is no meaningful conversation or, what is even worse, if there is just plain silence during gathering. 76. What is the main idea of the text? (no more than 8 words) ________________________________________________________________ 77. Which sentence in the text is closest in meaning to the following one? By having dinner together family members can have free exchange of information and develop closer ties among them. _________________________________________________________________ 78. Fill in the blank in Paragraph 4 with proper words. (no more than 8 words) _________________________________________________________________ 79. What information can parents probably get about their children at family dinner? (no more than 12 words) _________________________________________________________________ 80. Translate the underlined sentence in Paragraph 3 into Chinese. ________________________________________________________________ 答案: 76. Families benefit from eating together. /Having dinner together is beneficial to families. 77. Eating together helps families achieve open communication and build stronger and healthier relationships. 78. No family dinner can benefit. /Family dinner may not benefit. 79. their children’s school performance, daily activities and attitudes toward life 80. 父母也有更好的机会来减少孩子沾染烟、酒和毒品的可能性。/父母也更有可能降低 子女染上吸烟、酗酒、吸毒等不良习惯的几率。 (2016 天津) A WELCOM Welcome to Windsor Castle, the oldest and largest occupied castle in the world. Windsor is one of the official residences(住 所) of the Queen, who sometimes stays here. Audio tours Free audio tours are available on leaving the Admission Centre at the start of your visit. There is a descriptive audio tour for blind and poor-sighted visitors. Guided tours Visitors can explore the history of the Castle through a tour of the Precincts with an expert guide. Tours depart at regular intervals throughout the day from the Courtyard and finish at the entrance to the State Apartments. Visitors with children For those visiting with children, a special family tour and various activities are offered during school holidays and at weekends. Please Shopping Shops offer a wide range of souvenirs designed for the Royal Collection, including books, postcards, china, jewellery, and children’s toys. Please ask at the Middle Ward shop about our home delivery service. Refreshments Bottled water can be purchased from the Courtyard and Middle Ward shops. Form April to September ice cream is also available. Visitors wishing to leave the Castle for refreshments in the town may obtain re-entry permits from the castle shops. Eating and drinking are not permitted in the State Apartments or St George’s Chapel. Photography and mobile phones Non-commercial photography and filming are welcomed in the Castle. Photography, video recording and filming are not permitted inside the State Apartments or St George’s Chapel in note that, for safely reasons, pushchairs are not permitted in the State Apartments. However, baby carriers are available to borrow. St George’s Chapel Visitors arriving at the Castle after 15:00 from March to October are advised to visit St George’s Chapel first before it closes. consideration of other visitors. Security As Windsor Castle is a working royal palace, visitors and their belongings should get through airport-style security checks. For safety and security reasons a one-way system operates along the visitor route. 36. A visitor can apply for a free audio tour _________. A. in the Courtyard B. in the State Apartments C. at the Admission Center D. at St George’s Chapel 36. C. 细节理解题。在 Audio tours 标题下直接找到 the Admission Center。 37. What is specially offered to visitors with kids? A. A security guard. B. A pushchair. C. A free toy. D. A baby carrier. 37. D. 细节理解题。在 visitors with babies 标题下有 baby carriers are available to borrow. 38. Who can get re-entry permits? A. Visitors wishing to eat outside the Castle. B. Visitors buying gifts in the castle shops. C. Visitors buying water from the Courtyard. D. Visitors eating outside St George’s Chapel. 38. A. 细节理解题。在 Refreshments 标题下有 Visitors wishing to leave the castle for refreshments in the town may obtain the re-entry permits from the castle shops. 39. Why are visitors required to turn off their mobile phones? A. To ensure the safety of others. B. To ensure the security of the Castle. C. To prevent them from disturbing others. D. To prevent the use of the built-in cameras. 39. C. 细节理解题。在 Photography and mobile phones 标题下有 Mobile phones must be switched off … inconsideration of other visitors. 40. In the last part, a “working royal palace” refers to one ___. A. still being constructed[ks5u. com] B. still used by the royal family C. where the Queen usually works D. where works of art are on show 40. B. 词义猜测题。根据第一部分中的句子 Windsor is one of the official residences of the Queen, who sometimes stays there. 可以猜测 B 项正确,C 项是不正确的,因为该项只是 Queen usually works。 B Why play games? Because they are fun, and a 1ot more besides. Following the rules…planning your next move. . . acting as a team member…these are all “game” ideas that you will come across throughout your life. Think about some of the games you played as a young child, such as rope-jumping and hide-and-seek. Such games are entertaining and fun. But perhaps more importantly, they translate life into exciting dramas that teach children some of the basic rules they will be expected to follow the rest of their lives, such as taking turns and cooperating (合作) . Many children’s games have a practical side Children around the world play games that prepare them for work they will do as grown-ups. For instance, some Saudi Arabian children play a game called bones. Which sharpens the hand-eye coordination(协调)needed in hunting. Many sports encourage national or local pride. The most famous games of all, the Olympic Games, bring athletes from around the world together to take part in friendly competition. People who watch the event wave flags, knowing that a gold medal is a win for an entire country, not just the athlete who earned it. For countries experiencing natural disasters or war, an Olympic win can mean so much. Sports are also an event that unites people. Soccer is the most popular sport in the world. People on all continents play it—some for fun and some for a living. Nicolette Iribarne, a Californian soccer player, has discovered a way to spread hope through soccer. He created a foundation to provide poor children with not only soccer balls but also a promising future. Next time you play your favorite game or sport, think about why you enjoy it, what skills are needed, and whether these skills will help you in other aspects of your life. 41. Through playing hide-and-seek, children are expected to learn to ________. A. be a team leader B. obey the basic rules C. act as a grown-up D. predict possible danger 41. B. 细节理解题。根据第一段中 Following the rules…planning your next move. . . acting as a team member…these are all “game” ideas that you will come across throughout your life. 可知 B 项 正确。 42. The underlined part in Paragraph 2 most probably means that games can________. A. describe life in an exciting way B. turn real-life experiences into a play C. make learning life skills more interesting D. change people’s views of sporting events 42. C. 词义猜测题。这部分前面的谓语动词是 translate 有“翻译,解释,转移,调动”的意思, 该部分后面有一个定语从句,前后结合就可以理解这部分的意思了。 43. According to the passage, why is winning Olympic medals so encouraging? A. It inspires people’s deep love for the country. B. It proves the exceptional skills of the winners. C. It helps the country out of natural disasters. D. It earns the winners fame and fortune. 43. A. 细节理解题。根据第四段中的 People who watch the event wave flags, knowing that a gold medal is a win for an entire country, not just the athlete who earned it. 可知 A 项正确。 44. Iribarne’s goal of forming the foundation is to _______. A. bring fun to poor kids B. provide soccer balls for children C. give poor kids a chance for a better life[ks5u. com] D. appeal to soccer players to help poor kids 44. C. 推理判断题。根据倒数第二段中 He created a foundation to provide poor children with not only soccer balls but also a promising future. 可推出 C 项是正确的。 45.What can be inferred from the passage? A.Gamed benefit people all their lives. B.Sports can get all athletes together. C.People are advised to play games for fun. D.Sports increase a country’s competitiveness. 45. A. 推理判断题。整段分析做游戏的好处,不仅儿童、成人也可以玩游戏。由此可以推断有 些对人终身有益。 C In the kitchen of my mother’s houses there has always been a wooden stand(木架)with a small notepad(记事本)and a hole for a pencil. I’m looking for paper on which to note down the name of a book I am recommending to my mother. Over forty years since my earliest memories of the kitchen pad and pencil, five houses later, the current paper and pencil look the same as they always did. Surely it can’t be the same pencil? The pad is more modern, but the wooden stand is definitely the original one. “I’m just amazed you still have the same stand for holding the pad and pencil after all these year. ” I say to her, walking back into the living-room with a sheet of paper and the pencil. “You still use a pencil. Can’t you afford a pen?” My mother replies a little sharply. “It works perfectly well. I’ve always kept the stand in the kitchen. I never knew when I might want to note down an idea, and I was always in the kitchen in these days. ” Immediately I can picture her, hair wild, blue housecoat covered in flour, a wooden spoon in one hand, the pencil in the other, her mouth moving silently. My mother smiles and says, “One day I was cooking and watching baby Pauline, and I had a brilliant thought, but the stand was empty. One of the children must have taken the paper. So I just picked up the breadboard and wrote it all down on the back. It turned out to be a real breakthrough for solving the mathematical problem I was working on. ” This story—which happened before I was born—reminds me how extraordinary my mother was, and is also a gifted mathematician. I feel embarrassed that I complain about not having enough child-free time to work. Later, when my mother is in the bathroom, I go into her kitchen and turn over the breadboards. Sure enough, on the back of the smallest one, are some penciled marks I recognize as mathematics. Those symbols have traveled unaffected through fifty years, rooted in the soil of a cheap wooden breadboard, invisible(看不到的)exhibits at every meal. 46.Why has the author’s mother always kept the notepad and pencil in the kitchen? A.To leave messages. B.To list her everyday tasks. C.To note down maths problems. D.To write down a flash of inspiration. 46. D. 细节理解题。从 I never knew when I might want to note down an idea 到下一段的 and I had a brilliant thought, but the stand was empty. 可知正确答案时 D. 47. What is the author’s original opinion about the wooden stand? A. It has great value for the family. B. It needs to be replaced by a better one. C. It brings her back to her lonely childhood. D . It should be passed on to the next generation. 47. B. 根据第三段中 I’m just amazed you still have the same stand for holding the pad and pencil after all these year. 可推断,作者认为 the wooden stand 该换成更好的了。 48. The author feels embarrassed for . A. blaming her mother wrongly. B. giving her mother a lot of trouble. C. not making good use of time as her mother did. D. not making any breakthrough in her field. 48. C. 细节理解题。最后一段中有 I feel embarrassed that I complain about not having enough child-free time to work. 可知答案。 49. What can be inferred from the last paragraph? A . The mother is successful in her career. B. The family members like traveling. C. The author had little time to play when young. D. The marks on the breadboard have disappeared. 49. A. 推理判断题。根据最后一段的第一句 This story—which happened before I was born—reminds me how extraordinary my mother was, and is also a gifted mathematician. 推断。 50. In the author’s mind ,her mother is . A. strange in behavior. B. keen on her research. C. fond of collecting old things. D. careless about her appearance. 50. B. 推理判断题。作者通过 a wooden stand(木架)with a small notepad(记事本)and a hole for a pencil 描写了一位母亲对事业的执着和热爱,由此可以推断 B 项正确。 D To err is human. To blame the other guy is even more human. Common sense is not all that common. Why tell the truth when you can come up with a good excuse? These three popular misquotes(戏谑的引语)are meant to be jokes, and yet they tell us a lot about human nature . To err, or to make mistakes, is indeed a part of being human, but it seems that most people don’t want to accept the responsibility for the problem. Perhaps it is the natural thing to do . The original quote about human nature went like this:” To err is human, to forgive, divine(神圣 的). ”This saying mirrors an deal people should be forgiving of others’ mistakes. Instead, we tend to do the opposite –find someone else to pass the blame on to. However, taking responsibility for something that went wrong is a making of great maturity. Common sense is what we call clear thought. Having common sense means having a good general plan that will make things work well, and it also means staying with the plan. Common sense tells you that you take an umbrella out into a rainstorm, but you leave the umbrella home when you hear a weather forecast for sunshine.Common sense does not seem to be common for large organizations, because there are so many things going on that one person cannot be in charge of everything. People say that in a large company,”the right hand does not know what the left hand is doing. ” And what is wrong with a society that thinks that making up a good excuse is like creating a work of art? One of the common problems with making excuses is that people, especially young people. get the idea that it’s okay not to be totally honest all the time. There is a corollary(直接推 论)to that: if good excuse is “good”even if it isn’t honest, then where is the place of the truth? 51. According to tile passage, which of following seems the most human? A. To search for truth. B. To achieve one’s ideal C. To make fun of others’mistakes.D. To criticize others for one’s own error. 51. D. 细节理解题。引言后第二段最后一句话可知:一个人负责公司的全部工作属于非常识。 52.According to the author, what is a sign of a man’s maturity? A.Doing things his own way. B.Bearing responsibility for his mistakes. C.Making as few mistakes as possible. D.Thinking seriously about his wrongdoing. 52. B. 细节理解题。根据第四段最后一句话 However, taking responsibility for something that went wrong is a making of great maturity. 可得出答案。 53.Which of the following is NOT based on common sense? A.A man tries to take charge of everything in a large company. B.A student goes out with an umbrella in stormy weather. C.A company’s next move follows a good plan. D.A lawyer acts on fine judgments. 53. A. 细节判断题。引言后第二段最后一句话可知:一个人负责公司的全部工作属于非常识。 54. What is the author’s opinion about a good excuse? A. Making a good excuse is sometimes a better policy. B. Inventing a good excuse needs creative ideas. C. A good excuse is as rewarding as honesty. D. Bitter truth is better than a good excuse. 54. D. 推理判断题。从全文最后一句话直接得到答案“追求真理比寻找理由更为重要” 55. What would be the best title for his passage? A. A Mirror of Human Nature B. To Blame or to Forgive C. A Mark of Maturity D. Truth or Excuse 55. A. 主旨大意题。从全文和引言后第一段倒数第二句可知文章标题:人性的镜子。 引言后第一段第三句可知:批评别人的错误 202016 年普通高等学校招生全国统一考试(天津卷) 英 语 笔 试 第Ⅱ卷 注意事项: 1. 用黑色墨水的钢笔或签字笔将答案写在答题卡上。 2. 本卷共 6 小题,共 35 分。 第三部分:写作 第一节:阅读表达(共 5 小题;每小题 2 分,满分 2016 分) 阅读下面短文,并按照题目要求用英语回答问题。 When Dan Shaw gets up from the sofa in his home, Cuddles is never far away. When he wants to go outside, he doesn’t take Cuddles out for walk—Cuddles takes him for a walk. Cuddles is clearly no ordinary family pet. It is a two-foot-high miniature horse and serves as the guiding eyes of Shaw, who is blind. When Shaw lost his sight, his wife suggested he apply for a guide dog. Shaw, an animal lover, said he couldn’t bear to part with a dog (which usually lives about eight to ten years) and get used to a new one, perhaps several times in his life. Then Shaw heard of a program about the tiny guide horses. He learned that the horse possess many qualities that that make them an excellent choice for guiding people. They are clean friendly, smart and have great memories. They can be trained to remain calm in noisy and crowded places. Best of all, they live for 25-25years, which would enable Shaw to have the same guide companion for most or all of his life. Shaw immediately applied to be and was accepted as the first person to receive a guide horse. The instant he met Cuddles, he knew he was making the right choice. Then he began his training. Through training ,Shaw and Cuddles learned to find way on busy streets, step over curbs(便道 沿儿)and find elevator buttons. Cuddles even demonstrated(显示)its ability to step in front of Shaw and block him, to prevent him from walking into a dangerous situation. The little horse also expertly led Shaw through busy shopping malls. They got along without any difficulties. Now Shaw is confident that Cuddles will change his life for the better. 56. What’s the meaning of the underlined word “miniature” in Paragraph 1?(No more than 2 words) 57. Why didn’t Shaw want to choose a dog as his guide? (No more than 15 words) 58. What does Paragraph 3 mainly tell us? (No more than 2016 words) 59. How does Shaw feel about his future life with the help of Cuddles? (No more than 2016 words) 60. In your opinion, what is the ideal relationship between man and animals? (No more than 20 words) 答案解析: 56. 第三段第一个字 TINY. 从构词法也可判断 Tiny LittlesmallVery small. 57. 第二段第二句话可知 A dog’s life is short and Shaw could not bear to part with it. 58. 第三段第二句话可知 The Tiny, horses possess have many good qualities to guide people. On the Tiny horses possess have many qualities to be ideal guides, 59. 文章最后一句话可知 He feels that his future life will he better/easier, 60. 属于概括题,导盲马和主人之间属于互相帮助的关系。 They should be good friends and help each other, Or: They should get along as friends and take care of each other. (2016 四川) A I grew up in a house where the TV was seldom turned on and with one wall in my bedroom entirely lined with bookshelves, most of my childhood was spent on books I could get hold of. In fact, I grew up thinking of reading as natural as breathing and books unbelievably powerful in shaping perspectives (观点) by creating worlds we could step into, take part. in. and live in. With this unshakable belief, I, at. fourteen, decided to become a writer. Here too, reading became useful. Every writer starts off knowing that he has something to say, but being unable to find the right ways to say it. He has to find his own voice by reading widely and discovering which parts of the writers he agrees or disagrees with, or agrees with so strongly that it reshapes his own world. He cannot write without loving to read, because only through reading other people’s writing can one discover what works, what doesn’t and, in the end, together with lots of practice, what voice he has. Now I am in college, and have come to realize how important it is to read fiction (文学作 品).As a. law student, my reading is in fact limited to subject matter—the volume (量) of what I have to read for classes every week means there is little time to read anything else. Such reading made it all the clearer to me that I live in a very small part in this great place called life. Reading fiction reminds me that there is life beyond my own. It allows me to travel across the high seas and along the Silk Road, all from the comfort of my own armchair, to experience, though secondhand, exciting experiences that I wouldn't necessarily be able to have in my lifetime. 41. What can be inferred about the author as a child? A. He never watched TV. B. He read what he had to. C. He found reading unbelievable. D. He considered reading part of his life. 42. The underlined word "voice" in the second paragraph most probably means “ ”. A. an idea B. a sound quality C. a way of writing D. a world to write about 43. What effect does reading have on the author? A. It helps him to realize his dream. B. It opens up a wider world for him. C. It makes his college life more interesting. D. It increases his interest in worldwide travel. 44. Which of the following can be the best title of this text? A. Why do I read? B. How do I read? C. What do I read? D. When do I read? 语篇解读:本篇文章是记叙文。作者通过自己的亲身体验,讲述读书使人形成自己的独立观 点,读书使人开扩视野的重要作用。 41. 答案:D 考点::细节理解题 解析:根据首段第一二句, “I grew up in a house… with one wall in my bedroom entirely lined with bookshelves…. I grew up thinking of reading as natural as breathing”可知作者是在一个以书 为伴的环境中长大的,读书像呼吸那样已经成为生活必需。 42. 答案:C 考点::词义推测题。 解析:根据文中 “. . . but being unable to find the right ways to say it. He has to find his own …”可 以推断出 voice 所表达的意思是“写作方式”。 43. 答案 B 考点::细节理解题 解析:由文章的最后一句:“It allows me to travel across the high seas and along the Silk Road, all from the comfort of my own armchair, to experience, though secondhand, exciting experiences that I wouldn't necessarily be able to have in my lifetime. ”可得出答案 44. 答案:A 考点::主旨大意题。 解析:文章作者主要以自己的亲身体验讲述读书的重要性。因此 Why do I read?“我为什么读 书?”作题目最佳。 B Boiler rooms are often dirty and steamy, but this one is clean and cool. Fox Point is a very new47-unit living building in South Bronx, one of the city’s poorest areas. Two-thirds of the people living there are formerly (以前) homeless people, whose rent is paid by the government. The rest are low-income families. The boiler room has special equipment, which produces energy for electricity and heat. It reuses heat that would otherwise be lost to the air, reducing carbon emissions(碳排 放)while also cutting costs. Fox Point is operated by Palladia, a group that specializes in providing housing and services to needy, people. Palladia received support from Enterprise Community Partners (ECP), which helps build affordable housing by providing support to housing developers. ECP has created national standards for healthy, environmentally (环境方面) clever and affordable homes which are called, the Green Communities Standards. These standards include water keeping, energy saving and the use of environmentally friendly building materials. Meeting the standards increases housing construction costs by 2%, which is rapidly paid back by lower running costs. Even the positioning of a window to get most daylight can help save energy. Michael. Bloomberg, New York's mayor, plans to create 165,000 affordable housing units for500,000 New Yorkers. Almost 80% of New York City’s greenhouse-gas emissions come from buildings, and 40% of those are caused, by housing. . So he recently announced that the city’s Department of Housing and Preservation and Development (DHPD) , whose duty is to develop and keep the city’s supply of affordable housing, will require all its new projects to follow ECP’s green standards. Similar measures have been taken by other cities such as Cleveland and Denver, but New York’s DHPD is the largest city developer of affordable housing in the country. 45. What is the purpose of describing the boiler room in the first paragraph? A. To explain the measures the city takes to care for poor people. B. To suggest that affordable housing is possible in all areas. C. To show how the environment-friendly building works. D. To compare old and new boiler rooms. 46. What is an advantage of the buildings meeting the Green Communities Standards? A. Lower running costs. B. Costing less in construction. C. Less air to be lost in hot days. D. Better prices for homeless people. 47. It can be learned from the text that, A. New York City is seriously polluted B. people’s daily life causes many carbon emissions in New York City C. a great number of people in New York City don't have houses to live in D. some other cities have developed more affordable housing than New York City 48. What is the main purpose of this text? A. To call on people to pay more attention to housing problems. B. To prove that some standards are needed for affordable housing. C. To ask society to help homeless people and low-income families. D. To introduce healthy, environmentally clever-and affordable housing. 语篇解读:本篇文章为科技说明文。作者以 Fox Point 居住群的 boiler rooms 为切入点, 阐述此类房子的环保节能的优点,以及各城市特别是纽约队建造此类建筑的积极响应。 45. 答案:C 考点::作者的写作意图 解析:从第一段最后两句,作者描述开水房仅是从一个侧面展示整个建筑群的节能,环 保,可以得出答案。 46. 答案:A 考点::细节理解 解析:依据文章第三段倒数第二句:Meeting the standards increases housing construction costs by 2%, which is rapidly paid back by lower running costs. 可得到答案。句意为“达到这一 标准会将建筑成本提高 2%,但很快会被它的低成本运行成本补回” 47. 答案:B 考点::细节理解 解析:依据文章第四段第二句:Almost 80% of New York City’s greenhouse-gas emissions come from buildings, and 40% of those are caused, by housing. . 可得出答案 48. 答案:D 考点::作者的写作目的 解析:本文主要让读者了解这类健康、环保、经济类住房。 C Alex London Research Laboratory (ALRL) is part of Alex Co. , Ltd. , a major Australian medicine-making company. Opened in 1992, ALRL specialises in the development of new medicines for the treatment of heart diseases. A position is now open for a Research Operations Manager(ROM) to support our growing research team at the new laboratories in Hatfield,due to open in the autumn of 202016. Reporting to the Director,you will help set up and run the technical and scientific support services of our new laboratories now under construction.You will be expected to provide expert knowledge about and be in charge of all areas of ALRL’S Health and Safety,and to communicate (沟通)with support employees at ALRL’S laboratories based at University College London.Working closely with scientists and other operations and technical employees , you will manage a small number of research support employees providing services to help with the research activities to be carried out at the new laboratories. Candidates(申请人) will have experience of both management and research support/ technical services.Knowledge of research operations and excellent communication skills are necessary. Education to degree level is also desirable. If you are interested in this position,please send your CV(简历)to Alex London Research Laboratory,University College London,Hatfield,London,W1E 6B7 or by email to ALRL@alex. co. uk. For more Information.please visit www. alex. co. uk. 49.What can be learnt about the new laboratories from the text? A.They have not yet been set up. B.They are in Hatfield,Australia. C.They belong to University College London. D.They are new workplaces for Australian researchers only. 50.What are the duties of a ROM,according to the text? A.a,b,c. B.a,b,d. C.b,c,d. D.a,c,d. 51.What does the fourth paragraph mainly talk about? A.The technical skills of a would—be ROM. B.The practical experience of a would—be ROM. C.The personal information of a would—be ROM. D.The necessary requirements for a would—be ROM. 52.What is the purpose of the text? A.To describe the job of a ROM. B.To provide information about ALRL. C.To announce an open position at ALRL. D.To make known the opening of the new laboratories. 【语篇解读】本篇文章为应用性文题。提供了 Alex Co. , Ltd 招聘一名 ROM 的信息,并对要 招聘的职位提出了具体的要求。 49.答案 A考点::细节理解题。 解析:根据文章第二段可知新的实验室还未建立。 50.答案 C 考点::细节理解题。 解析:依据文章第三自然段,You will help set up and run the technical and scientific support a. To carry out research activities b. To be in charge of ALRL’s Health and Safety c. To help run the technical and scientific support services d. To manage a small number of research support employees 高考资源网 services ……;You will be expected to……be in charge of all areas of ALRL’s Health and Safety; ……you will manage a small number of research support employees ……可得答案。 51.答案 D 考点::主旨大意题。 解析:依据第四段提供的信息,申请人要有管理和技术服务的经验、研究操作的知识及极好 的交际技能等可知本段主要谈及对应聘职位者的具体能力要求。 52.答案 C考点::写作目的题。 解析:本篇文章为广告招聘。目的自然是让大家了解 ALRL 公司有一公开招聘的职位。 D Somali pirates(海盗) robbed three Thai fishing ships with 77 sailors on board nearly l,200 Miles off the Somali coast,the farthest-off-shore attack to date,an officer said Tuesday. Pirates have gone farther south and east in answer to increased patrols(巡逻)by warships off the Somali shore. The robbing of the three ships Sunday was about 600 miles outside the normal operation area for the international force,said a spokesman. The spokesman said the attack so far out at sea was a clear sign that the international patrols against pirates were having a “marked effect on pirate activity in the area”. “Once they start attacking that far out,you’re not even really talking about the Somali basin or areas of water that have any connection with Somalia,” said an officer,Roger Middleton.“Once you’re that far out,it's just the Indian Ocean,and it means you’re looking at trade going from the Gulf to Asia,from Asia to South Africa. ” "This is the farthest robbing to date.They are now operating near the Maldives and India,” said another officer. The three ships--the MV Prantalay 11,12,and l 4—had 77 members on board in total.All of them are Thai, the spokesman said.Before the Sunday robbing,pirates held l l ships and 228 sailors. Pirates have increased attacks over the past year in hopes。f catching more dollar payments. Because of increased patrols and defenses on board ships,the success rate(率)has gone down, though the number of successful attacks has stayed the same year over year. 53. The pirate attack reported in the text happened .A.far out in the Indian Ocean B.in the normal patrol area C.near the Somali, coast D.in the south of Africa 54.According to the text, which can best describe the situation of the pirate problems? A.More goods on board are lost. B.Pirate attacks happen in a larger area now. C.The number of attacks has stayed the same these years. D.Pirate attacks are as serious as before along the Somali coast. 55.Which is true about the warship patrols according to the text? A.The patrols are of little effect. B.The patrols are more difficult. C.More patrols are quite necessary even in Asia. D.The patrols only drive the pirates to other areas. 56.How many sailors were held by the pirates up to the time of the report? A.228. B.77. C.383. D.305. 语篇分析:本篇文章为新闻报道类文体。报道索马里海盗周日抢劫三艘泰国渔船,并引用部 分官员的话,让读者了解当前的索马里海盗的形势。 53. 答案:A 考点::细节理解 解析:由 “The robbing of the three ships Sunday was about 600 miles outside the normal operation area for the international force…”以及 “Once you’re that far out,it's just the Indian Ocean”句意为 “这次海盗袭击发生在国际保卫队正常保护区域六百英里外”,“一旦你到了那么远,那就是 印度洋了”可得出答案。 54. 答案:B 考点:: 主旨大意 解析:文章主要报道发生在周末的对泰国渔船的袭击,就此事引出索马里海盗的袭击已经超 越国际保护队的正常护卫范围,而进入更深的海域。 55. 答案:B 考点::细节理解 解析:依据… patrols against pirates were having a “marked effect on pirate activity in the area”及 “Once you’re that far out,it's just the Indian Ocean,and it means you’re looking at trade going from the Gulf to Asia,from Asia to South Africa. ”可知巡逻明显见效,又有一旦再超过那么远,那就 是印度洋,那就意味着你在看管从波斯湾到亚洲,从亚洲到南美洲的所有行线。可知应是巡 逻难度加大了。 56.答案 D考点::推理计算题。 解析:由文章首句 Somali pirates robbed three Thai fishing ships with 77 sailors on board……及 Before the Sunday robbing, pirates held 11 ships and 228 sailors. 可知到发报到为止,索马里海盗 应劫持水手 77+228=305 人。 E Fear plays no part in this latest problem.“I’m not afraid I'll lose my children.I won't lose my children. We live together, and nothing, nothing,” she repeats, her voice rising when speaking to John's lawyer, “will stop me from being with my children. A law? Year, right. Don't disturb me when it comes to my children. You are never going to win. If John wants to see them, I cannot stop him. He's their father—I want them to see him! However, his visit in his present condition will disturb the children's stable (稳定的) life.” John, who has spent much of his time in California recently, has only hired a house for himself in Pennsylvania, according to his lawyer. "If he'd like to stick to a regular life, I'd be more than happy to do that," Jane says. "The best thing for any child of a divorced (离婚的) parent is a stable life. I want nothing more than for him to set up a stable life for himself, so that he can be part of making our children's lives more stable. " While matters of money and care won't be settled for weeks, Jane hardly puts her life on hold. Last week the ABC network announced that Jane would return with her own show, Twist of Jane, in which she gives advice to other moms.Jane and her eight children will also return to ABC in a series of Jane Plus 8 specials showing them on various adventures.It’s a rest for Jane.who insists that she needs her new,busy life to provide for her family.“I have to lead such a life.and I’m thankful that I’ve built it to the top where now I can support my children.”she says. And whether a bellicose(好斗的)dance judge or a bellicose former wife,Jane plans to keep 0n facing attacks on her. “Some people try to knock me down—only to make me more fierce, more protective,more determined to do better,”she says.“Go ahead,take me on.This will just make me stronger.” 57. According to Jane in the first paragraph,the low A. can’t take her children away from her B. can’t do anything with the case C. will best settle the problem D. will disturb her children 58. What does “to do that” refer to in the second paragraph? A. To live with John. B.To hire a house for John. C.To allow John to see the children D.To ask John to set up a regular life. 59. What does the underlined sentence in the third paragraph mean? A.Jane’s life is very difficult. B.Jane continues to live as usual. C.Jane almost can’t control her lire D.Jane works very hard to live a happy life. 60. Which word can best describe Jane? A. Proud. B.Fierce. C.Determined. D.Independent 语篇解读:本文为记叙文。主要记叙 Jane 在和丈夫离婚后下决心面对一切困难,争取孩子的 抚养权,并在婚后钱财和孩子抚养等问题短期内难以解决的情况下努力生活的故事。 57. 答案:C 考点::细节理解 解析:依据文章第一段第5,6 句 “A law? Year, right. Don't disturb me when it comes to my children. You are never going to win. ”可知 58. 答案:C 解析:根据上一段的 “If John wants to see them…However, his visit in his present condition …”可 知以 John 现在的状况不适合见孩子们,结合画线词所在句 “if he’d like to stick to a regular life” 可以推断出如果 John 生活稳定了 Jane 允许他见孩子们。 59. 答案:B 考点::考查学生的推断能力 解析:依据第三段首句 “While matters of money and care won't be settled for weeks, Jane hardly puts her life on hold. ”此处 while 引导让步状语从句,意为“尽管钱和抚养的问题在几周内难以 解决,……”划线部分因为 Jane 几乎不让生活受到影响。因此正确答案为 B 60. 答案:C 考点::推理判断题。 解析: Jane 面对一切压力,坚决要求孩子的抚养权;钱和孩子的抚养短期内不能解决的情况 下,努力克服困难维持生活都表现了她的坚强决心。 第二节 根据对话内容,从对话后的选项中选出能填入空白处的最佳选项,并在答题卡上 将该项涂黑,选项中有两项多余选项。(共五小题;每小题 5 分,满分 2016 分) —Mum, my old bike has gone wrong again. What shall I do? —61 — Great! But can I have a mountain bike? — Why a mountain bike? There’re no mountains around here. — Mountain bikes are stronger, and they are better at going up hills. But I … — 62 — Money. Do you know how much they cost? At least $300 each! How can I get so much money? — 63 — Over a year! But I need one now. — 64 — I suppose about $ 150 each. — Well, that’s sounds more reasonable. If you save $ 5 a week, you’ll have enough in about six months. — 65 — Well, go head with your saving and I’ll make up the rest in six months. Now, go and get your old bike repaired. A. No problem. B. Buy a new one. C. Is that expensive? D. What’s troubling? E. Mm…, but I still hope… F. How much does an ordinary bike cost? G. You can save up the money, say $ 5 a week, then in just over a year… 61.答案 B 解析:依据上句的“我的旧自行车又坏了,我该怎么办?”及下句“太好了,我可以买一辆 山地车吗?”可知妈妈建议买一辆新车。故选 B。62. 答案:D 解析:根据上句的 But I…说明孩子有困难及后句的 Money…,可确定 D. What’s troubling?合适 63. 答案:G 解析:依据下句的 Over one year! But I need one now. 可知上面用妈妈的建议孩子攒钱,每周 五美元,一年多就好的选择较为合适。 64. 答案:F 解析:根据下句的 I suppose about $ 150 each 可得出答案 65. 答案:E 解析:根据下句 Well, that’s sounds more reasonable. If you save $ 5 a week, you’ll have enough in about six months. 可知上文应该是孩子想让妈妈为他出点钱。 (2016 广东) Ⅲ. 阅读(共两节,满分 40 分) 第一节 阅读理解(共 15 小题;每小 2 分,满分 30 分) 阅读下列短文,从每题所给的 A、B、C 和 D 项中,选出最佳选项,并在答题卡上将该项涂黑。 When I was growing up in America, I was ashamed of my mother’s Chinese English. Because of her English, she was often treated unfairly. People in department stores, at banks, and at restaurants did not take her seriously ,did not give her good service ,pretended not to Understand her ,or even acted as if they did not hear her . My mother has realized the limitations of her English as well. When I was fifteen, she used to have me call people on phone to pretend I was she . I was forced to ask for information or even to yell at people who had been rude to her. One time I had to call her stockbroker (股票经纪人). I said in an adolescent voice that was not very convincing, “This is Mrs. Tan. . ” And my mother was standing beside me ,whispering loudly, “Why he don’t send me cheek already two week lone. ” And then , in perfect English I said : “I’m getting rather concerned . You agreed to send the check two weeks ago, but it hasn’t arrived. ” Then she talked more loudly. “What he want? I come to New York tell him front of his boss. ” And so I turned to the stockbroker again, “I can’t tolerate any more excuse. If I don’t receive the check immediately , I am going to have to speak to your manager when I am in New York next week. ” The next week we ended up in New York. While I was sitting there red-faced, my mother, the real Mrs. Tan, was shouting to his boss in her broken English. When I was a teenager, my mother’s broken English embarrassed me. But now, I see it differently. To me, my mother’s English is perfectly clear, perfectly natural. It is my mother tongue. Her language, as I hear it, is vivid, direct, and full of observation and wisdom. It was the language that helped shape the way I saw things, expressed ideas, and made sense of the world. 41. Why was the author’s mother poorly served? A. She was unable to speak good English. B. She was often misunderstood. C. She was not clearly heard. D. She was not very polite. 答案:A 试题分析:由文中 Because of her English, she was often treated unfairly. 可知 高考考点::考查事实细节 易错提醒:易错选 B 42. From Paragorph 2, we know that the author was . A. good a pretending B. rude to the stockbroker C. ready to help her mother D. unwilling to phone for her mother 答案:D 试题分析:由文中 ‘I was forced to ask for information or even to yell at people who had been rude to her”可知 高考考点::考查故事细节 易错提醒:易错 A 43. After the author made the phone call, . A. they forgave the stockbroker B. they failed to get the check C. they went to New York immediately D. they spoke to their boss at once 答案:B 试题分析:由文中 “I said in an adolescent voice that was not very convincing” 及 “If I don’t receive the check immediately , I am going to have to speak to your manager when I am in New York next week. ”和 “The next week we ended up in New York. ” 可推断出答案 高考考点::考查文章分析推测能力 易错提醒:易错选 C 44. What does the author think of her mother’s English now? A. It confuses her. B. It embarrasses her. C. It helps her understand the world. D. It helps her tolerate rude people. 答案:C 试题分析:由文中最后一段 But now, I see it differently. To me, --------and made sense of the world. 高考考点::考查分析推测能力 45. We can inter from the passage that Chinese English . A. is clear and natural to non-native speakers B. is vivid and direct to non-native speakers C. has a verv bad reputation in America D. may bring inconvenience in America 45. 答案:D 试题分析:由文中作者母亲的经历可以推测出 高考考点::考查文章分析推断能力 B When something goes wrong,it can be very satisfying to say,”Well,it’s so-and-so’s fault. ”or “I know I’m late,but it’s not my fault;the car broke down. ”It is probably not your fault,but once you form the habit of blaming somebody or something else for a bad situation,you are a loser. You have no power and could do nothing that helps change the situation. However,you can have great power over what happens to you if you stop focusing on whom to blame and start focusing on how to remedy the situation. This is the winner’s key to success. Winners are great at overcoming problems. For example, if you were late because your car broke down, maybe you need to have your car examined more regularly. Or, you might start to carry along with you the useful phone numbers, so you could call for help when in need. For another example, if your colleague causes you problems on the job for lack of responsibility or ability, find ways of dealing with his irresponsibility or inability rather than simply blame the person. Ask to work with a different person, or don’t rely on the person. You should accept that the person. Ask to work with a different person, or don’t rely on this person. You should accept that the person is not reliable and find creative ways to work successfully regardless of how your colleague fails to do his job well. This is what being a winner is all about—creatively using your skills and talents so that you are successful no matter what happens. Winners don’t have fewer problems in their lives; they have just as many difficult situations to face as anybody else. They are just better at seeing those problems as challenges and opportunities to develop their own talents. So, stop focusing on “whose fault it is. ” Once you are confident about your power over bad situations, problems are just stepping stop on for success. 46. According to the passage, winners . A. deal with problems rather than blame others B. meet with fewer difficulties in their livesC. have responsible and able colleagues D. blame themselves rather that others 答案:A 试题分析:由第一段“However, you can have great power over what happens to you if you stop focusing on whom to blame and start focusing on how to remedy the situation. ”可知 高考考点::考查文章大意概括及分析 47. The underlined word remedy in Paragraph 1 is closest in meaning to . A. avoid B. accept C. improve D. consider 答案:C 试题分析:由第一段最后两句可知 高考考点::考查推断能力 48. When your colleague brings about a problem, you should . A. find a better way to handle the problem B. blame him for his lack of responsibility C. tell him to find the cause of the problem D. ask a more able colleague for help 48. 答案:A 试题分析:由第二段 if your colleague causes you problems on the job for lack of responsibility or ability, find ways of dealing with his irresponsibility or inability rather than simply blame the person. 可知 高考考点::考查细节理解 易错提醒:易错选 C 49. When problems occur, winners take them as A. excuses for their failures B. barriers to greater power C. challenges to their colleagues D. chances for self-development 49. 答案:D 试题分析:文中最后一段. Winners don't have fewer problems in their lives; they have just as many difficult situation to face as anybody else. 高考考点::考查细节分析理解 易错提醒:易错选 C 50. Which of the following is the best title for the passage? A. A Winner’s Secret. B. A Winner’s Problem. C. A Winner’s Opportunity. D. A Winner’s Achievement. 50. 答案:A 试题分析:由文章的重点句:第二段开头句,第三段开头句等分析概括可知 A 最合适 高考考点::考查文章大意概括能力 易错提醒:易选错 B C Food sometimes gets poisoned with harmful things. A person who eats such food can get an illness called food poisoning. Food poisoning is usually not serious, but some types are deadly. The symptoms of food poisoning usually begin within hours of eating the poisoned food. Fever is one of the most common symptoms. Certain microorganisms (微生物)cause most types of food poisoning. Bacteria and other microorganisms can poison eggs, meat, vegetables, and many other foods. After entering the body, these tiny living things release(释放)poisons that make people sick. Some chemicals can also cause food poisoning. They are often added to food while it is being grown, processed, or prepared. For example, many farmers spray chemicals on crops to kill weeds and insects. Some people may have a bad reaction to those chemicals when they eat the crops. Some plants and animals contain natural poisons that are harmful to people. These include certain kinds of seafood, grains, nuts, seeds, beans, and mushrooms. When people handle food properly, the risk of food poisoning is very small. Microorganisms multiply rapidly in dirty places and in warm temperatures. This means that people should never touch food with dirty hands or put food on unwashed surfaces. Food should be kept in a refrigerator to stop microorganisms from growing. Meat needs to be cooked thoroughly to kill any dangerous microorganisms. People should also wash food covered with chemicals before eating it. Finally, people should not eat wild mushrooms or other foods that grow in the wild. Some of these foods may contain natural materials that are poisonous to humans. In addition, some types of fish can be poisonous. Most people recover from food poisoning after a few days of resting and drinking extra water. If people eat natural poisons, they must go to the hospital right away to have their stomachs emptied. 51. Which of the following statements is NOT true? A. Food when poisoned can make people sick. B. Food poisoning means death. C. Food poisoning comes in varieties. D. Food poisoning can be serious. 51. 答案:B 试题分析:由第一段中 “Food poisoning is usually not serious, but some types are deadly. ”可知 高考考点::考查细节分析 易错提醒:易错选 C 52. We know from the passage that the symptoms of food poisoning . A. are always accompanied by a fever B. are too common to be noted C. can be noticed within hours D. can he ignored 52. 答案:C 试题分析:由第一段最后两句话可知 高考考点::考查学生细节分析能力 易错提醒:易错选 A 53. Food poisoning can be caused by all the following EXCEPT . A. some chemicals B. low temperatures C. some tiny living things D. certain natural materials 53. 答案:B 试题分析:由第五段 Food should be kept in a refrigerator to stop microorganisms from gnawing. 可知 low temperatures 不能导致 Food poisoning 高考考点::考查事实细节分析理解 易错提醒:易错选 C 54. From Paragraph 5,we can learn that . A. mushrooms should not be eaten B. vegetables are safer than meat and seafood C. natural poisons are more dangerous than chemicals D. different types of food should be handled differently 答案:D 试题分析:由第五段第一句话可以推测出 D 恰当,A 太绝对 B。C 无细节支持 高考考点::考查推测分析能力 易错提醒:易选错 C 55. It can be inferred from the passage that . A. natural materials are safe in food processing B. chemicals are needed in food processing C. food poisoning can be kept under control D. food poisoning is out of control 55. 答案:C 试题分析:由文中第五段第一句话可推测出 C 正确 高考考点::考查推测能力 易错提醒:易错选 A 第二节 信息匹配(共 5 小题;每小题 2 分,满分 2016 分) 阅读下列应用文及相关信息,并按照要求匹配信息。请在答题卡上将对应题号的相应选 项字母涂墨。 首先请阅读下列国外媒体上的插图及提示性文字: A. B. Harry Potter stars add A tour of discovering Normandy magic to young rich. C. D. Do Hollywood stars Save Emergency Rooms guarantee a film’s success? for emergencies. E. F. Her theories on children’s Jenny McCarthy and Jim psychological problems Carrey, with Ms. McCarthy’s created a sensation. son,in an anti-vaccine rally. 以下是关于这些插图的简要评论。请把评论与相关插图及提示性文字匹配起来。 56. The debate has been raging for years over the safety of,and necessity for, childhood vaccinations, which has been so much so that it is termed“The Vaccine War”. The debate has only a few moments that might be inspiring to those who have been following this now familiar issue. 答案:F 试题分析:由评论中"The Vaccine War". 与 F 提示性文字 anti-vaccine rally 匹配得到 57. There are certainly benefits of using a star in a film. It makes the film easier to market. Stars also help sell more tickets and drive DVD sales, which are a big part of studio revenue However, a star does not guarantee success. The simple fact is that if you pay a star a greal deal of money for a film that people don’t want to see, then it won’t work. 答案:C 试题分析:由评论中 There are certainly benefits of using a star in a film 与 C 中图片及文字 stars gurrantee a film’s success 58. They are barely in their twenties and are already multimillionaires. At the age when many people are looking for their first job, the youngsters of The Sunday Times Rich List are buying country estates or jetting off to their overseas homes. Daniel Radcliffe, for example, who plays Harry Potter, has a fortune of ξ42 million, at 20. 答案:A 试题分析:由 “who plays Harry Potter, has a fortune of ξ42 million, at 20. ”与 A 图及提示文字 全部匹配 59. Millions of jobless Americans, who might be suffering in anxiety and lacking a sense of security, are showing up at emergency rooms of state- owned hospitals, contributing to a longer waiting time and a higher risk of cursory treatment by overworked doctors and nurses. 答案:D 试题分析:由 “are showing up at emergency rooms of state- owned hospitals” 与 D 中图片 emergency 及提示文字相匹配 60. Alice Miller, a psychology expert, who died at 87 at home in Provence, France, on April 14, repositioned the family as a central place of abnormal psychological function with her theory that parental power and punishment lay at the root of nearly all human problems. 答案:E 试题分析:由 “a central place of abnormal psychological function with her theory”与 E 中图片及 文字相匹配 (2016 全国Ⅰ) A Shakespeare's Birthplace and Exhibition of Shakespeare's World Welcome to the world-famous house where William Shakespeare was born in 1564 and where he grew up. The property(房产)remained in the ownership of Shakespeare’s family until 1806. The House has welcomed visitors traveling from all over the word ,for over 250 years. ◆ Enter though the Visitors’ Centre and see the highly-praised exhibition Shakespeare’s World, a lively and full introduction to the life and work of Shakespeare. ◆ Stand in the rooms-where Shakespeare grew up. ◆ Discover examples of furniture and needlework from Shakespeare’s period. ◆ Enjoy the traditional(传统的)English garden, planted with trees and flowers mentioned in the poet’s works. ○p The Birthplace is within easy walking distance of all the car parks shown on the map; nearest is Windsor Street(3 minutes’ walk). The House may present difficulties but the Visitors’ Centre, its exhibition, and the garden are accessible(可进入的)to wheelchair user. The Shakespeare Coffee House (opposite the Birthplace). 56. How much is the admission for a family of two grown-ups and two children? A. $9. 80 B. $12. 00 C. $14. 20 D. $16. 40 57. Where is the nearest parking place to Shakespeare’s Birthplace? A. Behind the exhibition hall. B. Opposite the Visitors’ Centre. C. At Windsor street. D. Near the Coffee House. OPENING TIMES: 20 Mar to 19 Oct Mon to Sat: 9:00am to 5:00pm Sun:9:30am to 5:00pm 20 Oct to 19Mar Mon to Sat:9:30am to 4:00pm Sun:2016:00am to 4:00pm ADMISSION: Adult £4. 90 Child £2. 20 Family £12. 0 (2 adults +up to 3 children) 58. A wheelchair user may need help to enter . A . the House B the garden C the Visitor’s Centre D. the exhibition hall B EDGEWOOD—Every morning at Dixie Heights High school, customers pour into a special experiment :the district’s first coffee shop run mostly by students with special learning needs. Well before classes start, students and teachers order Lattes, Cappuccinos and Hot Chocolates. Then, during the first period, teachers call in orders on their room phones, and students make deliveries. By closing time at 9. 20 a. m. , the shop usually sells 90drinks. “whoever made the chi tea, Ms. Schatzman says it was good, ”Christy McKinley , a second year student , announced recently, after hanging up with the teacher. The shop is called the Dixie PIT, which stands for Power in Transition. Although some of the students are not disabled, many are, and the PIT helps them prepare for life after high school. They learn not only how to run a coffee shop but also how to deal with their affairs. They keep a timecard and receive paychecks , which they keep in check registers. Special-education teachers Kim Chevalier and Sue Casey introduced the Dixie PIT from a similar program at Kennesaw Mountain High School in Georgia. Not that it was easy. Chevalier’s first problem to overcome was product-related. Should schools be selling coffee? What about sugar content? Kenton County Food Service Director Ginger Gray helped. She made sure all the drinks, which use non-fat milk, fell within nutrition(营养)guidelines. The whole school has joined in to help. Teachers agreed to give up their lounge(休息室)in the morning. Art students painted the name of the shop on the wall. Business students designed the paychecks. The basketball team helped pay for cups. 59. What is the text mainly about? A. A best-selling coffee. B. A special educational program. C. Government support for schools. D. A new type of teacher-student relationship. 60. The Dixie PIT program was introduced in order to . A. raise money for school affairs B. do some research on nutrition C. develop students’ practical skills D. supply teachers with drinks 61. How did Christy McKinley know Ms. Schatzman’s opinion of the chi tea? A. She met her in the shop. B. She heard her telling others. C. She talked to her on the phone. D . She went to her office to deliver the tea. 62. We know from the text that Ginger Gray . A. manages the Dixie PIT program in Kenton County B. sees that the drinks meet health standards C. teaches at Dixie Heights High School D. owns the school’s coffee shop C Along the river banks of the Amazon and the Orinoco there lives a bird that swims before it can fly, flies like a fat chicken, eats green leaves, has the stomach of a cow and has claws(爪)on its wings when young . They build their homes about 4. 6m above the river ,an important feature(特 征)for the safety of the young. It is called the hoatzin. In appearance,the birds of both sexes look very much alike with brown on the back and cream and red on the underside . The head is small, with a large set of feathers on the top, bright red eyes, and blue skin. Its nearest relatives are the common birds, cuckoos. Its most striking feature ,though, is only found in the young. Baby hoatzins have a claw on the leading edge of each wing and another at the end of each wing tip . Using these four claws ,together with the beak(喙),they can climb about in the bushes, looking very much like primitive birds must have done. When the young hoatzins have learned to fly ,they lose their claws. During the drier months between December and March hoatzins fly about the forest in groups of 20 to 30 birds, but in April, when the rainy season begins, they collect together in smaller living units of two to seven birds for producing purposes. 63.What is the text mainly about? A.Hoatzins in dry and rainy seasons. B.The relatives and enemies of hoatzins. C.Primitive birds and hoatzins of the Amazon. D.The appearance and living habits of hoatzins. 64.Young hoatzins are different from their parents in that . A.they look like young cuckoos B.they have claws on the wings C.they eat a lot like a cow D.they live on river banks 65.What can we infer about primitive birds from the text? A.They had claws to help them climb. B.They could fly long distances. C.They had four wings like hoatzins. D.They had a head with long feathers on the top. 66.Why do hoatzins collect together in smaller groups when the rainy season comes? A.To find more food. B.To protect themselves better. C.To keep themselves warm. D.To produce their young. D Science can't explain the power of pets, but many studies have shown that the company of pets can help lower blood pressure (血压) and raise chances of recovering from a heart attack, reduce loneliness and spread all-round good cheer. Any owner will tall you how much joy a pet brings. For some, an animal provides more comfort than a husband/wife. A 2002 study by Karen Allen of the State University of New York measured stress (紧张) levels and blood pressure in people - half of them pet owners –while they performed 5 minutes of mental arithmetic (算术) or held a hand in ice water. Subjects completed the tasks alone, with a husband/wife, a close friend or with a pet. People with pets did It best . Those tested with their animal friends had smaller change in blood pressure and returned Most quickly to baseline heart rates . With pets in the room ,people also made fewer math mistakes Than when doing in front of other companions. It seems people feel more released (放松)around Pets, says Allen, who thinks it may be because pets don’t judge. A study reported last fall suggests that having a pet dog not only raises your spirits but may also have an effect on your eating habits. Researchers at Northwesterm Memorial Hospital spent a Year studying 36 fat people and were put on a diet-and-exercise programs; a separate group of 56 fat people without pets were put on a diet program. On average,people lost about 11 pounds, Or 5% of their body weight . Their dogs did even better,losing an average of 12 pounds,more then 15%of their body weight . Dog owners didn’t lose any more weight than those without dogs but ,say Researchers,got more exercise overall –mostly with their dogs –and found it worth doing. 67.What does the text mainly discuss? A.What pets bring to their owners B.How pets help people calm down C.people’s opinions of keeping pets D.Pet’s value in medical research 68.We learn from the text that a person with heart disease has a better chance of getting well if ____________ A.he has a pet companion B.he has less stress of work C.he often dose mental arithmetic D.he is taken care of by his family 69.According to Allen, why did the people do better with pets around when facing stressful tasks? A.They have lower blood pressure. B.They become more patient. C.They are less nervous. D.They are in higher spirits. 70.The research mentioned in the last paragraph reports that A.people with dogs did more exercise B.dogs lost the same weight as people did C.dogs liked exercise much more than people did D.people without dogs found the program unhelpful E There were smiling children all the way. Charily they knew at what time the train passed their homes and they made it their business to stand along the railway, wave to complete strangers and cheer them up as they rushed towards Penang. Often whole families stood outside their homes and waved and smiled as if those on the trains were their favorite relatives. This is the simple village people of Malaysia. I was moved. I had always traveled to Malaysia by plane or car, so this was the first time I was on a train. I did not particularly relish the long train journey and had brought along a dozen magazines to read and reread. I looked about the train. There was not one familiar face. I sighed and sat down to read my Economics. It was not long before the train was across the Causeway and in Malaysia. Johore Baru was just another city like Singapore, so I was tired of looking at the crowds of people as they hurried past. As we went beyond the city, I watched the straight rows of rubber trees and miles and miles of green. Then the first village came into sight, Immediately I came alive; I decided to wave hack. From then on my journey became interesting. I threw my magazines into the waste basket and decided to join in Malaysian life. Then everything came alive. The mountains seemed to speak to me. Even the trees were smiling. I stared at everything as if I was looking at it for the first time. The day passed fast and I even forgot to have my lunch until I felt hungry. I looked at my watch and was surprised that it was 3:00 pm. Soon the train pulled up at Butterworth. I looked at the people all around me. They all looked beautiful. When my uncle arrived with a smile, I threw my arms around him to give him a warm hug (拥抱). I had never done this before. He seemed surprised and then his weather-beaten face warmed up with a huge smile. We walked arm in arm to his car. I looked forward to the return journey. 71. The author expected the train trip to be A. adventurous B. pleasant C. exciting D. dull 72. What did the author remember most fondly of her train trip? A. The friendly country people. B. The mountains along the way. C. The crowds of people in the streets. D. The simple lunch served on the train. 73. Which of the following words can best take the place of the word“relish”in the second paragraph? A. choose B. enjoy C. prepare for D. carry on 74. Where was the writer going? A. Johore Baru. B. The Causeway. C. Butterworth. D. Singapore. 75. What can we learn from the story? A. Comfort in traveling by train. B. Pleasure of living in the country. C. Reading gives people delight. D. Smiles brighten people up. 四、阅读理解 56-60 BCABC 61-65 CBDBA 66-70 DAACA 71-75 DABCD A 篇 56. 选 B。根据第二个表格中 Family £12. 0 (2 adults + up to 3 children),即是一家人(2 个 成年人+至多 3 个小孩)需要£12. 0。 57. 选 C。根据第二个表格右边第一段最后几个单词 nearest is Windsor Street (3 minutes' walk). 58. 选 A。根据第二个表格右边第二段 The House may present difficulties but the Visitors' Centre, its exhibition, and the garden are accessible(可进入的) to wheelchair user.“对于轮椅使用者来 说,可能进入房子有些困难,但是游客中心,展览厅还有花园都可以进入。” B 篇 59. 选 B。考查文章主题大意。主要看首段 customers pour into a special experiment: the district's first coffee shop run mostly by students with special learning needs 和第五段最后 the PIT helps them prepare for life after high school. 文章主要讲的是一个特殊的教育项目。 60. 选 C。第五和第六段说明这个教育项目是为学生走出社会做准备的,所以主要是锻炼学生 的实际动手能力。 61. 选 C。第四段,考查 hang up 的意思“挂电话”。 62. 选 B。倒数第三段。 C 篇 63. 选 D。考查文章主题大意。纵观全文,文章主要讲的是 hoatzin 麝雉,这种鸟类的外貌以 及生活习性。 64. 选 B。根据第一段 has claws(爪)on its wings when young,和第三段可知,hoatzin 在幼年期是 有爪子的,而长大学会飞之后就没有了。 65. 选 A。关键是 primitive“原始的,早期的”的意思。第三段第二句话。 66. 选 D。最后一段最后一句话。 D 篇 67. 选 A。考查主题大意。根据首段,很多研究表明,宠物可以降低血压,提高、心脏病恢复 的几率,减少孤独感和传播快乐。 68. 选 A。根据第一段,参见上题解析。 69. 选 C。第二段最后一句话,和宠物在一起人们感到更加的放松。 70. 选 A。最后一段最后一句话,虽然狗主的体重没有多大变化,但是却比没有养狗的人得到 更多的锻炼,因为是和他们的狗在一起,他们认为很值得。 E 篇 71. 选 D。根据第二段,作者是第一次坐长途火车,带了很多杂志看,可以看出作者一开始认 为坐火车是 dull(乏味的)。 72. 选 A。根据第一段,马来西亚人很热情,总会微笑着在站台上对火车里的陌生人挥手,欢 迎乘客来到马来西亚。作者很感动。 73. 选 B。参照第 71 题。作者开始并不享受火车之旅。 74. 选 C。倒数第二段,Soon the train pulled up at Butterworth. 火车在 Butterworth 停下来。 75. 选 D。文章主要讲述作者从看到马来西亚人总会在站台上笑对火车里的陌生人挥手后,心 情由无聊、乏味变成高兴。可以看出,微笑能让人提起精神。 (2016 江苏) A Usually, when your teacher asks a question, there is only one correct answer. But there is one question that has millions of current answers. That question is “What’s your name?” Everyone gives a different answer, but everyone is correct. Have you ever wondered about people’s names? Where do they come from? What do they mean? People’s first names, or given names, are chosen by their parents. Sometimes the name of a grandparent or other member of the family is used. Some parents choose the name of a well-known person. A boy could be named George Washington Smith; a girl could be named Helen Keller Jones. Some people give their children names that mean good things. Clara means “bright”; Beatrice means “one who gives happiness”; Donald means “world ruler”; Leonard means “as brave as a lion”. The earliest last names, or surnames, were taken from place names. A family with the name Brook or Brooks probably lived near brook(小溪);someone who was called Longstreet probably lived on a long, paved road. The Greenwood family lived in or near a leafy forest. Other early surnames came from people’s occupations. The most common occupational name is Smith, which means a person who makes things with iron or other metals. In the past, smiths were very important workers in every town and village. Some other occupational names are: Carter — a person who owned or drove a cart; Potter —a person who made pots and pans. The ancestors of the Baker family probably baked bread for their neighbors in their native village. The Carpenter’s great-great-great-grandfather probably built houses and furniture. Sometimes people were known for the color of their hair or skin, or their size, or their special abilities. When there were two men who were named John in the same village, the John with the gray hair probably became John Gray. Or the John was very tall could call himself John Tallman. John Fish was probably an excellent swimmer and John Lightfoot was probably a fast runner or a good dancer. Some family names were made by adding something to the father’s name. English-speaking people added –s or –son. The Johnsons are descendants of John; the Roberts family’s ancestor was Robert. Irish and Scottish people added Mac or Mc or O. Perhaps all of the MacDonnells and the McDonnells and the O’Donnells are descendants of the same Donnell. 56. Which of the following aspects do the surnames in the passage NOT cover? A. Places where people lived. B. People’s characters. C. Talents that people possessed. D. People’s occupations. 选 B,根据文中 4,5,6 三段可知只有 B 选项未提及 57. According to the passage, the ancestors of the Potter family most probably _______. A. owned or drove a cart B. made things with metals C. made kitchen tools or contains D. built houses and furniture 选 C 文中第六段 potter-a person who made pots and pans,可知 potter 意为一个制作茶壶和平底锅 的人,由此推断选 C 58. Suppose and English couple whose ancestors lived near a leafy forest wanted their new-born son to become a world leader, the baby might be named _______. A. Beatrice Smith B. Leonard Carter C. George Longstreet D. Donald Greenwood 选 D 根据文中倒数第 2 段可知一些人取名是根据特征而来。比如灰色头发的人取名 john Gray, 高个的人取名 john Tallman. 所以根据题意,居住在森林旁边的人取名为 Donald Greenwood 59. The underlined word “descendants” in the last paragraph means a person’s _____. A. later generations B. friends and relatives C. colleagues and partners D. later sponsors 选 A 根据最后一段第一二行“some family names were made by adding something to the father’s name. The Johnsons are descendants of John 可知 descendants 指的是后代 B It is reported that conservation groups in North America have been arguing about the benefits and dangers of wolves. Some groups believe wolves should be killed. Other people believe wolves must be protected so that they will not disappear from the wilderndss(荒野) For Killing Wolves In Alaska,the wolf almost disappeared a few years ago,because hunters were killing hundreds 0f them forsport . However.1aws were established to protect the wolves from sportsmen and people who catch the animals for their fur.So the woIf population has greatly increased. Now there are so many wolves that they are destroying their own food supply. A wolf naturally eats animals in the deer family. People in the wilderness also hunt deer for food.Many of the animals have been destroyed by the very cold winters recently and by changes in the wilderness plant life.When the deer can’t find enough food,they die. If the wolves continue to kill large numbers of deer , their prey( 猎 物 )will disappear some day.And the wolves will.too.So we must change the cycle of life in the wilderness to balance the ecology.If we killed more wolves,we would save them and their prey from dying out.We’d also save some farm animals. In another northern state , wolves attack cows and chickens for food . Farmers want the government to send biologists to study the problem.They believe it necessary to kill wolves in some areas and to protect them in places where there is a small woIf population. Against Killing Wolves If you had lived long ago , you would have heard many different stories about the dangerous wolf.According to most stories,hungry wolves often kill people for food.Even today,the stories of the“big bad woIf'"will not disappear. But the fact is wolves are afraid of people.and they seldom travel in areas where there is a human smell.When wolves eat other animals,they usually kill the very young.or the sick and injured . The strongest survive . No kind of animal would have survived through the centuries if the weak members had lived.And has always been a law of nature. Although some people say it is good sense to kill wolves ,we say it is nonsense!Researchers have found wolves and their prey living in balance. The wolves keep the deer population from becoming too large,and that keeps a balance in the wilderness plant life. The real problem is that the areas where wolves can live are being used bv people.Even if wilderness land is not used directly for human needs.the wolves can’t always find enough food . So they travel to the nearest source,which is often a farm.Then there is danger.The“big bad wolf”has arrived! And everyone knows what happens next. 60 . According to the passage , some people in North America favor killing wolves for all the following reasons EXCET that .A.there are too many wolves B.they kill large numbers deer C.they attack cows and chickens for food D.they destroy the wilderness plant life 选 D 根据文中小标题 For killing wolves 第一段第三行“so the wolf population has increased”第三 段第一句和第四段第一句话,可知选项 D 不是原因 61.Some people are against killing wolves because . A.wolves help to keep the ecological balance in the wildemess B.there is too small a wolf population in the wilderness C.there are too many deer in the wilderness D.wolves are afraid of people and never attack people 选 A 小标题 Against killing wolves 中第三段最后一句 62.According to those against killing wolves,when wolves eat other animals, . A.they never eat strong and healthy onesB.they always go against the law of nature C.they might help this kind of animals survive in nature D.they disturb the ecological balance in the wilderness 选 C 小标题 Against killing wolves 中第二段最后两行可知强者生存,弱者淘汰是自然法则 63.The last sentence“And everyone knows what happens next”implies that in such cases . A.farm animals will be in danger and have to be shipped away B.woIves will kill people and people will in turn kill them C.wolves will find enough food sources on farms D.people will leave the areas where wolves can live 选 B 文中最后一段可知狼居住的地方被人类占据,它们就会跑到就近的农场等地吃人,那么 人类就会反过来杀死它们 C BORDER CROSSINGS While there are no restrictions on zhe amount of money that you can bring across the border. you must report to both the US and Canadian border services amounts equal to or greater than $2016. 000. PERSONAL EXEMPTIONS(免税)ON PURCHASES AMERICANS RETURNING TO THE US Less than 48 hours: $ 200 US 48 hours or more: $ 800 US duty-free personal exemption. next $ 1. 000 US at 3% Including up to 20160 cigars and 20160 cigarettes. CANADIANS RETURNING TO CANADA Less than 24 hours: $ 50 CAN 48 hours or more: $ 400 CAN Including up to 20160 cigars and 200 cigarettes. 7 days or more: $ 750 CAN Including up to 20160 cigars and 200 cigarettes. DOCUMENTATION NEEDED FOR BORDER CROSSING LAND OR SEA TO THE US(INCLUDING FERRIES) A valid passport or passport card, or a NEXUS card. (A NEXUS card is a Trusted Traveler Program that provides quick travel for pre-approved,low risk travelers through special lanes. ) A recent Washington State, New York or BC driver’s license. Note: Children 15 years of age and younger require only a birth certificate or copy. (Certified copies are not required but are advised. ) AIR TRAVEL TO THE US A valid passport, an Air NEXUS card, or a U. S. Coast Guard Merchant Marine Document. 64. If a Canadian who is on a 7-day trip to New York buys $ 800 CAN worth of goods,how much should he pay tax on when returning home? A. $ 800 CAN B. $750 CAN C. $ 400 CAN D. $ 50 CAN 选 D。根据文中 Canadians Returning To Canada 的要求$750 CAN 是免税的,所以只需交$50 CAN 的关税 65. For an American citizen on a 2-day tour of Canada,how much tax does he have to pay on $ 1. 600 US worth of purchases when returning to the US? A. $ 24 US B. $ 48 US C. $52 US D. $ 200 US 选 A. 根据文中 Americans Returning To the US 的要求$800 US 免税,而 next $201600US at3% 得知除 800 以外的 800 到 201600 以内需交 3% 800 乘以 3%=24 66. What documentation should a couple with a 7-year-old child carry when they drive a car from Canada to America? A. A BC driver’s license, an Air NEXUS card, and a birth certificate. B. An Air NEXUS card, a U. S. Coast Guard Merchant Marine Document, and a birth certificate. C. Two vaild passport crads and a certified copy of a birth certificate. D. A NEXUS card, a U. S. Coast Guard Merchant Marine Document. ,and a certified copy of a birth certificate. 选 C 根据题目中 hey drive a car from Canada to America,用排除法可排除两项 A,B,而 D 错在 a certified copy D Imagine,one day,getting out of bed in Beijing and being at your office in Shanghai in only a couple of hours,and then,after a full day of work,going back home to Beijing and having dinner there. Sounds unusual , doesn't it? But it's not that unrealistic , with the development of China’s high—speed railway system.And that’s not a11.China has an even greater high—speed railway plan—to connect the country with Southeast Asia,and eventually Eastern Europe. China is negotiating to extend its own high·-speed railway network to up to 17 countries in 1 0 to 15 years,eventually reaching London and Singapore. China has proposed three such projects . The first would possibly connect Kunming with Singapore via Vietnam and Malaysia.Another could start in Urumqi and go through Kazakhstan and Uzbekistan,and possibly to Germany.The third would start in the northeast and go north through Russia and then into Western Europe. If China’s plan for the high-speed railway goes forward,people could zip over from London to Beiling in under two days. The new system would still follow China’s high—speed railway standard.And the trains would be able to go 346 kilometers an hour,almost as fast as some airplanes. China’s bullet train(高速客车),the one connecting Wuhan to Guangzhou,already has the World’s fastest average speed.It covers 1,069 kilometers in about three hours. Of course,there are some technical challenges to overcome.There are so many issues that need to be settled,such as safety,rail gauge(轨距),maintenance of railway tracks.So,it’s important to pay attention to every detail. But the key issue is really money.China is already spending hundreds of billions of yuan on domestic railway expansion. China prefers that the other countries pay in natural resources rather than with capital investment.Resources from those countries could stream into China to sustain development. It’11 be a win-win project. For other countries,the railway network will definitely create more opportunities for business,tourism and so on,not to mention the better communication among those countnes. For China , such a project would not only connect it with the rest of Asia and bring some much-needed resources,but would also help develop China’s far west.We foresee that in the coming decades , millions of people will migrate to the western regions , where the land is empty and resources unused.With high-speed trains,people will set up factories and business centers in the west once and for a11.And they’11 trade with Central Asian and Eastern European countries. 67.China’s new high-speed railway plan will be a win-win project because . A.China will get much-needed resources and develop its western regions B.China and the countries involved will benefit from the project in various ways C.China will develop its railway system and communication with other countries D. the foreign countries involved will develop their railway transportation,business and tourism 选 B 在文章倒数第二段中有明确答案 68.According to the passage,the greatest challenge to the new high-speed railway plan is . A.technical issues B.safety of the systemC.financial problems D.maintenance of railway tracks 选 C . 在文章第 8 段看到 There are so many issues that need to be settled. . . 及第 9 节中 But the key issue is really money 69.Which of the following words best describes the author’s attitude towards China’s high-speed railway plan? A.Critical. B.Reserved. C.Doubtful D. Positive. 选 D . 尽管在文章第 8,9 段中 提到存在的问题,但从最后两段中不难看出作者是持支持的态 度 70.Which of the following might be the best title for the passage? A.New Railway Standards B.Big Railway Dreams C.High—speed Bullet Trains D.International Railway Network 选 B. 贯穿全文须知这只是一个梦想 第四部分:任务型阅读(共 2016 小题;每小题 1 分,满分 2016 分) 请认真阅读下列短文,并根据所读内容在文章后表格中的空格里填入一个..最恰当的单词。 注意:请将答案写在答题卡上相应题号的横线上。每个空格只填 1 个单词。 For more than twenty years scientists have been seeking to understand the mystery of the‘‘sixth sense"of direction.By trying out ideas and solving problems one by one,they are now getting closer to one answer. One funny idea is that animals might have a built-in compass(指南针). Our earth itself is a big magnet(磁体).So a little magnetic needle that swings freely lines itself with the big earth magnet to point north and south . When people discovered that idea about athousand years ago and invented the compass,it allowed sailors to navigate (航海)on oceanvoyages, even under cloudy skies. Actuallly the idea of the living compass came just from observing animals in nature. Many birds migrate twice a year between their summer homes and winter homes.Some of them fly for thousands of kilometers and mostly at night.Experiments have shown that some birds can recognize star patterns.But they can keep on course even under cloudy skies.How can they do that? A common bird that does not migrate but is great at finding its way home is the homing pigeon.Not all pigeons can find their way home.Those that can are very good at it,and they have been widely studied.One interesting experiment was to attach little magnets to the birds’ heads to block their magnetic sense—just as a loud radio can keep you from hearing a call to dinner.On sunny days, that did not fool the pigeons.Evidently they can use the sun to tell which way they are going.But on cloudy days,the pigeons with magnets could not find their way.It was as if the magnets had blocked their magnetic sense. Similar experiments with the same kind of results were done with honeybees.These insects also seem to have a special sense ot direction. In spite of the experiments,the idea of an animal compass seemed pretty extraordinary.How would an animal get the magnetic stuff for a compass? An answer came from an unexpected source.A scientist was studying bacteria that live in the mud of ponds and marshes.He found accidentally little rod-like bacteria that all swam together in one direction—north. Further study showed that each little bacterium had a chain of dense particles inside,which proved magnetic.The bacteria had made themselves into little magnets that could line up with the earth’s magnet. The big news was that a living thing,even a simple bacterium,can make magnetite.That led to a search to see whether animals might have it. . By using a special instrument called magnetometer,scientists were able to find magnetite in bees and birds,and even in fish.In each animal,except for the bee.the magnetic stuff was always in or closer to the brain. Thus.the idea of a built—in animal compass began to seem reasonable. The Magnetic Sense — The Living Compass Passage outline Supporting details The existence of the earth magnet and the invention of the navigating compass ◇Our earth is a big magnet and a little freely (71) ▲ magnetic needle lines itself with the earth magnet to point north and south. ◇(72) ▲ on the idea above, the navigating compass was invented. The possibility of birds’ built-in compasses ◇ One piece of evidence is the (73) ▲ of many birds between their summer homes and winter homes. ◇ Birds can recognize star patterns on clear nights and keep on course (74) ▲ under cloudy skies The (75) ▲ on pigeons’ and bees’ built-in compasses ◇Little magnets were tied to the pigeons’ heads to (76) ▲ their magnetic sense. ◇The pigeons’ magnetic sense seemed to be affected on (77) ▲ days. ◇ Similar things with the same results were done with bees. The (78) ▲ of the magnetic stuff for the animal compass ◇ Little rod-like bacteria were found by chance to swim together in the direction of (79) ▲ . ◇Some animals had a chain of dense magnetic particles in or close to the (80) ▲ inside their bodies. 答案: 71. swinging 72 . Based 73. migration 74 . even 75. experiments/tests/ study/research 76. block 77. cloudy 78. discovery 79. north 80 . brain(s) (陕西) A Ask Dr ? Jeffers This month Dr. Jeffers is answering questions about the human brain and how it works. Dear Dr. Jeffers, One of my colleagues, Felix Moeller, told me that scientists are learning to use computer to ‘read minds’. Is there any truth to this story/ —Jane Leon, New York, USA Dear Ms. Leon, Well, a lot of research is being conducted in this area, but so far, the brain scanning equipment and corresponding computer programs haven’t been able to actually read thoughts. In one experiment, test subjects(受试者)were connected to scanning equipment and shown two numbers on a screen. They were then asked to choose between adding or subtracting(减)the two numbers. Using this method, researchers were able to follow brain processes and make the correct assumptions(假设)70 percent of the time. It’s not quite mind reading, but it’s certainly a first step. —Dr. J. Dear Dr. Jeffers, My three-year-old son loves it when I dig my fingers into his sides and tickle (胳肢)him until he laughs uncontrollably. The other day I noticed him trying to tickle himself but he couldn’t do it. Why not? —Glenn Lewis, Vancouver, Canada Dear Mr. Lewis, It’s because of how the brain works. The brain is trained to know what to pay attention to and what to ignore. It causes us to ignore physical feelings we expect to happen, but it causes a mild panic reaction when there is an unexpected feeling. For example, you don’t notice how your shoulder feels while you’re walking down the street. But if someone comes up behind you and touches you lightly on the shoulder, you may jump in fear. It’s that unexpected part that causes the tickle reaction. —Dr. J. 46. What can we learn from the answer to the first question? A. Some equipment is able to read human minds. B. Some progress has been made in mind reading. C. Test subjects have been used to make decisions. D. Computer programs can copy brain processes. 47. People laugh when tickled by others because the feeling is _______. A. unexpected B. expected C. comfortable D. uncomfortable 48. Who has got a little child according to the text? A. Ms. Leon B. Mr. Lewis C. Mr. Moeller D. Dr. Jeffers 49. According to the text, Jeffers is probably _______. A. a computer programmer B. a test subject C. a human brain expert D. a medical doctor 【文章导读】本文是一篇报刊新闻,大意是说明本月内心理专家 Jeffers 博士将做客本报心理 咨询栏目,回答读者提出的各种问题。 46. B. 【解析】推理判断题。根据第一个问题的应答的最后一句可以推断出本题选 B。 47. A. 【解析】推理判断题。由问题二的应答句中 but it causes a mild panic reaction when there is an unexpected feeling. 可知本题选 A。 48. B. 【解析】推理判断题,根据问题二可知本题选 B。 49. C. 【解析】推理判断题。根据本报道的栏目一可知本题选 C。 B Brave Frenchman Found Half-way Around the World (NEW YORK) A French tourist highly praised for rescuing a two-year-old girl in Manhattan said he didn’t think twice before diving into the freezing East River. Tuesday’s Daily News said 29-year who left the spot quickly after the rescue last Saturday. He lifted the little girl out of the water after she fell off the bank at the South Street Scaport museum. He handed the girl to her father, David Anderson, who had dive in after him. “I didn’t think at all,” Duret told the Daily News. “It happened very fast. I reacted very fast. ” Duret, an engineer on vacation ,was walking with his girlfriend along the pier(码头)when he saw something falling into the water . He thought it was a doll, but realized it was a child when he approached the river. In an instant ,he took off his coat and jumped into the water. When he reached the girl, she appeared lifeless, he said . Fortunately, when she was out of the water, she opened her eyes. Anderson said his daughter slipped off the bank when he was adjusting his camera. An ambulance came later for her, said Duret, who was handed dry clothes from cookers. Duret caught a train with his girlfriend shortly after. The rescue happened on the day before he left for France. Duret said he didn’t realize his tale of heroism he was leaving the next morning . “I don’t really think I’m a hero,” said Duret. “Anyone would do the same ting. ” 50. Why was Duret in New York? A. To meet his girlfriend B. To work as an engineer C. To spend his holiday D. To visit the Andersons. 51. What did Duret do shortly after the ambulance came? A. He was interviewed by a newspaper B. He asked his girlfriend for his dry clothes C. He went to the hospital in the ambulance D. He disappeared from the spot quickly 52. Who divide after Duret into the river to save the little girl? A. David Anderson B. A passer-by C. His girlfriend D. a taxi driver. 53. When was duet most probably found to be the very hero? A. The day when he was leaving for home. B. A couple of days after the girl was rescued C. The first day when he was in New York D. The same day when he was interviewed. 【文章导读】本文是一篇新闻报道,大意是说明一位法国游客从冰冷的河水里救了一位落水 儿童,他的这一勇敢行动感动整个纽约市。 50. C. 【解析】推理判断题。根据第五自然段的首句可知本题选 C。 51. D. 【解析】细节理解题。根据第二自然段的最后一句可知本题选 D。 52. A. 【解析】细节理解题。由第三自然段的首句可知本题选 A。 53. B. 【解析】推理判断题。根据文章倒数第二自然段可以推断出本题选 B。 C The 1900 house The bowler family was one of more than 400 families who applied to 1900 house, a reality TV shout which took a typical family back a hundred years to se how people lived in the days before the internet, computer games and even electricity. The bowler family spent three months in a London home without a telephone, computers, TV, or fast food. The bowlers wore clothes from 1900, are only food available in English at that time, and cooked their meals on a single stove. Paul bowler still went to work every day in a then uniform. The children changed their clothes on the way to and from school and their classmates didn’t know about then unusual home life. Joyce stayed at home, cooking and cleaning like a typical housewife of the time, though everything took three times as long. So does Joyce think that people’s lives were better in the old days? “I think people in the old days had just ad many troubles and worries,” Joyce said. And I don’t think their life was better or worse, there were lots of things back then that I’m happy I don’t have to deal with nowadays, but on the other hand life was simpler. ” “We had a lot more time with our family, and it was hard being nice to each other all the time,” eleven-year-old Hilary said. So what did the Bowler family miss most about modern life while living in the 1900 house? Paul, 39:” telephone and a hot shower” Joyce, 44:” a quick cup of tea from a kettle you could just turn on” Hilary, 11:” rock CD” Joseph, 9:” hamburger and computer games” 54. While the Bowler family was living in 1900 house, _____. A the mother spent more time on housework B the two children wore the then clothes for school C they prepared their meals together on a stove D they ate simple foods they had never seen 55. According to Paragraph 4, what’s Joyce’s opinion about life in 1900? A There were fewer problems for the family B Life was simpler but worse than it is now C There were things she liked and disliked D The family had more time to stay together 56. What would Hilary expect most from modern life in the three months? A To play computer games B. To make phone calls C To listen to music D. To chat on the Internet 【文章导读】在英国,Bowler 一家有幸成为 400 户体验 20 世纪初期人们日常生活的家庭之一, 在这项体验活动中 Bowler 住进那个时期的房子,没有电话,电脑,电视,快餐身穿那个时代 的服饰,通过这为期三个月的体验活动,他们一家人对于现在的生活有了更深的理解。 54. A. 【解析】推理判断题。根据第二自然段的最后一句可知本题选 A。 55. C. 【解析】推理判断题。在本自然段中 Joyce 谈到了她对当时生活的感受,有褒有贬,由 此可知本题选 C。 56. C. 【解析】推理判断题。根据 Hilary,11:rock CD 一句可以推断出她想听音乐,故本题 选 C。 D Stop Spam When I first got an e-mail account ten years ago, I received communications only from family, friends, and colleagues. Now it seems that every time I check my e-mail, I have an endless series of advertisements and other correspondence that do not interest me at all. If we want e-mail to continue to be useful, we need specific laws that make spamming(发送垃圾邮件) a crime. If lawmakers do not do something soon to prohibit spam, the problem will certainly get much worse. Computer programs allow spammers to send hundreds of millions of e-mails almost instantly. As more and more advertisers turn to spam to sell their products, individual(个人的) e-mail boxes are often flooded with spam e-mails. Would people continue to use e-mail if they had to deal with an annoying amount of spam each time? This problem is troubling for individuals and companies as well. Many spam e-mails contain computer viruses that can shut down the entire network of a company. Companies rely on e-mail for their employees to communicate with each other. Spam frequently causes failures in their local communications networks, and their employees are thus unable to communicate effectively. Such a situation results in a loss of productivity and requires companies to repeatedly repair their networks. These computer problems raise production costs of companies, which are, in the end, passes on to the consumer. For these reasons, I believe that lawmakers need to legislate (立法) against spam. Spammers should be fined, and perhaps sent to prison if they continue to disturb people. E-mail is a tool which helps people all over the world to communicate conveniently, but spam is destroying this convenience. 57. What does the underlined word “correspondence” in the Paragraph 1 probably mean? A. messages B. ideas C. connections D. programs 58. According to the text, what is the major cause of the flooding spam? A. Companies rely on e-mail for communications. B. More people in the world communicate by e-mail. C. Many computer viruses contain spam e-mail. D. More advertisers begin to promote sales through spam. 59. According to Paragraph 3, who is the final victim of spam? A. The business B. The advertiser C. The employee D. The consumer 60. What is the purpose of the text? A. To inform B. To educate C. To persuade D. To instruct 【文章导读】随着网络信息技术的发展,越来越多的人们实用网络邮箱来传递信息,但现在 垃圾邮件的泛滥已经成为网络用户们深恶痛绝的事情,人们呼吁立法来阻止垃圾邮件的泛滥。 57. A. 【解析】词义猜测题。根据本文中的 e-mail 一词以及 endless series of advertisements 可 以推断出 correspondence 在此处意思是:信件,垃圾邮件,由此可知本题选 A。 58. D. 【解析】推理判断题。由第二自然段中的 As more and more advertisers turn to spam to sell their products, individual e-mail boxes are often flooded with spam e-mails. 一句可知本题选 D。 59. D. 【解析】推理判断题。根据本段的最后一句可知本题选 D。 60. C. 【解析】推理判断题。通读全文可知本文作者摆出了垃圾邮件的危害,其目的是说服立 法人员尽快出台法律阻止垃圾邮件的蔓延,故本题选 C。 第二节(共 5 小题,每小题 2 分,满分 2016 分) 根据短文内容,从下框的 A-F 选项中选出能概括每一段主题的最佳选项。选项中有一项 为多余项。 A Gift giving proven to be valuable. B. Memories from gift giving C. Moments and events for gift giving D. Various functions of gift giving E. Gift giving as a wasteful practice F. Gift giving as a two-way social activity Gift Giving 61. C There are many occasions(场合) for giving gifts in modern industrialized societies; birthdays, naming ceremonies, weddings, anniversaries, New Year, It is common to give gifts on many of these celebrations in western cultures. In addition, special events, such as one’s first day of school or graduation from university, often require gift fiving. 62. F What is happening when we give gifts? Most important, we are exchanging gifts. If someone gives me a gift for my birthday, I know that I am usually expected to give one on his or her next birthday. A gift builds up or confirms a social obligation(义务). 63. D Gifts tighten personal relationships and provide a means of communication between loved ones. People say that a gift lets the recipient(接受者) know we are thinking of them, and that we want to make the person “feel special. ” We want people to feel wanted, to feel part of our social or family group. We give presents to say “I’m sorry. ” Sometimes it is difficult for us to find a present that someone will like. Sometimes we give things that we like or would frrl comfortable with. In all these cases, the gifts are sending out messages-often very expressive ones. 64. B People tend to talk about presents in a fairly loving way. A woman whose mother had died years ago described the many gifts around her house. These were gifts that her mother had given her over the years: “I appreciate these, and they mean something to me,” the woman said, “because I remember the occasions they were given on, and that they were from my mother, and the relationship we’ve had. ” The gifts remain and keep the relationship alive in mind. This woman felt the same way about the gifts she gave to others. She hoped that the recipients would look at here gifts in years to come and 65. A Emotions(情感)like these suggest that a positive spirit still lies behind gift giving. They prove that the anthropologist Claude Levi-Strauss was wrong to say that modern western gift giving is highly wasteful. Studies in Canada and elsewhere have also shown that this is not the case. Each gift is unique even if so many are given. The emotional benefit for those who exchange gifts is the very reason for the tradition to continue. 61. C. 【解析】根据本段首句中的 occasions 和最后一句中的 special events 可知本段大意选 C。 62. F. 【解析】由本段的最后一句可知推断出本段大意选 F。 63. D. 【解析】根据本自然段的主题句——最后一句可知本题选 D。 64. B. 【解析】根据本段首句可以推断出本段大意是说明礼物可以给予人们美好的会议,故选 B。 65. A. 【解析】有本段末句可以推断出本题大意选 A。 书面表达 【写作指导】本题要求是应用文写作,材料比较详细,信息量大,这对于考生发挥很有好处, 考生需要在写作之前认真审题,弄清材料的具体要求,命题的意图和写作的目的,认真组织 语言,并注意行文的逻辑性,连贯性和书写的整齐,以及卷面的整洁度。 (2016 全国Ⅱ) 第一节 阅读下列短文,从每题所给的四个选项(A、B、C 和 D)中,选出最佳选项,并 在答题卡上将该项涂黑。 【总体评析】纵观五篇阅读,可以说是这套试卷比较容易的部分,相信多数考生这部分会有 一份得心应手的轻松心情。与往年比较而言,总体难度低于去年。失分率会小,学生做得相 对轻松。词汇量不大,语篇不长。A 篇回忆宠物狗;B 篇度假也可谋生;C 篇北京滑雪热潮; D 篇世界上最凉爽的宾馆信息查找题;E 篇文字的起源和运用。 A When I was six, Dad brought home a dog one day, who was called “Brownie”. My brothers and I all loved Brownie and did different things with her. Ore of us would walk her, another would feed her, then there were baths, playing catch and many other games, Brownie, in return. loved each and every one of us. One thing that most touched my heart was that she would go to whoever was sick and just be with them we always felt better when she was around. One day, as I was getting her food, she chewed up(咬破)one of Dad’s shoes, which had to be thrown away in the end. I knew Dad would be mad and I had to let her know what she did was wrong. When I looked at her and said, ”Bad girl,” she looked down at the ground and then went and hid. I saw a tear in her eyes. Brownie turned out to be more than just our family pet, she went everywhere with us . People would stop and ask if they could pet her. Of course she’d let anyone pet her. She was just the most lovable dog. There were many times when we’d be out walking and a small child would come over and pull pm her hair. she never barked(吠) or tried to get away. Funny thing is she would smile. This frightened people because they thought she was showing her teeth. Far from the truth, she lovely everyone. Now many years have passed since Brownie died of old age. I still miss days when she was with us. 41. What would Brownie do when someone was ill in the family? A. Look at them sadly. B. Keep them company. C. Play games with them. D. Touch them gently. 42. We can infer from Paragraph 2 that Brownie__________. A. world eat anything when hungry B. felt sorry for her mistake C. loved playing hide-and-seek D. disliked the author’s dad 43. Why does the author say that Brownie was more than just a family pet? A. She was treated as a member of the family. B. She played games with anyone she liked. C. She was loved by everybody she met. D. She went everywhere with the family. 44. Some people got frightened by Brownie when she__________. A. smiled B. barked C. rushed to them D. tried to be funny 45. Which of the following best describes Brownie? A. Shy B. Polite C. Brave D. Caring 解析: 这是一篇大家比较熟悉的一个话题:“宠物”。这篇文章文字简单,情节具有生活 化的特点。就总体而言,考生能理解和掌握它的大意:说明了宠物惹人喜欢、可爱。 从选材和文字特点方面可以看出,这篇文章通俗易懂,不“偏”不“怪”。如果感觉完形难度 偏大的学生在做这道试题时,会相对轻松。 41. 答案:B 考点:分析:理解文章具体信息; 解析: 从第一段“she would go to whoever was sick and just be with them”暗示出它的表现 方式,选择 B,keep them company(陪伴他们) 42. 答案:B 考点:分析;理解文中的具体信息 解析: 从第二段结尾句“she looked down at the ground and then went and hid. I saw a tear in her eyes”可以推断出它为做错的事感到悲伤 43. 答案:C 考点:分析:考查学生的判断和推理能力。 解析: 通过全文的理解,和倒数第二段的提示 “There were many times when…. ”总有很多 时候,可以推断出它为人们所喜爱 44. 答案:A 考点:分析:理解文中具体信息,情节推断 解析: 文中 “Funny thing is she would smile. This frightened people because they thought she was showing her teeth. ”是解释,有趣的是:“露出牙齿”吓到了大家,其实它是在“笑” 45. 答案:D 考点:分析:考查学生的概括和判断能力。 解析: 上下文的描写突出 love each and everyone,第三段中 She was just the most lovable dog 都做出了暗示。 B When you’re lying on the white sands of the Mexican Riviera, the stresses(压力) of the world seem a million miles away. Hey, stop! This is no vacation-you have to finish something! Here lies the problem for travel writer and tood critic(评论家),Edie Jarolim “I always loved traveling and always liked to eat, but it never occurred to me that I could make money doing both of those things,” Jarolim said. Now you can read her travel advice everywhere in Arts and Antiques, in Brides, or in one of her there books, The Complate Idiot’s Travel Guide to Mexican Beach Resarts. ……writing began some eight years ago. After getting a PhD in English in Canada, she took a Test Frommer’s travel guides, passed it, and got the job. After working at Frommer’s, Jarolim workedfor a while at Rough Guides in London, then Fodor’s, where she fell so in love with a description of the Southwest of the U. S. that she moved there. Now as a travel writer, she spends one-third of her year on the road. The rest of the time is spent completing her tasks and writing reviews of restaurants at home in Tucson, Arigona. As adventurous as the job sounds, the hard part is fact-checking all the information. Sure, it’s great to write about a tourist attraction, but you’d better get the local(当地的)museum hours correct or you could really ruin someone’s vacation. 46. Which country does Jarolim live in now? A.Mexico B.The U. S. C.The U. K. D.Canada 47. What is most difficrlt for Jarolim? A.Working in different places to collect information B.Checking all the facts to be written in the guides C.Finishing her work as soon as possible D.Passing a test to write travel guides 48. What do we know about Jarllim from the text? A.She is successful in her job B.She finds her life full of stresses C.She spends half of her time traveling D.She is especially interested in museums 49. What would be the best title for the text? A.Adventures in Travel Writing B.Working as a Food Critic C.Travel Guides on the Market D.Vacationing for a Living B 篇阅读理解的文字风格略显诙谐,Hi, stop! 但有实际的描述了一个很普遍的谋生问题: 谋生在 Jarolim 身上既是度假休闲,又是写作和评论的源泉。这是一篇展示生活的文章。 从文字设计上看,影响考生理解完整意思的因素可能是专有名词和对话的插入,但是如 果避开障碍,处理好这些内容,整篇文章文字比较通顺流畅。难度并不大。但是在 47、48 题 的理解上,部分考生会出现问题。 46. 答案:B 考点::考查细节 解析:文章第三段提示 “then Fodor’s, where she fell so in love with a description of the Southwest of the U. S. that she moved there. ” 47. 答案:B 考点::理解文中具体信息 解析:文章最后一段第一句 “As adventurous as the job sounds, the hard part is fact-checking all the information. ” 提示 48. 答案:A 考点::作出简单判断和推理 解析:根据全文的介绍,说明这是一种成功的生活方式 49. 答案:D 考点::考主旨大意 解析:提示在文章第一段结尾句 “Hey, stop! This is no vacation-you have to finish something!”和 第二段第二句 “but it never occurred to me that I could make money doing both of those things,”说 明了没有想到“度假也可以谋生”的欣喜之情。 C Thousands of people living in the Chinese eapital will celebrate the start of the Chinese New Year by heading for the ski resorts(滑雪场). Never mind that Beijing’s dry weather seldom produces now. It is cold enough in winter for snow-making machines to make a covering for the hills north to the capital. And the rapid growth of a pleasure-seeking middle class has formed the basis for this New craze(热潮). Since Beijing’s first ski resort was opened ten years ago, the sport has enjoyed an astonishing increase. There are now more than a dozen resorts. Clothes markets in the city have added bright colored ski suits to their winter collections. Mr. Wei, a manager of a newly-opened ski resort in Beijing, sees the growth of an industry that could soon lead Chinese to head for the ski resorts of Europe. In recent years ski resorts offering natural snow have opened in China. But many are in faraway areas of the country and can’t really match the equipment and services of some ski resorts in Europe. Beijing’s sking craze is partly a result of the recent increase in private(私有的)cars. This has led to the growth of a leisure industry in the capital’s suburbs(郊区). which until the late-1990s were unreachable to ordinary people. According to Mr. Wei,About 40% of the visitors to his resort some in their own cars. The rest are bused in by schools, businesses or government offices. The problem is making money. Starting ski resorturequires quite a lot of money:hiring land from the local government,preparing the hills,buying snow machines,making sure there are enough water and electricity to run them,and buying ski equipment for hiring out to customers. The ski resort where Mr. Wei works cost nearly $4m to set up. And,as so often in China when someone comes up with a good idea,many others rush in and price wars break out. Beijing now offers some of the cheapest ski training classes in the world,though with most people rather new to the sport, expecting a few more doing the same job. 50. What does this text mainly talk about? A.Convenience for skiers brought about by private cars B.Skiing as a new way of enjoying one’s spare time C.Things to be considered when starting a ski resort D.A sudden increase of ski training classes in Beijing 51. Why are some Chinese likely to go skiing in Burope? A.To visit more ski areas B.To ski on natrual snow C.For a large collection of ski suits D.For better services and equipment 52. The underlined words”leisure industry”in Paragraph 3 refer to A.transport to ski resorts B.production of familycars C.business of providing spare time enjoyments D.part-time work for people living in the suburbs 53. What is the main problem in running a ski resort? A.Difficulty in hiring land B.Lack of business experience C.……ski resorts. D.Shortage of water and electricity C 篇阅读理解属于一篇社会新闻的报道。报道北京首都的庆祝新年的活动-----滑雪。介绍 了滑雪胜地的一些情况。这篇文章看似新闻报道,但是与百姓生活息息相关,文字简单,偶 尔有构词法,语篇略长。但是不妨碍绝大多数考生对整篇文章的完全清楚地理解。尤其是试 题选项均可以在原文中找到直接句子即该题答案。52、53 小题,容易理解出错。 50. 答案:B 考点::考查主旨大意 解析:注意选项的四个提示性的中心词 A. Convenience B. Skiing C. Things to be considered D. A suddent increase of 确定文章核心主旨是关于 skiing 故选择 B 51 答案:C 考点::通过细节信息,理解意图和态度 解 析 : 文 中 提 到 “But many are in faraway areas of the country and can’t really match the equipment and services of some ski resorts in Europe. ” 其中 match 正是对比之意。说明在设备和 服务反面,远远无法与欧洲的滑雪胜地相媲美 52. 答案:C 考点::词义推测 解析:承接上文 “the rapid growth of a pleasure-seeking middle class…”;下文 “the sport has enjoyed an astonishing increase”可知。第 50 题的选项 B “…a new way of enjoying one’s spare time”中也暗示了这是一个提供娱乐的行业 53 答案:C 考点::根据细节信息,做出简单判断和推理。 解析:文中 “And,as so often in China when someone comes up with a good idea,many others rush in and price wars break out”作了提示。中国的跟风现象,导致价格大战 D Coolest Hotels in the World Ariau Amazon Towers The Ariau Amazon Towers hotel lets you sleep in a tree house. Eight towers make up this hotel that offers over 300 rooms. If you really want to get into the spirit, book the Tarzan suit which is large enough for a big family. You’ll be thirty feet up in the air and can travel between the towers through their wooden walkways. Prices, starting at $300 one night for each person for a regular room and going all the way up to$3000 for the Tarzan Suite. For more information, visit the website:http://www. ariautowers. com The Ice Hotel Every winter in Jukkasjarvi, Sweden kind of hotel called the Ice Hotel is built. Each year,world-famous artists are invited to design and produce works of art from the ice, many of which can be found in the rooms. You’ll have your choice between hot or cold romms,but you will be well advised to stay at least one night in a cold room for a true experience. Prices: starting at $318 one night for each person for either a cold room or a warm one. For more information, visit the website:http://www. icehotel. com Propeller Island Propeller Island City Lodge is a very special hotel that was designed by a German tist. Each room provides you with the promibility of living in a work of art. Every single piece of furmiture in the thirty rooms of the hotel has been hand-made and each room is completely different. You’ll be able to choose a room based on your own personal tastes. Prices:starting at just $91 a night,and an additional person for only 20 extra dollars. For more information , visit the website:http://www. propeller island. com For information about other cool hotels in the world ,visit the website: http://www. bahamabeachclub. com 54. What is special about the Ariau Amazon Towers hotel? A.You can sleep in the houses B.You can choose any of the towers C.It is designed for big families D.Every room has a walkway 55. For two persons spending a night in one of these hotels,they have to pay at least A.$111 B.$182 C.$600 D.$636 56. Which website should you visit if you want to find out whether there exists a hotel the sea? A.http://www. icehotel. com B.http://www. ariautowers. com C.http://www . propeller-island. com D.http://www. bahamabeachclub. com 57. Which hotel would invite artists to come to work every year? A.Propeller Island City Lodge B.Ariau Amazon Towers C.The Ice Hotel D.Bahama Beach Club D 篇阅读理解是一篇旅游手册特点的文章,侧重考查信息查找。描写了 Coolest Hotels in the World,介绍了各自的特点、价格和信息咨询方式,更加突出了该篇的旅游手册的特点。 该篇文章文字也比较容易,试题设计迷惑度不大。直接可以查找到答案。 54. 答案:A 考点::理解文中具体信息 解析:从 “The Ariau Amazon Towers hotel lets you sleep in a tree house. ”可以得出答案 55. 答案:A 考点::理解文中具体信息 解 析: 价钱 最少 的是 “starting at just $91 a night,and an additional person for only 20 extra dollars. ”作出了提示 56. 答案:D 考点::理解文中具体信息 解析:文章最后一节有提示 57. 答案:C 考点::理解文中具体信息 解析:在 “The Ice Hotel” 中 “Each year,world-famous artists are invited to design and produce works of art from the ice…”的提示 E How words came into being is unknown. All we assume is that some early men invented cortain sounds,in one way or another,to express thoughts and feelings,actions and things,so that they could talk with each other. Later they agreed upon certain signs,called letters,which could be put together to show those sounds,and which could be written down. Those sounds,whether spoken or written in letters,are called words. The power of words,then,lies in their associations-the things they bring up to our minds. Words become filled with meaning for us by experience;and the longer we live,the more certain words bring back to us the happy and sad events of our past;and the more we read and learn,the more the number of words that mean something to us increases. Great writers are those who not only have great thoughts but also express these thoughts in words which have powerful effects on our minds and feelings. This clever use of words is what we call literary style. Above all,the real poet is a master of words. He can express his meaning in words which sing like music,and which by their position and association can move men to tears. We should therefore learn to choose our words carefully and use then correctly,or they will make our speech silly and common. 58. We learn from the text that language might have begun with( ) A.expressions B.actions C.signs D.sounds 59. What is mainly discussed in Faragraph 2? A.The ……of new words B.The importance of old words C.The relation of human experience with words D.The gradual change and development of words 60. In the last paragraph,what does the anthor suggest that we should do? A.Use words skilfully B.Make musical speeches C.Learn poems by heart D.Associate with listeners E 篇阅读理解是一篇说明文,阐述文字的起源和运用。给我们呈现了文字从起源到现在对 我们生活、思想和智慧发展的影响。建议巧妙运动和注意遣词,促进文字的发展。 58. 答案:D 考点::作出简单判断和推理 解析:文中第一段的第二句 “All we assume is that some early men invented cortain sounds,in one way or another…”作出了提示。说明 sounds 很可能是最初的语言雏形 59. 答案:C 考点::段落主旨概括 解析:第二段首句 “The power of words,then,lies in their associations-the things they bring up to our minds. ”The relation 等同于文中的 associations 60. 答案:A 考点::理解作者的意图和态度,理解主旨要义 “We should therefore learn to choose our words carefully and use then correctly,or they will make our speech silly and common. 作者所举例证及 用词的感情色彩可知,作者鼓励我们用词的技巧,如同文学作品,诗文等一样,要注重遣词。 第二节 根据对话内容,从对话后的选项中选出能填入空白处的最佳选项,并在答题卡 上将该项涂黑。选项中有两项为多余选项。 W:Hi ,John M: Hi,Lucy W: Yes,I’m required to tell a story in English before the lesson begins. M: Oh,I see W: You’re right. I need to practice on the stage before they all come M: W: Yes,here,in my pocket……Oops,where is it? M: Don’t worry W: No,not in the bag,either. I’m afraid . I left it at home. M: W: Aha!It’s just the key to the classroom!Thank you so much. A.Glad to see you B.What’s the story about? C.What’s that around your neck? D.Maybe you put it in your schoolbag E.Do you have the key to the classroom F.You’re goying to school rather early today G.So you want to get to the classroom earlier to make some preparation? 【解析】该题难易度小,与往年持平,难度不大。 61.F.由下句又 before lesson begins,故知对方“早走”rather early 更合理。 62.G.承接上句的回答,make some preparation。 63.E.根据最后可知 the key to the classroom,in my… where is it?及 yes 回答。影视对方就 钥匙提出的问题。 64.D.由下句对 No, not in the bag, either. 作了进一步的解释可知是暗示是否在书包 schoolbag 里。 65.C.后文 Aha!It’s just the key to the classroom!表达了找到钥匙的欣喜之情可知脖子上挂的 就是班级的钥匙。 (2016 湖北) A It was a Sunday and the heavy storm had lasted all night. The morning after the storm, though, was beautiful: blue skies, warm air and a calm, inviting sea touching the shore gently. My father realized it was a good day for fishing and invited my sister and me to go with him. I was only 14 and fishing had never been my thing, but I decided to go all the same. I’m so glad I did. On the road to the harbour we could see the terrible destruction on the coast, but the harbour itself was in fairly good shape. After all, it was protected by the arms of a bay that had only one tiny channel to the sea. As we got on board, we noticed two big humps(脊背) in the distance. On approaching them, we saw it was a mother whale with her baby. We couldn’t believe it — —there aren’t any whales along the coast here. The storm must have driven them across the ocean into the bay, in which the still water was so badly polluted that nothing could survive. The little baby whale——actually as big as our boat——was obviously stuck and could not move. The mother dived under the water and came up suddenly, making big whirlpools(漩涡) and waves. ”She’s trying to help her baby, but on the wrong side, ”my father said. At this point, my father moved our boat in a semicircle to the other side and, heading the boat towards the baby whale, pushed it gently. With our several gentle pushes the big hump turned over and disappeared under water. Then it swam up right beside its mum. They struggled in their desperate attempts to escape but missed the exit and started heading in the wrong direction. We hurried up to the whales and tried to lead them towards the bay channel. Slowly, they let us lead them, some-times rising from the water right beside us to breathe——and to give us a trusting look with those huge eyes. Once they hit their first part of clean water flowing straight from the sea, the mum gave us a wave with her tail and off they swam into the distance. In the excitement it had felt like only a few minutes, but we had been with those wonderful animals for almost an hour and a half. That was the simple and lasting beauty of the day, Nearly four decades later, I still look back fondly to that golden day at sea. 51. The author says “I’m so glad I did. ”(in Para. 2)because __________. A. he witnessed the whole process of fishing B. he enjoyed the beauty of the calm sea C. he experienced the rescue of the whales D. he spent the weekend with his family 52. The harbour survived the storm owing to____________. A. the shape of the harbour B. the arms of the bay C. the still water in the channel D. the long coast line 53. The mother whale failed to help her baby because__________. A. she had stayed in the polluted water for too long B. the whirlpools she had made were not big enough C. she had no other whales around to turn to for help D. the waves pushed her baby in the wrong direction 54. what is the theme of the story? A. Saving lives brings people a sense of happiness B. Fishing provides excitement for children C. It’s necessary to live in harmony with animals D. It’s vital to protect the environment 51. 答案:C 考点::考查学生的推理能力 解析: “I’m so glad I did”是个承上启下的句子,为后文做了铺垫。只要读懂后文所讲的内容, 简单概括一下,不难得出答案 52. 答案:B 考点::细节理解 解析:从 “it was protected by the arms of a bay that had only one tiny channel to the sea. As we got on board, we noticed two big humps(脊背) in the distance. ”可以得出答案 53. 答案:D 考点::细节理解 解析:从 “She’s trying to help her baby, but on the wrong side, ” 可以得出答案 54. 答案:A 考点::考查主旨大意 解析:从文章最后一段可以概括出来 B For many parents, raising a teenager is like fighting a long war, but years go by without any clear winner. Like a border conflict between neighboring countries, the parent-teen war is about boundaries: Where is the line between what I control and what you do? Both sides want peace, but neither feels it has any power to stop the conflict. In part, this is because neither is willing to admit any responsibility for starting it. From the parents’ point of view, the only cause of their fight is their adolescents’ complete unreasonableness. And of course. the teens see it in exactly the same way, except oppositely. Both feel trapped In this article. I’ll describe three no-win situations that commonly arise between teens and parents and then suggest some ways out of the trap. The first no-win situation is quarrels over unimportant things. Examples include the color of the teen’s hair, the cleanliness of the bedroom, the preferred style of clothing, the child’s failure to eat a good breakfast before school, or his tendency to sleep until noon on the weekends. Second, blaming. The goal of a blaming battle is to make the other admit that his bad attitude is the reason why everything goes wrong. Third, needing to be right, It doesn’t matter what the topic is –politics. The taws of physics, or the proper way to break an egg –the point of these arguments is to prove that you are right and the other person is wrong. for both wish to be considered an authority—someone who actually knows something — and therefore to command respect. Unfortunately, as long as long as parents and teens continue to assume that they know more than the other, they’ll continue to fight these battles forever and never make any real progress 55. Why does the author compare the parent-teen war to a border conflict? A. Both can continue for generations. B. Both are about where to draw the line C. Neither has any clear winner D. Neither can be put to an end 56. What does the underlined part in Paragraph 2 mean? A. The teens blame their parents for starting the conflict. B. The teens agree with their parents on the cause of the conflict C. The teens cause their parents of misleading them D. The teens tend to have a full understanding of their parents 57. Parents and teens want to be right because they want to ________. A. give orders to the other B. know more than the other C. gain respect from the other D. get the other to behave properly 58. What will the author most probably discuss in the paragraph that follows? A. Causes for the parent –teen conflicts B. Examples of the parent –teen war. C. Solutions for the parent –teen problems D. Future of the parent-teen relationship 55. 答案:B 考点::细节理解 解 析 : 从 “Like a border conflict between neighboring countries, the parent-teen war is about boundaries: Where is the line between what I control and what you do?”可以得出答案 56. 答案:A 考点::细节理解 解析:从 “In part, this is because neither is willing to admit any responsibility for starting it. From the parents’ point of view, the only cause of their fight is their adolescents’ complete unreasonableness. ”和 “except oppositely”可以得出答案 57. 答案:C 考点::细节理解 解析:题干的关键词是 want to be right because,回原文定位到最后一个自然段的“Third, needing to be right”. 原文接着往下看“It doesn’t matter what the topic is – politics, the laws of physics, or the proper way to break an egg - the point of these arguments is to prove that you are right and the other person is wrong, for both wish to be considered an authority – someone who actually knows something – and therefore to command respect. ” 这句话虽然看起来很复杂,但在新概 念的课堂上老师们反复强调阅读长难句时,如果遇到插入语可以跳过不看,这里破折号中间 的内容均为插入语,将其省略,句子就会变得简单的多。又由于题干是对原因的提问,这里 我们只用重点看 for 后面的内容,“for both wish to be considered an authority and therefore to command respect”,正确答案为 C。 58. 答案:C 考点::考查学生的概括能力 解析:原文第三段说 “In this article. I’ll describe three no-win situations that commonly arise between teens and parents and then suggest some ways out of the trap. ”清楚地说明了文章的结构, 先分类阐述家长与孩子之间矛盾的三种情况,然后给出解决方法,因此答案为 C C They wear the latest fashions with the most up-to-date accessories (配饰). Yet these are girls in their teens or twenties but women in their sixties and seventies. A generation which would once only wear old-fashioned clothes is now favoring the same high street looks worn by those half their age. Professor Julia Twigg, a social policy expert, said, “Women over 75 are now shopping for clothes more frequently than they did when they were young in the 1960s. In the 1960s buying a coat for a woman was a serious matter. It was an expensive item that they would purchase only every three or four years — now you can pick one up at the supermarket whenever you wish to. Fashion is a lot cheaper and people get tired of things more quickly. ” Professor Twigg analyzed family expending(支出)data and found that while the percentage of spending on clothes and shoes by women had stayed around the same — and 5 or 6 per cent of spending—the amount of clothes bought had risen sharply. The professor said,“Clothes are now 70 per cent cheaper than they were in the 1960s because of the huge expansion of production in the Far East. In the 1960s Leeds was the heart of the British fashion industry and that was where most of the clothes came from, but now almost all of our clothes are sourced elsewhere. Everyone is buying more clothes but in general we are not spending more money on them. ” Fashion designer Angela Barnard, who runs her own fashion business in London, said older women were much more affected by celebrity(名流) style than in previous years. She said, “When people see stars such as Judi Dench and Helen Mirren looking attractive and fashionable in their sixties, they want to follow them. Older women are much more aware of celebrities. There’s also the boom in TV programmes showing people how they can change their look, and many of my older customers do yoga to stay in shape well in their fifties. When I started my business a few years ago, my older customers tended to be very rich, but now they are what I would call ordinary women. My own mother is 61 and she wears the latest fashions in a way she would never have done ten years ago. ” 59. Professor Twigg found that, compared with the 1960s, _______. A. the price of clothes has generally fallen by 70% B. the spending on clothes has increased by 5% or 6% C. people spend 30% less than they did on clothes D. the amount of clothes bought has risen by 5% or 6% 60. What can we learn about old women in terms of fashion? A. They are often ignored by fashion designers. B. They are now more easily influenced by stars. C. They are regarded as pioneers in the latest fashion. D. They are more interested in clothes because of their old age. 61. It can be concluded that old women tend to wear the latest fashions today mainly because A. they get tired of things more quickly B. TV shows teach them how to change their look C. they are in much better shape now D. clothes are much cheaper than before 62. Which is the best possible title of the passage? A. Age Is No Barrier for Fashion Fans B. The More Fashionable, the Less Expensive C. Unexpected Changes in Fashion D. Boom of the British Fashion Industry 59. 答案:A 考点::细节理解 解析:从 “The professor said, “Clothes are now 70 per cent cheaper than they were in the 1960s because of the huge expansion of production in the Far East. In the 1960s”可以得出答案 60. 答案:B 考点::细节理解 解析:从 “When people see stars such as Judi Dench and Helen Mirren looking attractive and fashionable in their sixties, they want to follow them. Older women are much more aware of celebrities. ”可以推断出答案 61. 答案:D 62. 答案:A 考点::考查主旨大意 D This brief book is aimed at high school students, but speaks to anyone learning at any stage of life. Its formal, serious style closely matches its content, a school-masterly book on schooling. The author, W. H. Armstrong, starts with the basics: reading and writing. In his opinion, reading doesn’t just mean recognizing each word on the page; it means taking in the information, digesting it and incorporating it into oneself just as one digests a sandwich and makes it a part of him. The goal is to bring the information back to life, not just to treat it as dead facts on paper from dead trees. Reading and writing cannot be completely separated from each other; in fact, the aim of reading is to express the information you have got from the text. I’ve seen it again and again : some-one who can’t express an idea after reading a text is just as ineffective as someone who hasn’t read it at all. Only a third of the book remains after that discussion, which Armstrong devotes to specific tips for studying languages, math, science and history. He generally handles these topics thoroughly(透 彻地) and equally, except for some weakness in the science and math sections and a bit too much passion(激 情) regarding history to his students, that was a hundred times more than my history teachers ever got across. To my disappointment, in this part of the book he ignores the arts. As a matter of fact, they demand all the concentration and study that math and science do, though the study differs slightly in kind. Although it’s commonly believed that the arts can only be naturally acquired, actually, learning the arts is no more natural than learning French or mathematics. My other comment is that the text aged. The first edition apparently dates to the 1960s—none of the references(参考文献)seem newer than the late 1950s. As a result, the discussion misses the entire computer age. These are small points, though, and don’t affect the main discussion. I recommend it to any student and any teacher, including the self-taught student. 63. According to Armstrong, the goal of reading is to________. A. gain knowledge and expand one’s view B. understand the meaning between the lines C. experts ideas based on what one has read D. gets information and keeps it alive in memory 64. The author of the passage insists that learning the arts_________. A. requires great efforts B. demands real passion C. is less natural than learning maths D. is as natural as learning a language 65. What is a shortcoming of Armstrong’s work according to the author? A. Some ideas are slightly contradictory. B. There is too much discussion on studying science. C. The style is too serious. D. It lacks new information. 66. This passage can be classified as________. A. an advertisement B. a book review C. a feature story D. A news report 63. 答案:C 考点:: 解析:题干的关键词是 goal of reading,回原文定位到第二自然段,第二自然段有两处出现了 关键词,一处说“The goal is to bring the information back to life, not just to treat it as dead facts on paper from dead trees. ”,另一处说“the aim of reading is to express the information you have got from the text. ” C 选项正好与第二处原文相呼应,题干里的 goal of reading 呼应原文的 aim of reading,C 项的 express ideas 呼应原文的 express the information,based on what one has read 呼 应原文的 information you have got from the text。虽然我们不可否认读书的目的也包括 A 和 D 两个选项提到的获取知识,扩大视野,但由于原文没有提及,所以不能成为正确选项。 64. 答案:A 65. 答案:D 考点::细节理解 解析:从第三段可以得出答案 66. 答案:B 考点::考查文章体裁 解析: 通过阅读每段的首句,不难发现第一自然段讲本书的写作对象(This brief book is aimed at high school students),二、三自然段讲本书的写作内容(The author, W. H. Armstrong, starts with the basics … only a third of the book remains after that discussion …),第四自然段是作者对本书 的评价(My other comment is that …),最后一段总结,向读者推荐这本书(I recommend it to any student and any teacher)。把握住了全文的结构,就不难看出这篇文章是一篇书评了。因此正确 答案为 B。 E Have you winterized your horse yet? Even though global warming may have made our climate more mild, many animals are still hibernating(冬眠). It’s too bad that humans can’t hibernate. In fact, as a species, we almost did. Apparently, at times in the past, peasants in France liked a semi-state of human hibernation. So writes Graham Robb, a British scholar who has studied the sleeping habits of the French peasants. As soon as the weather turned cold people all over France shut themselves away and practiced the forgotten art of doing nothing at all for months on end. In line with this, Jeff Warren, a producer at CBC Radio’s The Current, tells us that the way we sleep has changed fundamentally since the invention of artificial(人造的) lighting and the electric bulb. When historians began studying texts of the Middle Ages, they noticed something referred to as “first sleep”, which was not clarified, though. Now scientists are telling us our ancestors most likely slept in separate periods. The business of eight hours’ uninterrupted sleep is a modern invention. In the past, without the artificial light of the city to bathe in, humans went to sleep when it became dark and then woke themselves around midnight. The late night period was known as ”The Watch” It was when people actually kept watch against wild animals, although many of them simply moved around or visited family and neighbours. According to some sleep researchers, a short period of insomnia(失眠) at midnight is not a disorder. It is normal. Humans can experience another state of consciousness around their sleeping, which occurs in the brief period before we fall asleep or wake ourselves in the morning. This period can be an extraordinarily creative time for some people. The impressive inventor, Thomas Edison, used this state to hit upon many of his new ideas. Playing with your sleep rhythms can be adventurous, as anxiety may set in. Medical science doesn’t help much in this case. It offers us medicines for a full night’s continuous sleep, which sounds natural; however, according to Warren’s theory,it is really the opposite of what we need. 67. The example of the French peasants shows the fact that________. A. people might become lazy as a result of too much sleep B. there were signs of hibernation in human sleeping habits C. people tended to sleep more peacefully in cold weather D. winter was a season for people to sleep for months on end 68. The late night was called “The Watch” because it was a time for people______. A. to set traps to catch animals B. to wake up their family and neighbours C. to remind others of the time D. to guard against possible dangers 69. What does the author advise people to do? A. Sleep in the way animals do. B. Consult a doctor if they can’t sleep. C. Follow their natural sleep rhythm. D. Keep to the eight-hour sleep pattern. 70. What is the author’s purpose in writing the passage? A. To give a prescription for insomnia. B. To urge people to sleep less. C. To analyze the sleep pattern of modern people. D. To throw new light on human sleep. 67. 答案:B 考点::推理题 解析:本题是对“论点—论据”的考查,即考查该事例用来证明何种观点。题干的关键词是 example of the French peasants,回原文定位到第二自然段。该论据与第一自然段的论点 之间 有一个衔接过渡句,In fact, as a species, we almost did. 这里的 did 指代上文提到的 hibernate, 因此本题正确答案为 B。 68. 答案:D 考点::细节理解 解析:从 “It was when people actually kept watch against wild animals, although many of them simply moved around or visited family and neighbours. ”可得出答案 69. 答案 C 考点::细节理解 解析:从 “Playing with your sleep rhythms can be adventurous, as anxiety may set in “可以得出答 案 70. 答案:D 考点::主旨大意 (2016 辽宁) A It was a village in India. The people were poor. However, they were not unhappy. After all, their forefathers had lived in the same way for centuries. Then one day. Some visitors from the city arrived. The told the villagers there were some people elsewhere who liked to eat frog’s legs. However, they did not have enough frogs of their own, and so they wanted to buy frogs from other place. This seemed like money for nothing. There were millions of frogs in the fields around, and they were no use to the villagers. All they had to do was catch them. Agreement was reached, and the children were sent into the fields to catch frogs. Every week a truck arrived to collect the catch and hand over the money. For the first time, the people were able to dream of a batter future. But the dream didn’t last long. The change was hardly noticed at first, but it seemed as if the crops were not doing so well. More worrying was that the children fell ill more often, and, there seemed to be more insects around lately. The villagers decided that they couldn’t just wait to see the crops failing and the children getting weak. They would have to use the money earned to buy pesticides(杀虫剂) and medicines. Soon there was no money left. Then the people realized what was happening. It was the frog. They hadn’t been useless. They had been doing an important job---eating insects. Now with so many frogs killed, the insects were increasing more rapidly. They were damaging the crops and spreading diseases. Now, the people are still poor. But in the evenings they sit in the village square and listen to sounds of insects and frogs. These sounds of the night now have a much deeper meaning. 56. From paragraph I we learn that the villagers __________. A. worked very hard for centuries B. dreamed of having a better life C. were poor but somewhat content D. lived a different life from their forefathers 56. C. 细节理解题。第一段中有 The people were poor. However, they were not unhappy. 和 C 项 意思一致。 57. Why did the villagers agree to sell frogs? A. the frogs were easy money B. They needs money to buy medicine C. they wanted to please the visitors D. the frogs made too much noise 57. A. 细节理解题。根据第三段和本段第一句 This seemed like money for nothing. 句中 for nothing 是“免费的”意思,说明青蛙容易得到,并能赚到钱,村民才答应买。 58. What might be the cause of the children’s sickness? A. the crops didn’t do well B. there were too many insects C. the visits brought in diseases D. the pesticides were overused 58. B. 推理判断题。根据倒数第二段中 They had been doing an important job---eating insects. Now with so many frogs killed, the insects were increasing more rapidly. They were damaging the crops and spreading diseases. 可以推断出庄稼收成不好,孩子生病与青蛙减少,害虫增多有关。 59. What can we infer from the last sentence of the text? A. Happiness comes from peaceful life in the country B. Health is more important than money C. The harmony between man and nature is important D. good old day will never be forgotten 59. C. 推理判断题。最后一句 These sounds of the night now have a much deeper meaning. 现在 夜晚的这些声音具有更深刻的意义。该句是一个中介句,说明人们过度捕杀造成生态失衡, 由此也影响了人类,因此,可以推断人与自然的和谐是重要。 B I hated dinner parties. But I decided to give them another shot because I'm in London. And my friend Mallery invited me. And because dinner parties in London are very different from those back in New York. There, '“I’m having a dinner party' means: "I'm booking a table for 12 at a restaurant you can't afford and we'll be sharing the checque evenly, no matter what you eat. " Worse, in Manhattan there is always someone who leaves before the bill arrives. They'll throw down cash, half of what they owe, and then people like me, who don’t drink, end up paying even more. But if I try to use the same trick, the hostess will shout: "Where are you going?" And it's not like I can say I have somewhere to go: everyone know I have nowhere to go. But in London, dinner parties are in people's homes. Not only that, the guests are an interesting mix. The last time I went to one, the guests were from France, India. Denmark and Nigeria; it was like a gathering at the United Nations in New York. The mix is less striking. It's like a gathering at Bloomingdale's, a well-known department store. For New Yorkers, talking about other parts of the world means Brooklyn and Queens in New York. But at Mallery's, when I said that I had been to Myanmar recently, people knew where it was. In New York people would think it was a usual new club 60. What does the word "shot" in Paragraph 1 probably mean? A. Choice. B. Try C. Style. D. Goal 60. B. 词义猜测题。从第一句的 hate,到 but 意义上的转折,说明尽管不喜欢,还是要“尝试 一下”,所以 try 的意思更接近 shot。 61. What does the writer dislike most about dinner parties in New York? A There is a strange mix of people. B. The restaurants are expensive. C. The bill is not fairly shared. D. People have to pay cash 61. C. 细节理解题。根据第一段中 They'll throw down cash, half of what they owe, and then people like me, who don’t drink, end up paying even more. 可知类似作者这样的人在纽约吃饭吃亏,因 为付钱多。 62. What does the author think of the parties in London? A. A bit unusual B.Full of tricks. C.Less costly. D More interesting. 62. D. 推理判断题。根据第二段和其中的句子 Not only that, the guests are an interesting mix. 可以推断伦敦的 party 要比纽约的有趣。 63. What is the author's opinion of some New Yorkers from her experience? A. Easy-going. B. Self-centered. C. Generous. D. Conservative. 63. B. 推理判断题。从最后一段的 talking about other parts of the world means Brooklyn and Queens in New York. 和 In New York people would think it was a usual new club 可以推断,作者 对一些纽约人的看法,是“以自我为中心的”。 C Too much TV-watching can harm children’s ability to learn and even reduce their chances of getting a college degree, new studies suggest in the latest effort to examine the effects of television on children. One of the studies looked at nearly 400 northern California third-graders. Those with TVs in their bedrooms scored about eight points lower on math and language arts tests than children without bedroom TVs. A second study ,looking at nearly 201600 grown-ups in New Zealand, found lower education levels among 26-year –olds who had watched lots of TV during childhood. But the results don’t prove that TV is the cause and don't ride out that already poorly motivated youngsters (年轻人) may watch lots of TV. Their study measured the TV habits of 26-year-olds between ages5 and 15. These with college degrees had watch an average of less than two hours of TV per week night during childhood, compared with an average of more than 2 1/2 hours for those who had no education beyond high school. In the California study, children with TVs in their rooms but no computer at home scored the lowest while those with no bedroom TV but who had home computers scored the highest. While this study does not prove that bedroom TV sets caused the lower scores, it adds to accumulating findings that children shouldn't have TVs in their bedrooms 64. According to the California study, the low-scoring group might _________. A. have watched a lot of TV B. not be interested the in math C. be unable to go to college D. have had computers in their bedrooms 64. A. 细节理解题。根据第二段中 Those with TVs in their bedrooms scored about eight points lower on math and language arts tests than children without bedroom TVs. 可得出答案。 65. What is the researchers' understanding of the New Zealand study results? A. Poorly motivated 26-year-olds watch more TV. B. Habits of TV watching reduce learning interest. C. TV watching leads to lower education levels of the 15-year-olds. D. The connection between TV and education levels is difficult to explain 65. D. 细节理解题。根据第三段 But the results don’t prove that TV is the cause and don't ride out that already poorly motivated youngsters (年轻人)may watch lots of TV. 可以确定答案。 66. What can we learn from the last two paragraphs? A. More time should be spent on computers. B. Children should be forbidden from watching TV. C. 'IV sets shouldn't be allowed in children's bedrooms, D. Further studies on high-achieving students should be done 66. C. 推理判断题。注意最后一段开头的 while 是“尽管”的意思,所以这两段都是围绕儿童 卧室不应该放电视机的问题。 67. What would be the best title for this text? A. Computers or Television B. Effects of Television on Children C. Studies on TV and College Education D. Television and Children's Learning Habits 67. B. 主旨大意题。通读全文可知,文章主要通过两项研究分析了儿童看电视所造成的影响。 文章第一段第一句话是主题句,所以 B 项作为标题是最佳的。 D On May 23,1989, Stefania Follini came out from a cave at Carlsbad, New Mexico. She hadn’t seen the sun for eighteen and a half weeks. Stefania was in a research program, and the scientists in the program were studying body rhythms (节奏)。In this experiment Stefania had spent 130 days in a cave, 30 feet in depth. During her time in the cave, Stefania had been completely alone except for two white mice. Her living place had been very comfortable, but there had been nothing to fell her the time. She’d had no clock or watches, no television or radio. There had been no natural light and the temperature had always been kept at 21℃. The results were very interesting. Stefania had been in the cave for over four months, but she thought she had been there for only two. Her body clock had changed. She hadn't kept to a 24-hour day. She had stayed awake for 20-25 hours and then had slept for 2016 hours. She had eaten fewer meals and had lost 17lbs in weight as a result! She had also become rather depressed (抑郁). How had she spent her time in the cave? As part of the experiment she'd done some physical and mental tests. She'd recorded her daily activities and the results of the tests on a computer. This computer had been specially programmed for the project. Whenever she was free, she'd played cards, read books and listened to music. She'd also learned French from tapes. The experiment showed that our body clocks are affected by light and temperature, For example, the pattern of day and night makes us wake up and go to sleep. However, People are affected in different ways. Some people wake up naturally at 5:00 am, but others don't start to wake up till 9:00 or 2016;00 am. This affects the whole daily rhythm. As a result, the early risers are at their best in the late morning. The late risers, on the other hand, are tired during the day and only come to life in the afternoon or evening! 68. Stefania stayed in the cave for a long time because___________. A. she was asked to do research on mice B. she wanted to experience loneliness C. she was the subject of a study D. she needed to record her life 68. C. 细节理解题。根据第一段中 Stefania was in a research program 和整段的意思,可以排除 A、B、D 项。 69. What is a cause for the change of Stefania’s body clock? A. Eating fewer meals. B. Having more hours of sleep C. Lacking physical exercise. D. Getting no natural light. 69. D. 细节理解题。根据第二段中 She hadn't kept to a 24-hour day. She had stayed awake for 20-25 hours and then had slept for 2016 hours. 可知答案。 70. Where does the text probably come from? A. A novel. B. A news story. C. A pet magazine. D. A travel guide 70. B. 推理判断题。从全文的内容来看,本文主要报道了一个实验及实验得出的结果,所以由 此判断本文可能来自于新闻报道,A 项小说不可能。因为文章没有小说的特点;C 项不正确, 本文没有涉及宠物问题;D 项不正确,因为本文也没涉及旅游的问题。 第二节 (共 5 小题;每小题 2 分,满分 2016 分) 根据短文内容,从短文后的选项中选出能填入空白处的最佳选项。选项中有两项为多余 选项。 I was ten when I first sat with my grandmother behind the cashier(收银台)in her general store. 71 I quickly learned the importance of treating customers politely and saying “thank you. ” At first I was paid in candy. 72 I worked every day after school, and during the summer and on weekends and holidays from 8 a. m. to 7 p. m. My father helped me set up a bank account. 73 By the time I was 12, My grandmother thought I had done such a good job that she promoted me to selling cosmetics(化妆品). I developed the ability to look customers directly in the eye. Even though I was just a kid, women would ask me such things as “ What color do you think I should wear?” I took a real interest in their questions and was able to translate what they wanted into makeup(化妆)ideas. 74 The job taught me a valuable lesson: to be a successful salesperson, you didn’t need to be a Rocket scientist—you needed to be a great listener. __75___ Except they are no longer women purchasing cosmetics from me; instead, they are kids who tell me which toys they would like to see designed and developed. A. Later I received 50 cents an hour. B. Before long, she let me sit there by myself. C. I ended my selling a record amount of cosmetics. D. Today I still carry that lesson with me: I listen to customers. E. My grandma’s trust taught me how to handle responsibility. F. Soon I found myself looking more beautiful than ever before. G. Watching my money grow was more rewarding than anything I could have bought. 71. B. 从后一句 I quickly learned the importance of treating customers politely and saying “thank you. ”可推知,作者的祖母先让作者跟着她学,然后让作者独立工作,所以他很快学会有礼貌 地对待客人的重要性。 72. A. 前面有 at first,和本项中的 later 一致。 73. G。根据前一句的 My father helped me set up a bank account. 可确定答案。 74 C. 同样根据前一句确定答案。 75. D. 上文提到 a valuable lesson,这里承接上下文。 (2016 北京) 202016 年高考北京英语卷阅读理解分析 概述 今年是新课改第一年,为了新旧顺利衔接,加快新课改方案在高中的顺利推进,阅读理 解部分增加了新题型"七选五",命题难度降低是情理之中的。但经过一年的"试水",师生们适 应新课改的思路后,明年难度很有可能会升上去。这样,对今年的阅读理解试题的专业分析 非常有必要。学而思乐加乐英语,特邀高中阅读单项教研专家杨超老师,对今年的阅读理解 做了全方位的权威解析。 今年的阅读理解题遵循《202016 年高考考试说明》,题型与往年保持了一致,难度稍有降 低。文章本身和所附问题总词数约 2300 词,在往年 2000-2200 的词数基础上稍有增加。全面 考察了细节题、推断题、文章结构题、词义猜测题。在选材上覆盖面广,语言地道,文体特 征鲜明。 文章体裁和主要内容介绍 夹叙夹议:通过记叙作者与一只被遗弃的小狗的故事,说明不应该轻易对他人的行为下 定论。(A 篇) 应用文:一位主编的公开信,劝说同行要注重手下记者的职业发展,激发他们的潜能。(B 篇) 说明文:介绍不同文化背景的人有不同的谈话节奏,及其带来的问题。(C 篇) 议论文:支持高等教育应该由受教育人自付费用。(D 篇) 说明文:介绍缪扎克音乐(一种通过线路向机场、商场、餐馆等播放的背景录音音乐)。(E 篇) 与往年比较的变化 1.每一篇文章都加上了标题:一定程度上降低了考试难度,帮助学生把握文章的中心意 思。 2.应用文"消失":今年阅读理解形式上保留了应用文,因为 B 篇体裁仍为书信,实则为 说理性质,增加了考题的难度。 答案解析及技巧运用 A Goldie's Secret She turned up at the doorstep of my house in Cornwall. No way could I have sent her away. No way, not me anyway. Maybe someone had kicked her out of their car the night before. "We're moving house. '; "No space for her any more with the baby coming. " "We never really wanted her, but what could we have done? She was a present. " People find all sorts of excuses for abandoning an animal. And she was one of the most beautiful dogs I had ever seen. I called her Goldie. If I had known what was going to happen I would have given her a more creative name. She was so unsettled during those first few days. She hardly ate anything and had such an air of sadness about her. There was nothing I could do to make her happy, it seemed. Heaven knows what had happened to her at her previous owner's. But eventually at the end of the first week she calmed down. Always by my side, whether we were out on one of our long walks or sitting by the fire. That's why it was such a shock when she pulled away from me one day when we were out for a walk. We were a long way from home, when she started barking and getting very restless. Eventually I couldn't hold her any longer and she raced off down the road towards a farmhouse in the distance as fast as she could. By the time I reached the farm I was very tired and upset with Goldie. But when I saw her licking (舔) the four puppies (幼犬) I started to feel sympathy towards them. "We didn't know what had happened to her," said the woman at the door. "I took her for a walk one day, soon after the puppies were born, and she just disappeared. " "She must have tried to come back to them and got lost," added a boy from behind her. ' I must admit I do miss Goldie, but I've got Nugget now, and she looks just like her mother. And I've learnt a good lesson: not to judge people. 56. How did the author feel about Goldie when Goldie came to the house? A. Shocked. B. Sympathetic. C. Annoyed. D. Upset. 57. In her first few days at the author's house, Goldie . AI felt worried B. was angry C. ate a little D. sat by the fire 58. Goldie rushed off to a farmhouse one day because she . A. saw her puppies B. heard familiar barking C. wanted to leave the author D. found her way to her old home 59. The passage is organized in order of . A. time B. effectiveness C. importance D. complexity A 篇 阅读技巧:夹叙夹议,叙为议服务,议为叙指导。把握了这句话就把握了阅读此类文章 的真谛。叙述不分重点关注人物,情节的变化;议论一定看清楚作者想表明的道理。 答题技巧:本篇题目由 3 个细节题和一道文章结构题组成,难度都不大。细节题要求学 生把握文章中的原文,做合理推断和转述。比如 57 题,将 unsettled 理解成 worried。文章结构 题基本属于送分题,要求学生作答,此篇记叙文按照时间顺序发展。 56. B 情绪推断,较难题。此推断题的难点在于,原文没有直接的形容词表述,要求学生通过 具体的描述总结。原文第 2 段 She hardly ate anything and had such an air of sadness about her. There was nothing I could do to make her happy, it seemed. Heaven knows what had happened to her at her previous owner's. 划线句子表明作者同情被遗弃的小狗,希望帮助它高兴起来。 57. A 细节题,较难题。要做对此题主要要排除 C 项的干扰,原文第 2 段 She hardly ate anything and had such an air of sadness about her. hardly ate anything 的表述与 ate little 是不一样的。前者 是几乎什么都不吃,后者是吃得少,此错误选项属于"改变否定/肯定的程度"。 58. D 原因推断题,简单题,通过原文第 4 段的描写可得正确答案。 59. A 问文章的行文结构,简单题,此篇记叙文按照时间顺序发展。 B Open Letter to an Editor I had an interesting conversation with a reporter recently---one who works for you. In fact, he's one of your best reporters. He wants to leave. Your reporter gave me a copy of his resume (简历) and photocopies of six stories that he wrote for you. The headlines showed you played them proudly. With great enthusiasm, he talked about how he finds issues (问题), approaches them, and writes about them, which tells me he is one of your best. I'm sure you would hate to lose him. Surprisingly, your reporter is not unhappy. In fact, he told me he really likes his job. He has a great assignment (分工), and said you run a great paper. It would be easy for you to keep him, he said. He knows that the paper values him. He appreciates the responsibility you've given him, takes ownership of his profession, and enjoys his freedom. So why is he looking for a way out? He talked to me because he wants his editors to demand so much more of him. He wants to be pushed, challenged, coached to new heights. The reporter believes that good stories spring from good questions, but his editors usually ask how long the story will be, when it will be in, where it can play, and what the budget is. He longs for conversations with an editor who will help him turn his good ideas into great ones. He wants someone to get excited about what he's doing and to help him turn his story idea upside down and inside out, exploring the best ways to report it. He wants to be more valuable for your paper. That's what you want for him, too, isn't it? So your reporter has set me thinking. Our best hope in keeping our best reporters, copy editors, photographers, artists---everyone--is to work harder to make sure they get the help they are demanding to reach their potential. If we can't do it, they'll find someone who can. 60. What does the writer think of the reporter? A. Optimistic. B. Imaginative. C. Ambitious. D. Proud. 61. What does the reporter want most from his editors in their talks? A. Finding the news value of his stories. B. Giving him financial support. C. Helping him to find issues. D. Improving his good ideas. 62. Who probably wrote the letter? A. An editor. B. An artist. C. A reporter. D. A reader. 63. The letter aims to remind editors that they should __ A. keep their best reporters at all costs B. give more freedom to their reporters C. be aware of their reporters' professional development D. appreciate their reporters' working styles and attitudes B 篇 阅读技巧:第一段出现重要信息:特殊标点"破折号"和隐性转折词 in fact,因此后面信息 极其重要:写信给一个编辑说他的 reporter 很棒,但是要离开了;第二段要把握两个中文注释, 此 reporter 给作者简历并充满热情地向其描述了自身特质;第三段平淡无奇,机械寻找重要信息 出处—in fact,故得知该 reporter 干一行爱一行,精神可嘉;接着第四段一个问句,基本预示情 节的转折,一眼即得知下面要详细阐述该 reporter 要离开的原因;继续往下,重要信息处稍微留 意,即转折词 but,因果词 so,最后段的最高级 best 句,以及有特殊标点出现的最后一句。通 过这几处内容的梳理,文意会变得清晰很多-年轻记者需要主编充分发挥他们的潜能,在竞争 与挑战中寻求职业发展。 答题技巧:此篇的 4 个题目包括 3 个推断题,其中包括对人物态度、作者身份、写作目 的的推断。人物态度要求学生具备将具体描述…wants to be coached to new heights 等转换成形 容词表述的能力;作者身份推断题要求学生学会通过人称代词 our 判断;写作目的要求学生对 全文的把握能力。 60. C 判断推理题,难题。原文He wants to be pushed, challenged, coached to new heights. . 要通过 划线处的具体描述总结出ambitious这个词:有抱负的。 61. D 细节题,较难题。困难之处在于不好定位原文,故用排除法。从选项中找关键词,逐个 排查,得出正确选项 D. 62. A 推断作者身份,较难题。此类型题目把握方法很简单:文章对象(主编)+人称(our),不 难得出正确答案作者也是主编。 63. C 考查文章写作目的,较难题。抓住文章结尾是王道:Our best hope Our best hope in keeping our best reporters, copy editors, photographers, artists---everyone--is to work harder to make sure they get the help they are demanding to reach their potential 再结合前面 60 题的答案,不难选出正 确答案。 C Pacing and Pausing Sara tried to befriend her old friend Steve's new wife, but Betty never seemed to have anything to say. While Sara felt Betty didn't hold up her end of the conversation, Betty complained to Steve that Sara never gave her a chance to talk. The problem had to do with expectations about pacing and pausing. Conversation is a turn-taking game. When our habits are similar, there's no problem. But if our habits are different, you may start to talk before I'm finished or fail to take your turn when I'm finished. That's what was happening with Betty and Sara. It may not be coincidental that Betty, who expected relatively longer pauses between turns, is British, and Sara, who expected relatively shorter pauses, is American. Betty often felt interrupted by Sara. But Betty herself became an interrupter and found herself doing most of the talking when she met a visitor from Finland. And Sara had a hard time cutting in on some speakers from Latin America or Israel. The general phenomenon, then, is that the small conversation techniques, like pacing and pausing, lead people to draw conclusions not about conversational style but about personality and abilities. These habitual differences are often the basis for dangerous stereotyping (思维定式). And these social phenomena can have very personal consequences. For example, a woman from the southwestern part of the US went to live in an eastern city to take up a job in personnel. When the Personnel Department got together for meetings, she kept searching for the right time to break in--and never found it. Although back home she was considered outgoing and confident, in Washington she was viewed as shy and retiring. When she was evaluated at the end of the year, she was told to take a training course because of her inability to speak up. That's why slight differences in conversational style--tiny little things like microseconds of pause-can have a great effect on one's life. The result in this case was a judgment of psychological problems---even in the mind of the woman herself, who really wondered what was wrong with her and registered for assertiveness training. 64. What did Sara think of Betty when talking with her? A. Betty was talkative. B. Betty was an interrupter. C. Betty did not take her turn. D. Betty paid no attention to Sara. 65. According to the passage, who are likely to expect the shortest pauses between turns? A. Americans. B. Israelis. C. The British. D. The Finns. 66. We can learn from the passage that __ A. communication breakdown results from short pauses and fast pacing B. women are unfavorably stereotyped in eastern cities of the US C. one's inability to speak up is culturally determined sometimes D. one should receive training to build up one's confidence 67. The underlined word "assertiveness" in the last paragraph probably means __ A. being willing to speak one's mind B. being able to increase one's power C. being ready to make one's own judgment D. being quick to express one's ideas confidently C 篇 阅读技巧:开篇复杂的人物关系,以及第三段几个国家之间的复杂对比是学生阅读的最 大困难。攻略:1. 动笔:中国学生对英语名字太不敏感,请划出来。2. 再难读说明文的目标 不变:找出文章说明对象是什么。 答题技巧:包括 3 个细节题和一个词义猜测题,其中 65 题细节比较排序题是亮点,要求 学生从复杂的表述中理清顺序,区别出哪国人的说话节奏最快。 64. C 细节题,难题。难在弄不清谁是 S 谁是 B,耐心读,动笔划,从第一段得出正确答案并不 难:Sara felt Betty didn't hold up her end of the conversation 65. B 细节排序题,难题。还是耐心读,动笔划。S 代表美国人,B 代表英国人(加起来?),S 比 B 期待谈话间隙时间更短,又在以色列人(Israelis)说话时插不上嘴,故答案选说话嗒嗒嗒嗒 的以色列人。 66. C 变态细节题,较难题。A 不符原文,B 无中生有,D 无中生有,C 关键要理解 culturally determined,由文化决定或者受文化影响。 67. A 词义猜测题,较难。难点在于 A 选项的干扰作用。原文说那位 MM 的 inability to speak up 注意别人认为她没能力,不是说她不愿意。所以 D 比 A 好,此处用反义对比方法。 D The Cost of Higher Education Individuals (个人) should pay for their higher education. A university education is of huge and direct benefit to the individual. Graduates earn more than non-graduates. Meanwhile, social mobility is ever more dependent on having a degree. However, only some people have it. So the individual, not the taxpayers, should pay for it. There are pressing calls on the resources (资源) of the government. Using taxpayers' money to help a small number of people to earn high incomes in the future is not one of them. Full government funding (资助) is not very good for universities. Adam Smith worked in a Scottish university whose teachers lived off student fees. He knew and looked down upon 18th-century Oxford, where the academics lived comfortably off the income received from the government. Guaranteed salaries, Smith argued, were the enemy of hard work; and when the academics were lazy and incompetent, the students were similarly lazy. If students have to pay for their education, they not only work harder, but also demand more from their teachers. And their teachers have to keep them satisfied. If that means taking teaching seriously, and giving less time to their own research interests, that is surely something to celebrate. Many people believe that higher education should be free because it is good for the economy ( 经 济 ). Many graduates clearly do contribute to national wealth, but so do all the businesses that invest (投资) and create jobs. If you believe that the government should pay for higher education because graduates are economically productive, you should also believe that the government should pay part of business costs. Anyone promising to create jobs should receive a gift of capital from the government to invest. Therefore, it is the individual, not the government, who should pay for their university education. 68. The underlined word "them" in Paragraph 2 refers to A. taxpayers B. pressing calls C. college graduates D. government resources 69. The author thinks that with full government funding A. teachers are less satisfied B. students are more demanding C. students will become more competent D. teachers will spend less time on teaching 70. The author mentions businesses in Paragraph 5 in order to A. argue against free university education B. call on them to finance students' studies C. encourage graduates to go into business D. show their contribution to higher education D 篇 阅读技巧:此篇议论文难度较低。把握议论文的阅读两步:1. 作者想说服你什么?2. 分 了几个并列的点来说服你? 答题技巧:此篇包括 2 个推断题和 1 个细节题。其中 68 题要求学生找出 them 的所指, 对学生分析代词的所指提出了更高的要求,此项技能在完型和阅读中都有体现;70 题属于基 于议论文结构的设问,问最后一段的例子在文章中的作用,对议论文的结构和写作模式有了 解的同学都可以轻松应答。 68. B 推断代词所指,较难题。There are pressing calls on the resources (资源) of the government. Using taxpayers' money to help a small number of people to earn high incomes in the future is not one of them. ww 往前看,发现前句就一个复数名词,就它了。 69. D 细节题,简单题。根据题干关键词"full government funding"定位原文,根据"文题顺序一 致"原则从上一题 them 后面去找,会找到 Full government funding (资助) is not very good for universities…. and when the academics were lazy and incompetent, the students were similarly lazy. "lazy"转换成"spendless time"不难得出正确选项。 70. A 结构推断题,简单题。题目问提到 business 有什么目的,就等于问议论文中的论据有什么 用,地球人都知道:支持论点。马上从本段段首找论点:Many people believe that higher education should be free…从全文的哪都可以看出作者就是要反对 free higher education. 根据短文内容,从短文后的七个选项中选出能填入空白处的最佳选项。选项中有两项为 多余选项。 Muzak The next time you go into a bank, a store, or a supermarket, stop and listen. What do you hear? 71 It's similar to the music you listen to, but it's not exactly the same. That's because this music was especially designed to relax you, or to give you extra energy. Sometimes you don't even realize the music is playing, but you react to the music anyway. Quiet background music used to be called "elevator (电梯) music" because we often heard it in elevators. But lately we hear it in more and more places, and it has a new name "Muzak". About one-third of the people in America listen to "Muzak" everyday. The music plays for 15 minutes at a time, with short pauses in between. It is always more lively between ten and eleven in the morning, and between three and four in the afternoon, when people are more tired. 72 If you listen to Muzak carefully, you will probably recognize the names of many of the songs. Some musicians or songwriters don't want their songs to be used as Muzak, but others are happy when their songs are chosen. Why? 73 Music is often played in public places because it is designed to make people feel less lonely when they are in an airport or a hotel. It has been proven that Muzak doeswhat it is designed to do. Tired office workers suddenly have more energy when they hear the pleasant sound of Muzak in the background. 74 Supermarket shoppers buy 38 percent more groceries. 75 . They say it's boring to hear the same songs all the time. But other people enjoy hearing Muzak in public places. They say it helps them relax and feel calm. One way or another, Muzak affects everyone. Some farmers even say their cows give more milk when they hear Muzak! A. Some people don't like Muzak. B. The music gives them extra energy. C. Music is playing in the background. D. Factory workers produce 13 percent more. E. Muzak tends to help people understand music better. F. They get as much as $4 million a year if their songs are used. G. Muzak is played in most of the big supermarkets in the world. E 篇 阅读技巧:1. 找说明对象(Muzak)2. 找文章讲了说明对象的那些方面。 答题技巧:第一年出题果然不难,7 选 5 只有 74 题稍难,其余 4 空都可以根据相邻句子 间的指代关系、因果关系、并列关系,较容易地选出正确答案。 71. C 王道:看前后邻句。找不到再扩大范围找。 此题,前句没看头,看后句:It's similar to the music you listen to…捉住主语 it 及它对应的 music,从 7 个句子中选出唯一用 music 开头的 C 句。 72. B 同样用指代的原则。前句 when people are more tired. 根据指代原则,确定答案在 B 和 F 里面:B. The music gives them extra energy. F. They get as much as $4 million a year if their songs are used. F 明显不符合文章发展,提钱,俗!选 B 73. F 利用因果关系 前句 Why?其他人高兴什么?钱啊!从剩下的 A,D,E,F 里找原因,只能找出"钱"来。 74. D 这是唯一单纯看一句话找不出答案的题,也是 5 题中最难的题。 利用并列关系。前一句描述 tired office workers,后一句描述 supermarket shoppers,后句 出 现 Supermarketshoppersbuy38percentmoregroceries. 由 此 推 断 D 为 正 确 答 案 。 Factoryworkersproduce13percentmore. 主语和百分数都是一一对应关系。 75. A 利用并列原则,后面有 But other people enjoy hearing…,故选择 A. Some people don't like Muzak. 这是送分题。 11 年阅读理解备考注意 1、保证一定的阅读量,每天 2-4 篇。 2、有精读有泛读,不要篇篇都仔细看。 3、精读应以读近几年各地高考试题,特别是北京试题为主,因为它们才代表了高考命题 的真髓。 4、精读时应适当练习句子成分分析,长句难句增加,必然对学生分析句子结构的基本功 要求越来越高。 5、精读时还要注意词义词性的辨别,后置定语,连接词语以及由动词形式变化反映的句 意的变化。 6、考前练限时阅读,35 分钟完成 5 篇总词数约 2400 的各类文章。 7、注意总结六类问题的答题规律(主旨大意题,具体细节题,推断词义句义题,判断推 理题,文章结构题和观点态度题)。 8、除掌握高考词汇表上的单词外,还应掌握考纲要求的构词法知识(比如今年高考中对 于 A 篇 unsettled 一词、C 篇 a turn-taking game 等词语的理解)。 (2016 重庆) A One morning more than thirty years ago, I entered the Track Kitchen, a restaurant where everyone from the humblest(卑微的) to the most powerful came for breakfast. I noticed an empty chair next to an elderly, unshaven man, who looked somewhat disheveled. He was wearing a worn-out hat and was alone. I asked if I might join him. He agreed quietly and I sat down to have my breakfast. We cautiously began a conversation and spoke about a wide rang of things. We never introduced ourselves. I was concerned that he might have no money and not be able to afford something to eat. So as I rose to go back to the counter and buy a second cup of coffee, I asked, “My I get you something ?” “A coffee would be nice. ” Then I bought him a cup of coffee, We talked more, and he accepted another cup of coffee, Finally, I rose to leave, wished him well, and headed for the exit. At the door I met one of my friends. He asked, “How did you get to know Mr. Galbreath?” “Who?” “The man you were sitting with. He is chairman of the Board of Churchill Downs. ” I could hardly believe it. I was buying, offering a free breakfast, and feeling pity for one of the world’s richest and most powerful men! My few minutes with Mr. Galbreath changed my life. Now I try to treat everyone with respect, no matter who I think they are, and no matter another human being with kindness and sincerity. 56. What does the underlined word “disheveled” mean? A. Unfriendly. B. Untidy. C. Gentle. D. Kind. 57. The author bought coffee for the old man because A. he thought the old man was poor B. he wanted to start a conversation C. he intended to show his politeness D. he would like to thank the old man 58. How did the author probably feel after he talked with his friend? A. Proud. B. Pitiful. C. Surprised. D. Regretful 59. What is the message mainly expressed in the story? A. We should learn to be generous. B. It is honorable to help those in need. C. People in high positions are not like what we expect. D. We should avoid judging people by their appearances. 【语篇解读】本文讲述了“我”在吃早点的时候遇到一个看起来穿着邋遢的老人,并且帮助他买 了一杯咖啡,事后得知他是最富有的 Mr. Galbreath,旨在告诉我们不要以貌取人。。 56. 答案:B 解析:猜测词义题。由上文中的 I noticed an empty chair next to an elderly, unshaven man 何下文 中的 He was wearing a worn-out hat and was alone. 可知 disheveled 在这儿是指仪容不整,穿着 邋遢的意思。所以答案:选 B 项。 57. 答案:A 解析:细节理解题。根据文中 I was concerned that he might have no money and not be able to afford something to eat. 可以判断选 A 项。 58. 答案:C 解析:推理判断题。根据 I could hardly believe it. I was buying, offering a free breakfast, and feeling pity for one of the world’s richest and most powerful men!可知,作者在他朋友告诉他那个老人是 Mr. Galbreath 之后感到很吃惊。由此判断选 C 项。 59. 答案:D 解析:主旨大意题。根据文章最后一句 Now I try to treat everyone with respect, no matter who I think they are, and no matter another human being with kindness and sincerity. 可知,作者想要表 达的是不要以貌取人。由此判断选 D 项。 B Love, success, happiness, family and freedom----how important are these values to you? Here is one interview which explores the fundamental questions in life. Question: Could you introduce yourself first? Answer: My name is Misbah, 27 years old. I was born in a war-torn area. Right now I’m a web designer. Q: What are your great memories? A: My parents used to take us to hunt birds, climb trees, and play in the fields. For me it was like a holiday because we were going to have fun all day long. Those are my great memories. Q: Does your childhood mean a lot to you? A: Yes. As life was very hard, I used to work to help bring money in for the family. I spent my childhood working, with responsibilities beyond my age. However, it taught me to deal with problems all alone. I learnt to be independent. Q: What changes would you like to make in your life? A: If I could change something in my life, I’d change it so that my childhood could have taken place in another area. I would have loved to live with my family in freedom. Who cares whether we have much money, or whether we have a beautiful house? It doesn’t matter as long as I can live with my family and we are safe. Q: How do your get along with your parents? A: My parents supported me until I came of age. I want to give back what I’ve got. That’s our way. But I am working in another city. My only contact with my parents now is through the phone, but I hate using it. It filters(过滤) out your emotion and leaves your voice only. My deepest feelings should be passed through sight, hearing and touch. 60. In Misbah’s childhood, . A. he was free from worry B. he liked living in the countryside C. he was fond of getting close to nature D. he often spent holidays with his family 61. What did Misbah desire most in his childhood? A. A colorful life. B. A beautiful house. C. Peace and freedom. D. Money for his family. 62. How would Misbah prefer to communicate with his parents? A. By chatting on the Internet. B. By calling them sometimes. C. By paying weekly visits. D. By writing them letters. 63. If there were only one question left, what would it most probably be? A. What was your childhood dream? B. What is your biggest achievement? C. What is your parents’ view of you? D. What was your hardest experience in the war? 【语篇解读】本文通过一个采访,把 Misbah 这个人的生活、成长展示给我们。 60. 答案:C 解析:细节理解题。由 My parents used to take us to hunt birds, climb trees, and play in the fields. For me it was like a holiday because we were going to have fun all day long. Those are my great memories. 可知,Misbah 喜爱大自然。所以答案:选 C 项。 61. 答案:C 解析:推理判断题。根据文中 I would have loved to live with my family in freedom. Who cares whether we have much money, or whether we have a beautiful house? It doesn’t matter as long as I can live with my family and we are safe. 可知,作者向往和平自由的生活。故选 C 项。 62. 答案:C 解析:细节理解题。根据My only contact with my parents now is through the phone, but I hate using it. It filters out your emotion and leaves your voice only. My deepest feelings should be passed through sight, hearing and touch. 可知,Misbah 讨厌在电话里与父母联系,因为它只剩下了声音, 而 Misbah 想要的是与父母面对面的交流。四个选项中只有 C 项能够让 Misbah 与父母面对面。 由此判断选 C 项。 63. 答案:B 解析:推理判断题。其它三个问题在上文中都有所涉及,只有 B 项没有。所以如果还剩一个 问题,很有可能就是 B 项。 C It is hardly surprising that clothing manufacturers(生产商) follow certain uniform standards for various features(特征) of clothes. What seems strange, however, is that the standard adopted for women is the opposite of the one for men. Take a look at the way your clothes button. Men’s clothes tend to button from the right, and women’s form the left. Considering most of the word’s population----men and women----are right-handed, the men’s standard would appear to make more sense for women. So why do women’s clothes button from the left? History really seems to matter here. Buttons first appeared only on the clothes of the rich in the 17th century, when rich women were dressed by servants. For the mostly right-handed servants, having women’s shirts button from the left would be easier. On the other hand, having men’s shirts button form the right made sense, too. Most men dressed themselves, and a sword drawn from the left with the right hand would be less likely to get caught in the shirt. Today women are seldom dressed by servants, but buttoning form the left is still the standard for them. Is it interesting? Actually, a standard, once set, resists change. At a time when all women’s shirts buttoned form the left, it would have been risky for any single manufacturer to offer women’s shirts that buttoned from the right. After all, women had grown so used to shirts which buttoned from the left and would have to develop new habits and skills to switch. Besides, some women might have found it socially awkward to appear in public wearing shirts that buttoned from the right, since anyone who noticed that would believe they were wearing men’s shirts. 64. What is surprising about the standard of the clothing industry? A. It has been followed by the industry for over 400 years. B. It is different for men’s clothing and women’s. C. It woks better with men than with women. D. It fails to consider right-handed people. 65. What do we know about the rich men in the 17th century? A. They tended to wear clothes without buttons. B. They were interested in the historical matters. C. They were mostly dressed by servants. D. They drew their swords from the left. 66. Women’s clothes still button from the left today because . A. adopting men’s style is improper for women B. manufacturers should follow standards C. modern women dress themselves D. customs are hard to change 67. The passage is mainly developed by . A. analyzing causes B. making comparisons C. examining differences D. following the time order 【语篇解读】本文由男女衣服的不同,分析产生这种不同的原因,并且告诉我们这种不同为 什么延续到现在。 64. 答案:B 解析:细节理解题。由 “What seems strange , however, is that the standard adopted for women is the opposite of the one for men. ”可知男女衣服的标准不一样令人惊奇。所以答案:选 B 项。 65. 答案:D 解析:细节理解题。根据文中“Most men dressed themselves, and a sword drawn from the left with the right hang would be less likely to get caught in the shirt. ”可判断选 D 项。 66. 答案:D 解析:细节理解题。根据“Actually, a standard , once set, resists change. ”可知,习惯一旦养成, 就很难改变是女式衣服的纽扣仍然在左边的原因。由此判断选 D 项。 67. 答案:A 解析:组织结构题。通观整篇文章可以看出,作在开始提到一个现象――男女衣服的标准相 反,然后分析这种现象产生的历史原因,最后告诉我们现在这是这样的原因。由此判断选 A 项。 D Sitting on the peaceful coast of Galapagos Islands. Ecuador, watching the sun move quietly into the sea, you shouldn’t forget that Charles Darwin (1809-1882) arrived here in 1835. He stayed on the islands for five weeks, observing various animals. This finally inspired (启发) his famous work, On the Origin of Species. You can certainly follow Darwin’s footsteps and enjoy a trip from four to seven days to the islands. The islands are certainly a paradise (天堂) for wildlife, as there are no natural killers on the islands and the number of boats and visitors is under government control. Though you cannot walk freely as Darwin did about 200 years ago, each day is as impressive as it could be. The most well-known animals of the Galapagos is the giant tortoise(巨型海龟), which can be seen moving slowly around the highlands of Sanra Cruz, the second largest island in the archipelago(群岛). Some of these creatures are so old that they might have been seen in their youth by Darwin himself. Despite strict control over activities and timing, your stay on the Galapagos will be remembered as a chain of incomparable pictures: diving with sea lions that swim and play within inches of you; feeling small sharks touch your feet as you swim; and, most magically, seeing a whale and her baby surface with a great breath of air. Travelling between the islands and observing the wildlife that so inspired Darwin, you will feel as though you are getting a special view of an untouched world. At night you will sleep on board the ship, leaving the wildlife in complete occupation of the islands, which are as undisturbed now as they have been since the beginning of time. 68. What do we know about Darwin’s visit to the islands? A. He studied different creatures on the islands B. He completed his famous book on the islands C. He was touched by the geography of the islands D. He was attracted by well-known animals of the islands 69. Which of the following plays a role in making the islands “a paradise for wildlife”? A. Animals on the islands feed on grass. B. Local government forbids killing wildlife. C. People cannot visit the islands as they wish. D. Tourists are not allowed to touch the animals. 70. Your stay on the islands will be most impressive mainly because of . A. The beautiful sea views B. Darwin’s inspiring trip C. a closer view of animals D. various daring activities 71. Which of the following would be the best title for the passage? A. A Unique Attraction for wildlife Lovers B. Calapagos as a Paradise for Adventures C. Charles Darwin as a symbol of Galapagos D. A successful Example of wildlife protection 【语篇解读】本文介绍了一个冒险家的乐园 Galapagos Islands,达尔文曾在这儿受到启发写出 了《物种起源》一书,同时,由于这儿保护得很好,直到现在依然是旅游的好去处。 68. 答案:A 解析:细节理解题。由文中的 He stayed on the islands for five weeks, observing various animals. 可知达尔文在这儿观察了各种各样的动物。 69. 答案:C 解析:细节理解题。根据文中. . . strict control over activities and timing. . . 可知,当地对游客的 活动和参观时间都是有严格限制的,所以答案:选 C 项。 70. 答案:C 解析:细节理解题。根据 your stay on the Galapagos will be remembered as a chain of incomparable pictures: diving with sea lions that swim and play within inches of you; feeling small sharks touch your feet as you swim; and, most magically, seeing a whale and her baby surface with a great breath of air. 可知,在 Galapagos 岛上能与动物亲密接触。 71. 答案:A 解析:主旨大意题。这篇文章主要是向我们推荐 Galapagos Islands,因此题目要具有醒目的特 点,方能达到吸引读者眼球的目的,故题目以 A 为佳。 E Humans are naturally drawn to other life forms and the worlds outside of our own. We take delight in the existence of creatures and even whole societies beyond our everyday lives. This sense of wonder is universal. Look at the efforts that scientists have made to find out whether life of some kind exists on Mars, and the popularity of fantasy(幻想) literature or movies like The Lord of the Rings. This sense of wonder draws us to each other, to the world around us, and to the world of make-believe. But have we gone so far in creating worlds of fantasy that we are missing the pleasure of other worlds that already exist all around us? Human beings, as biologists have suggested, possess an inborn desire to connect with and understand other life forms. However, people, especially in big cities, often lead rather isolated lives. In a study of British schoolchildren, it was found that children by age eight were much more familiar with characters from television shows and video games than with common wildlife. Without modem technology, a small pond could be an amazing world filled with strange and beautiful plants, insects, birds, and animals. When we lack meaningful interaction( 交 互 ) with the world around us, and sometimes even with our families and friends, we seek to understand and communicate with things that exist only in our imaginations or on a computer screen。 The world of make-believe is not necessarily bad. But when the world of fantasy becomes the only outlet(出路) for our sense of wonder, then we are really missing something. We are missing a connection with the living world. Other wonderful worlds exist all around us. But even more interesting is that if we look closely enough, we can see that these worlds, in a broad sense, are really part of our own. 72. The popularity of The Lord of the Rings proves A. the close connection between man and the fantasy world B. the wonderful achievements of fantasy literature C. the fine taste of moviegoers around the world D. the general existence of the sense of curiosity 73. What can we infer from the underlined sentence in Paragraph 3? A. People are far less familiar with the world of fantasy. B. The world around us could serve as a source of wonder. C. The world of fantasy can be mirrored by a small and lively pond. D. Modern technology prevents us from developing our sense of wonder. 74. If our sense of wonder relies totally on the world of make-believe, we will A. fail to appreciate the joy in our lives B. be confused by the world of make-believe C. miss the chance to recognize the fantasy world D. be trapped by other worlds existing all around us 75. What is the main purpose of the passage? A. To show us the hidden beauty in our world. B. To warn us not to get lost in the fantasy world. C. To argue against the misuse of the sense of wonder. D. To discuss the influence of the world of make-believe. 【语篇解读】本文介绍了一个冒险家的乐园 Galapagos Islands,达尔文曾在这儿受到启发写出 了《物种起源》一书,同时,由于这儿保护得很好,直到现在依然是旅游的好去处。 72. 答案:D 解析:细节理解题。由第二段可知,举 The Lord of the Rings 这个例子是为了证明人们的好奇 心是普遍存在的。所以答案:选 D 项。 73. 答案:B 解析:推理判断题。Without modem technology, a small pond could be an amazing world filled with strange and beautiful plants, insects, birds, and animals. 这句话告诉我们,如果没有现代技术,即 使一个小小的池塘也能成为一个令人惊异的世界。从而我们可以推测出我们现实世界也是很 美丽的,也是一切期间产生的根源。 74. 答案:A 解析:细节理解题。根据 The world of make-believe is not necessarily bad. But when the world of fantasy becomes the only outlet for our sense of wonder, then we are really missing something. We are missing a connection with the living world. 可知,如果幻想成为我们好奇感的唯一出路的话, 我们就会失去一些东西,就会失去与现实世界的联系,我们也就不能体会到我们现实生活中 的快乐。由此判断选 A 项。 75. 答案:B 解析:作者意图题。由文章最后一段可知,作者写这篇文章的目的就是要告诉我们不要沉迷 在幻想之中。由此判断选 B 项。 (2016 浙江) A. When you are little, the whole world feels like a big playground. I was living in Conyers, Georgia the summer it all happened. I was a second grader, but my best friend Stephanie was only in the first grade. Both of our parents were at work and most of the time they let us go our own way. It was a hot afternoon and we decided to have an adventure in Stephanie’s basement. As I opened the basement door, before us lay the biggest room, full of amazing things like guns, dolls, and old clothes. I ran downstairs, and spotted red steel can. It was paint. I looked beyond it and there lay even more paint in bright colors like purple, orange, blue and green. “Stephanie, I just found us a project for the day. Get some paintbrushes. We are fixing to paint. ” She screamed with excitement as I told her of my secret plans and immediately we got to work. We gathered all the brushes we could find and moved all of our materials to my yard. There on the road in front of my house, we painted bit stripes ( 条 纹 ) of colors across the pavement (人行道). Stripe by stripe, our colors turned into a beautiful rainbow. It was fantastic! The sun was starting to sink. I saw a car in the distance and jumped up as I recognized the car. It was my mother. I couldn’t wait to show her my masterpiece. The car pulled slowly into the driveway and from the look on my mother’s face, I could tell that I was in deep trouble. My mother shut the car door and walked towards me. Her eyes glaring, she shouted, “What in the world were you thinking? I understood when you made castles out of leaves, and climbed the neighbors’ trees, but this! Come inside right now!” I stood there glaring hack at her for a minute, angry because she had insulted (侮辱) my art. “Now go clean it up!” Mother and I began cleaning the road. Tears ran down my cheeks as I saw my beautiful rainbow turn into black cement. Though years have now passed, I still wonder where my rainbow has gone. I wonder if, maybe when I get older, I can find my rainbow and never have to brush it away. I guess we all need sort of rainbow to brighten our lives from time to time and to keep our hopes and dreams colorful. 本篇文章是一篇记叙文。 41. What did the writer want to do when his mother came home? A. To introduce Stephanie to her. B. To prevent her from seeing his painting. C. To put the materials back in the yard. D. To show his artwork to her. 答案:D 考点:: 解析:细节理解题。根据第四段“I saw a car in the distance and jumped up as I recognized the car. It was my mother. I couldn’t wait to show her my masterpiece. ”可知我当时是很高兴的想 把我的杰作给母亲看。所以选择 D。 42. In his mother’s eyes, the writer_______. A. was a born artist B. always caused trouble C. was a problem solver D. worked very hard 答案:B 考点:: 解析:推理判断题。根据第五、六段可以推断出,母亲特别反感作者的一些行为,所以很 生气,认为她是一个麻烦制造者。 43. The underlined word “rainbow” in the last paragraph refers to ______. A. the rainbow in the sky B. the stripes on the pavement C. something imaginative and fun D. important lessons learned in childhood 答案:C 考点:: 解析:推理判断题。根据最后一段作者的感慨“to keep our hopes and dreams colorful”,可 以推断此时作者提到的 rainbow 不是指自己曾经画过的街道上的彩虹,而是指自己的人生 中的彩虹。 44. It can be learned from the passage that parents should ________. A. encourage children to paint B. value friendship among children C. discover the hidden talent in children D. protect rather than destroy children’s dreams 答案:D 考点:: 解析:推理判断题。从整篇文章开头叙述作者的开心的画彩虹的过程,到后来被母亲要求 把街道清洗干净,到最后的感慨,表明作者写作的最终想要表达的目的之一是:父母应该 支持孩子的某些创造性的行为,而不是一味的否定。 B. www. ks*5u. co Below is a housing guide for students going to London. University accommodation (住所) offices Many university accommodation offices have their own list of registered landlords ( 房 东 ). Others also provide information on accommodation agencies and other housing organization. The advantage of using your university accommodation is that you can get support if you have a problem. The disadvantage is that they are unlikely to have enough registered landlords to houses all their students. Property papers: Loot and Renting Loot is an important source (来源) of information about private housing for co-renters. The 本篇是一篇应用文。介绍在伦敦留学的学生们住宿情况。 45. What is the advantage of using Loot? A. It has more offers from accommodation agencies than Renting. B. It gives you personal information about other co-renters. C. Their website is designed mainly for students. D. There are some good bargains. 答案:D 考点:: 解析:细节理解题。关于 Loot,选择相应的部分,仔细阅读,得知 Loot 的好处是:The advantage of using Loot is that there are some excellent bargains. 即价格方面好商量。故选择 D。 46. A good agent can help you ____. A. know more people B. find cheap accommodation C. get the right accommodation quickly D. get free information about most accommodations 答案:C 考点:: 解析:细节理解题。涉及到 agent,在“Accommodation agencies”部分寻找答案。根据“A good agent will listen to your requirements and can save you time in looking for the right accommodation. ”可知好的代理人,能听取意见,并节省时间寻找合适的住宿。故选择 C。 47. The information passed on by word of mouth is important because____. A. it is better than that found through any other source B. it helps you find some of the best housing never advertised C. the final year students always offer better information D. the landlords have little valuable information 答案:B 考点:: 解析:推理判断题。在“Word of mouth”部分寻找答案。口传的信息很重要是因为“is never advertised ;It might be that you can find out about good offers from final year students. ”但是也 有弊端,分析四个选项,只有 B 是正确的。 48. For students going to London for the first time, which of the following provides the most reliable information? A. University accommodation offices. B. Loot and Renting C. Noticeboards. D. Family. 答案:A 考点:: 解析:推理判断题。根据第一部分“The advantage of using your university accommodation is that you can get support if you have a problem. ”,而其他的都没有涉及到问题的解决这部分, 所以选择 A。 C. www. ks*5u. co The term “multitasking” originally referred to a computer’s ability to carry out several tasks at one time. For many people, multitasking has become a way of life and even a key to success. In fact, some excellent mental aerobic exercises (大脑训练) involve engaging the brain in two or more challenging activities at a time. Although checking e-mail while talking on a phone and reading the newspaper may be second nature for some people, many times multitasking can make us less productive, rather than more. And studies show that too much multitasking can lead to increased stress, anxiety and memory loss. In order to multitask, the brain uses an area known as the prefrontal cortex (前额叶脑皮 层). Brian scans of volunteers performing multiple tasks together show that as they shift from task to task, this front part of the brain actually takes a moment of rest between tasks. You may have experienced a prefrontal cortex “moment of rest” yourself if you’ve ever dialed (拨电话) a phone number and suddenly forgotten who you dialed when the line is answered. What probably occurred is that between the dialing and the answering, your mind shifted to anther thought or task, and then took that “moment” to come back. Research has also shown that for many volunteers, job efficiency (效率) declines while multitasking, as compared to when they perform only one task at a time. k*s5u Multitasking is easiest when at least one of the tasks is habitual, or requires little thought. Most people don’t find it difficult to eat and read the newspaper at the same time. However, when two or more attention-requiring tasks are attempted at one time, people sometimes make mistakes. We often don’t remember things as well when we’re trying to manage several details at the same time. Without mental focus, we may not pay enough attention to new information coming in, so it never makes it into our memory stores. That is one of the main reasons we forget people’s names---even sometimes right after they have introduced themselves. Multitasking can also affect our relationships. If someone checks their e-mail while on the phone with a friend, they may come off as absent-minded or disinterested. It can also cause that person to miss or overlook key information being passed on to them. 本篇文章是一篇说明文。 49. Why are some mental aerobic exercises designed to engage people in multitasking? A. To make them more productive. B. To reduce their stress and anxiety. C. To develop their communication skills. D. To help them perform daily tasks more easily. 答案:A 考点:: 解析:推理判断题。根据第一段的内容可知,作者提出:“In fact, some excellent mental aerobic exercises ( 大 脑 训 练 ) involve engaging the brain in two or more challenging activities at a time. ”,之后用 although 引出转折,这种训练有时并不能 productive。所以选择 A。有部 分人认为这种大脑的训练能够帮助他们高效的工作。 50. According to Paragraph 2, why may a person suddenly forget who has called? A. He may leave his prefrontal cortex temporarily damaged. B. He is probably interrupted by another task. C. He is probably not very familiar with the person he has called. D. He may need a rest between dialing and speaking. k*s5u 答案:B 考点:: 解析:细节理解题。主要根据“this front part of the brain actually takes a moment of rest between tasks”,可以推断出之后的“if you’ve ever dialed (拨电话) a phone number and suddenly forgotten who you dialed when the line is answered”的情况,所以选择 B。 51. People tend to make mistake when ____. A. they perform several challenging tasks at a time. B. new messages are processed one after another C. their relationships with others are affected D. the tasks require little thought 答案:A 考点:: 解析:细节理解题。根据第三段内容,特别是最后一句话可知当人在同时从事两项或多项 需要花费精力的工作时,常常会出错。故选择 A。 52. What is the main idea of the passage? A. Multitasking has become a way of life. B. Multitasking often leads to efficiency decline. C. Multitasking exercises need to be improved. D. Multitasking enables people to remember things better. 答案:B 考点:: 解析:主旨大意题。最后一段中作者继续描述了之前的观点:一个人同时从事多项工作会 出现很多问题。回到整篇文章,作者主要观点就是告诉大家:多项工作不能产生高效的结 果,故选择 B。k*s5u D. My family and I lived across the street from Southway Park since I was four years old. Then just last year they city put a chain link fence around the park and started bulldozing (用推 土机推平) the trees and grass to make way for a new apartment complex. When I saw the fence and bulldozers, I asked myself, “Why don’t they just leave it alone?” Looking back, I think what sentenced the part to oblivion (别遗忘) was the drought (旱灾) we had about four years ago. Up until then, Southway Park was a nice green park with plenty of trees and a public swimming pool. My friends and I rollerskated on the sidewalks, climbed the tress, and swam in the pool all the years I was growing up. The park was almost like my own yard. Then the summer I was fifteen the drought came and things changed. There had been almost no rain at all that year. The city stopped watering the park grass. Within a few weeks I found myself living across the street from a huge brown desert. Leaves fell off the park tress, and pretty soon the trees started dying, too. Next, the park swimming pool was closed. The city cut down on the work force that kept the park, and pretty soon it just got too ugly and dirty to enjoy anymore. As the drought lasted into the fall, the park got worse every month. The rubbish piled up or blew across the brown grass. Soon the only people in the park were beggars and other people down on their luck. People said drugs were being sold or traded there now. The park had gotten scary, and my mother told us kids not to go there anymore. The drought finally ended and things seemed to get back to normal, that is, everything but the park. It had gotten into such bad shape that the city just let it stay that way. Then about six months ago I heard that the city was going to “redevelop” certain worn-out areas of the city. It turned out that the city had planned to get rid of the park, sell the land and let someone build rows of apartment buildings on it. The chain-link fencing and the bulldozers did their work. Now we live across the street from six rows of apartment buildings. Each of them is three units high and stretches a block in each direction. The neighborhood has changed without the park. The streets I used to play in are jammed with cars now. Things will never be the same again. Sometimes I wonder, though, what changes another drought would make in the way things are today. 本文是一篇夹叙夹议的文章。 53. How did the writer feel when he saw the fence and bulldozers? k*s5u A. Scared. B. Confused. C. Upset. D. Curious. 答案:C 考点:: 解析:推理判断题。根据第一段叙述了解到作者从四岁开始住在 Southway Park 对面。从 第二段了解到作者小时候和朋友在公园里玩耍。而现在公园被围了起来,事实上,作者是 很失望、沮丧,自己不能去玩了,而且树都被砍掉了。 54. Why was the writer told not to go to the park by his mother? A. It was being rebuilt. B. It was dangerous. C. It because crowded. D. It had turned into a desert. 答案:B 考点:: 解析:推理判断题。仔细阅读第四段,可以了解到,公园被废弃了,里面住着流浪汉,甚 至有人贩毒,所以母亲叮嘱我们不要去那里,因为危险。k*s5u 55. According to the writer, what eventually brought about the disappearance of the park? A. The drought. B. The crime. C. The beggars and the rubbish. D. The decisions of the city. 答案:D 考点:: 解析:推理判断题。从第五段,可以明显的了解到:干旱结束了,但是政府仍然对公园不 管不问,而是要重新规划这块废弃地,在作者眼中是“the city had planned to get rid of the park”。故选择 D。 56. The last sentence of the passage implies that if another drought came, ______. A. the situation would be much worse B. people would have to desert their homes C. the city would be fully prepared in advance D. the city would have to redevelop the neighborhood 答案:A 考点:: 解析:推理判断题。最后一段作者描述了目前自己居住的街道的变化,变得更加拥挤了。 从而作者有了另一种思考:如果再来一次干旱,是否会造成如这次干旱一样的一些变化。 从而可推测作者是隐射,环境会变的更加糟糕。k*s5u E. www. ks*5u. co I needed to buy a digital camera, one that was simply good at taking good snaps (快照), maybe occasionally for magazines. Being the cautious type, I fancied a reliable brand. So I went on the net, spent 15 minutes reading product reviews on good websites, wrote down the names of three top recommendations and headed for my nearest big friendly camera store. There in the cupboard was one of the cameras on my list. And it was on special offer. Oh joy. I pointed at it and asked an assistant, “Can I have one of those?” He looked perturbed (不安). “Do you want to try it first?” he said. It didn’t quite sound like a question. “Do I need to?” I replied ,“There is nothing wrong with it?” This made him look a bit insulted and I started to feel bad. “No, no. But you should try it,” he said encouragingly. “Compare it with the others. ” I looked across at the others: shelves of similar cameras placed along the wall, offering a wide range of slightly different prices and discounts, with each company selling a range of models based around the same basic box. With so many models to choose from, it seemed that I would have to spend hours weighing X against Y, always trying to take Z and possibly H into account at the same time. But when I had finished, I would still have only the same two certainties that I had entered the store with: first, soon after I carried my new camera out of the shop, it would be worth half what I paid for it; and second, my wonderful camera would very quickly be replaced by a new model. But something in the human soul whispers that you can beat these traps by making the right choice, the clever choice, the wise choice. In the end, I agreed to try the model I had chosen. The assistant seemed a sincere man. So I let him take out of my chosen camera from cupboard, show how it took excellent pictures of my fellow shoppers… and when he started to introduce the special features, I interrupted to ask whether I needed to buy a carry-case and a memory card as well. Why do we think that new options(选择) still offer us anything new? Perhaps it is because they offer an opportunity to avoid facing the fact that our real choices in this culture are far more limited than we would like to imagine. 本文是一篇夹叙夹议的文章。 57. The shop assistant insisted that the writer should A. try the camera to see if there was anything wrong with it. B. compare the camera he had chosen with the others. C. get more information about different companies. k*s5u D. trust him and stop asking questions. 答案:B 考点:: 解析:细节理解题。根据第一段作者与推销员之间的对话,可以了解到,推销员一直在劝 说作者试一下机器,并且与其他的机器进行比较。故选择 B。 58. What does the writer mean by “it would be worth half what I paid for it ”(paragraph 2) A. He should get a 50% discount. B. The price of the camera was unreasonably high. C. The quality of the camera was not good. D. The camera would soon fall in value. 答案:D 考点:: 解析:推理判断题。第二段开头,作者分析了在卖场,自己将会被各种照相机的种类,弄 的不知如何选择。但是“But when I had finished, I would still have only the same two certainties that I had entered the store with”,最终我还是必须选择一款,但是无论选择哪一款都会有两 个必定的结果:(1)会立马贬值。(2)会很快有新的产品。所以选择 D。 59. The writer decided to try the model he had chosen because he A. knew very little about it. B. didn’t trust the shop assistant. k*s5u C. wanted to make sure the one he chose would be the best. D. had a special interest in taking pictures of his fellow shoppers. 答案:C 考点:: 解析:推理判断题。阅读第四段,可知作者经过内心的思量,明白最后还是得做决定,得 买一款,所以还是需要作出明智的决定,就是试一下自己最初选择那一款,从而确定自己 的选择。 60. I t can be inferred from the passage that in the writer’s opinion, . A. people waste too much money on cameras B. cameras have become an important part of our daily life C. we don’t actually need so many choices when buying a product D. famous companies care more about profit than quality 答案:C 考点:: 解析:推理判断题。最后一段作者分析了人们为什么老是喜欢新的事物,因为旧的事物我 们了解了,有局限性,而新的事物会带给我们更多我们没想到的。根据作者在购物的过程 中,最终选择了试用自己最初的照相机,所以得出作者的观点是:我们并不需要了解很多 新的东西,只要达到自己的最初的要求就行。 第二节:Peter, Helen, Catherine, Elizabeth 和 Levin 想根据各自在环保方面的兴趣(61-65) 进行案例研究。阅读下面某杂志的专题报道摘要(A、B、C、D、E 和 F),选出适合他们研究 的最佳案例,并在答题纸上相应选项的标号涂黑。选项中有一项是多余选项。www. ks*5u. co 61. Peter: Reducing plastic and other wastes through DIY. k*s5u 62. Helen: Making use of the heavy traffic to produce electricity. 63. Catherine: Building a community without private cars 64. Elizabeth: Building houses with recycled materials and energy-efficiency systems 65. Levin: Developing a new type of urban car which burns less gas A B Vauban We know cars are terrible polluters, but would you give yours up? Vauban, a community in southwestern Germany, did just that, and its 5,000 citizens are doing fine. Most streets are free of vehicles, and there are generous green spaces and good public-transport links, including fast buses and bicycle paths. When people must drive, they can turn to car-sharing clubs. “All the citizens had the chance to plan their own city,” says Andreas Delleke, an energy expert, “and it’s just how we wanted it to be. ” Denmark During the period of gas shortage in the early 70s, Denmark decided to become self-sufficient( 自 足 ). So they began a few projects making smart investments along the way. On the island of Samsoe, local families, fishermen and farmers bought wind turbines( 涡 轮 机 ) to produce their own energy, Within seven years these turbines were completely paid for. And can you believe just one of wind turbines produces enough electricity for 600 households? C D Trey Parker and Matt Stone Trey Parker and Matt Stone, creators of South Park, have built a sustainable(可持续的) castle with outer siding and inner flooring of recycled wood, recycled carpeting, high-efficiency boiler systems. “I think more and more today, people are willing to make a statement about the Earth and how they want to protect it,” Michael Ruth, home designer and builder says. “For high-end homes in this valley, this is entirely P-NUT Who doesn’t love the name P-NUT—short for Personal-Neo Urban Transport? It’s Honda’s latest attempt to create a tiny footprint for a new urban vehicle. This little P-NUT is unique. With a central driving position, the car is designed to move in tight settings. The 11-foot micro car will seat three with two rear-seat passengers behind the driver. “The P-NUT concept explores the packaging and design potential for a vehicle designed for the consistent with what they cost. ” city lifestyle. ” Said Dave Marek, a Honda design spokesman. E F Israel Company Is it possible that annoying rush hour traffic could become a source of renewable energy? Israel’s Technion Institute of Technology claims that if we placed special generator(发电 机) under roads, railways, and runways —we could harvest enough energy to mass-produce electricity. A trial process has been used on a smaller scale, in dance clubs for instance, where the pounding feet of dancers light up the floor. “We can produce electricity anywhere there is a busy road using energy that normally goes to waste,” said Uri Amit, chairman of Israel’s Technion Institute of Technology. Coffee Coffee. Some of us can’t start our day without it, and we don’t mind waiting 2016 minutes in line for it. Here is the most effective tip to make you a superstar in environment protection. Get a coffee machine for your home or coffee, or persuade your company into buying one. (Tell them it will improve productivity. ) Skip the coffee line on the way to work and make something that is better-tasting and much better for your wallet. Plus, you won’t need those plastic cups or carrying cases that just get thrown away. Better yet, use your favorite travel mug. 答案:61. F 62. E 63. A 64. C 65. D 考点:: 解析:首先了解每个人的兴趣是什么,然后分析 A-F 六个主题,从而可以得出 A:创建一种 车的俱乐部,大家共用,这样会降低污染。B:用风涡轮机制造能源。
查看更多

相关文章

您可能关注的文档